Anda di halaman 1dari 314

Co-ordinate geometry

Co-ordinate Geometry is one of the most scoring and


easy to prepare Unit in the entire preparation of IIT JEE,
AIEEE, DCE, EAMCET and other engineering entrance
examinations. It is of no doubt that along with Calculus,
Co-ordinate Geometry is the most scoring of all topics in
Mathematics.
The importance of Co-ordinate Geometry lies in the
fact that almost all the students who aspire to get high All
India rank in IIT JEE, AIEEE, DCE, EAMCET and other
engineering entrance examinations give a good emphasis
on Co-ordinate Geometry.
Co-ordinate Geometry is important not only from the
point of view of Mathematics but also from the point of
view of Physics and Physical Chemistry as it helps in
problems pertaining to graphs, especially Cartesian Coordinates.
Co-ordinate Geometry can be further divided into
many parts but The Straight Line and The Circle holds
high importance not only because more questions appear
from these topics but also from the point of view that
these two chapters are prerequisite to Conic Sections.
Conic Sections can be further divided into Parabola,
Ellipse and Hyperbola. It is well known that it is really
easy to score in Conic Sections as questions which come
every year has few fixed pattern and hence can be easily
dealt with.

Since Co-ordinate Geometry is not new to the students


as they do study some of its basics in earlier classes.
Hence, it is easy to pick up and a good take-off is not
difficult. This gives an early advantage to the students
not only in Mathematics but also in Physics and Physical
Chemistry.
There are few students who find Co-ordinate
Geometry a bit difficult but to these students we can
advise more and more practice.
Wishing you "All the Best" for the preparation of Coordinate Geometry with askIITians.
Topics Covered under Coordinate Geometry are:1.Straight lines
2.Circle
3.Parabola
4.Ellipse
5.Hyperbola
6.3D Geometry

Straight Lines
Straight Lines is a fundamental and an important topic under Coordinate
Geometry.

Several methods have been developed by mathematicians to uniquely locate


the position of the points in space. The easiest and most widely used one is
the Cartesian coordinate system, which is based on mutually perpendicular
axes. In this chapter you will learn about this system: locating the points in
this system and finding equations of line passing through these points.
If we say a line passes through two given points, we are imposing two
conditions on this line. The conditions can be imposed in several other
manners also e.g. we may say that a line passes through one point and is
perpendicular to another line. These two conditions are sufficient to uniquely
known our line. In this chapter you will also learn various manners of
imposing conditions and finding the equation of line under those conditions.
Having known the line uniquely in space we shall try to a lots timings with it
e.g. divide it in a given ratio, find distances of other points or lines from it,
find the angles, which this line make with some straight line.
Straight Lines is one of the easiest and important chapters of Co-ordinate
Geometry in the Mathematics syllabus of IIT JEE, AIEEE and other
engineering examinations. These laws are not new to any aspirant as he is
using
the
principles
even
in
day
to
day
life.
The chapter is important not only because it fetches 2-3 questions in most of
the engineering examination but also because it is prerequisite to the other
chapters of Co-ordinate Geometry.
Topics Covered:

Representation of points in a plane


Distance between two points
Centroid Incentre and Circum Centre
Area of a triangle
Standard equations of the Straight Line
Different Forms of line
Examples based on straight line
Angle between two straight lines
Equation of Locus
Examples on Angle Between two straight lines
Length of the Perpendicular from a Point on a Line
Distance between two parallel lines
Examples on distance between two parallel lines
Family of lines
Concurrency of Straight Lines

Position of two points with respect to a given line


Angle Bisectors
Pair of Straight Lines
Angle between pair of lines
Combined equations of the angle bisectors of the lines
Joint Equation of Pair of Lines

Introduction
Coordinate Geometry is the unification of algebra and geometry in which algebra is used in
the study of geometrical relations and geometrical figures are represented by means of
equations. The most popular coordinate system is the rectangular Cartesian system.
Coordinates of a point are the real variables associated in an order to describe its location in
space. Here we consider the space to be two-dimensional. Through a point O, referred to as
the origin, we take two mutually perpendicular lines XOX and YOY and call them x and y
axes respectively. The position of a point is completely determined with reference to these
axes of means of an ordered pair of real numbers (x, y) called the coordinates of P where |x|
and |y| are the distances of the point P from the y-axis and the x-axis respectively. X is called
the x-coordinate or the abscissa of P and y is called the y-coordinate or the ordinate of P.

Representation of points in a plane


We are familiar with the representation of real numbers on a line, which we call a real line. In
this representation we fix a point O (called origin) and represent a real number by a point A
on this line such that its distance OA (see figure given below) is equal to the value of real
number. In the left side of O we represent negative real numbers and in the right side of O
we represent positive real numbers. Thus, not only the magnitude of OA but the direction of
the line OA is also considered for representation.

Hence OA = AO
Similarly ordered pairs are represented in a plane. To represent an ordered pair (a, b) we
take two reference lines which are mutually perpendicular. The ordered pair (a, b) represents
in such a plane, by a point P(a, b) such that (see figure given below) OA = a and OB = b.

This system is called Cartesian co-ordinate system. Since elements of an ordered pair are
not inter changeable (i.e., (a, b) (b, a) unless a = b) so they are represented in particular
order, the first element a is represented on horizontal line called abscissa and the second
element b on a vertical line called ordinate. Like the real number notation the positive side
of the x-axis is the right side of O and positive side of O and positive side of y-axis is upper
side of O.

So, the two lines divide the region in 4 parts. These are called quadrants. These quadrants
are characterized as
I

quadrant

x > 0, y > 0

II

quadrant

x < 0, y > 0

III

quadrant

x < 0, y < 0

IV

quadrant

x > 0, y < 0

Here the point O represents x = 0 and y = 0, hence order pair becomes (0, 0).
There is a second type of representation called the polar co-ordinate system. In this system
a reference is fixed to a line (Called the initial line), and a point called the origin in the
system. Any point P is represented by ordered pair (r, ).
Such that

OP = r; The distance of point from origin.


and POX = The angular displacement of line OP from fixed line i.e. the initial line, (in
the anticlockwise direction)
Clearly a = r cos and b = r sin (see figure given below)

Distance between two points


The distance between two points P(x1, y1) and Q(x2, y2) is (see the figure
given below).

Length PQ = (x2 x1)2 + (y2 y1)2


Proof:

Let P(x1, y1) and Q(x2, y2) be the two points and let the distance between
them be d. Draw PA, QR parallel to y-axis and PR parallel to x-axis.
Angle QRP = 90o

d2 = PR2 + RQ2
d2 = (x2 x1)2 + (y2 y1)2

d = (x2 x1)2 + (y2 y1)2.


Section Formula

Let us say we want to know the co-ordinates of point which divides a line
segment between two points A(x1, y1) and B(x2, y2) in the ratio m : n.
The coordinates of such a point are given by
(nx1 + mx2/m+n, ny1 + my2/m+n) (for internal division)
Note:
This is called section formula.
Let P divide the line segment AB in the ratio m : n. If P is inside AB then it is
called internal division; if it is outside AB then it is called external division.
However in each case AP/BP [or AP'/BP' or AP"/BP"] = m/n.
Proof:
Consider ? ABB
Since BB||PQ and AP:PB = m:n (see figure given below)

AQ/AB' = PQ/BB' = m/m+n (= AP/AB)


x x1/x2 x1 = m/m+n
x = nx1 + mx2/m+n and y = ny1 + my2/m+n
If P is outside AB (less assume it is at P)
We have
x x1/x2 x1 = m/m+n
x = nx1 + mx2/m+n and y = ny1 + my2/m+n
Similarly if P is at P then
x = mx2+m+n/nm, y = my2+ny1/m+n
Note:

m:n can be written as m/n or :1. So any point on line joining A and B will
be P(x2+x1/+1.y2+y1/+1). It is useful to assume :1 because it involves
only one variable.
illustration:
Find the ratio in which line segment A(2, 1) and B(5, 2) is divided by x-axis.
Solution:

Let x-axis intersect line at point P(xp, 0) such that AP/BP = /1


yP = 0 = y2+1.ya/+1 = 2+(1)/+1 = 1/2
AP/BO = 1/2
Illustration:
Prove that altitudes of a triangle are concurrent and prove that the coordinates of the point of con-currency are
(x1 tan A + x2 tan B + x3 tan C/tan A + tan B + tan C, y1 tan A + y2 tan B +
y3 tan C/tan A + tan B + tan C),
Solution:
In triangle A(x1, y1), B(x2, y2) and C(x3, y3), draw AD perpendicular to BC. Our
effort now should be to find the co-ordinates of the point D.

To do that, we need to find BC/CD. (figure is given above)


tan B = AD/BD and tan C = AD/CD
BD/DC = tan C/tan B
Now we apply section formulae.
xD = x2 tan B + x3 tan C/tan B + tan C
yD = y2 tan B + y3 tan C/tan B + tan C
(ii)

(i)

We know that orthocenter will lie on AD. We need to find this point and its
co-ordinates.
We should select a point H1 on AD and take the ratio AH1/H1D in such a
manner so that xH1 and yH1 calculated form (i) should be symmetric in x1, x2,
x3, tan A, tan B and tan C. Think before you proceed
Let

AH1/H1D = tan B + tan C/tan A


xH1 = x1 tan A + x2 tan B + x3 tan C/tan A + tan B + tan C
and yH1 = y1 tan A + y2 tan B + y3 tan C/tan A + tan B + tan C

Since the result is symmetric, this point H1 will lie on other altitude as well
i.e. the altitudes are concurrent
xH = xH1 and yH = yH1

Illustration:
Prove analytically that in a right angled triangle the midpoint of the
hypotenuse is equidistant from the three angular points.
Solution:
While proving a problem analytically take most convenient co-ordinates of
known points.
In the present case triangle is assumed as AOB with coordinates as shown in
figure given below, C is midpoint of AB.

So co-ordinates of C will be (a/2, b/2)

Now AB = a2 + b2
CA = CB = AB/2 (C is mid point of AB)
= a2 + b2

and, we know that the distance between two points C and O is given by
CO = (a/2 0)2 + (b/2 0)2 = a2 + b2/2
Hence CA = CB = CO
Coordinates of the point P dividing the join of two points A(x1, y1) and
B(x2,
y2)
internally
in
the
given
= 1/2 areP(2x1+1x2/2+1, 2y1+1y2/2+1).

ratio

1 : 2 i.e.,

AP/BP

Coordinates of the point P dividing the join of two points A(x1, y1) and B(x2,
y2) externally in the ratio 1 : 2 i.e., Ao/BP = 1/2 areP(2x1+1x2/2
1, 2y1+1y2/21).

Centroid of Triangle
The centroid of a triangle is the point of concurrency of the medians. The centroid G of the
triangle ABC, divides the median AD, in the ratio of 2 : 1.
Illustration:
Find the centroid of the triangle the coordinates of whose vertices are given by A(x 1, y1),
B(x2, y2) and C(x3, y3) respectively.

Solution:
AG/AD = 2/1
Since D is the midpoint of BC, coordinates of D are (x 2+x3/2, y2+y3/2)
Using the section formula, the coordinates of G are
(2(x2+x3/2)+1.x1/2+1, 2(y2+y3/2)+1.y1/2+1)
Coordinates of G are (x1+x2+x3/3, y1+y2+y3/3).
Incentre of Triangle
The incentre I of a triangle is the point of concurrency of the bisectors of the angles of the
triangle.
Illustration:
Find the incentre of the triangle the coordinates of whose vertices are given by A(x 1, y1),
B(x2, y2), C(x3, y3).

Solution:
By geometry, we know that BD/DC = AB/AC (since AD bisects A).
If the lengths of the sides AB, BC and AC are c, a and b respectively, then BD/DC = AB/AC =
c/b.
Coordinates of D are (bx2+cx3/b+c, by2+cy3/b+c)
IB bisects B. Hence ID/IA = BD/BA = (ac/b+c)/c = a/c+b.

Let the coordinates of I be (x, y).

Then x = ax1+bx2+cx3/a+b+c, y = ay1+by2+cy3/a+b+c.


Circum Centre of Triangle
This the point of concurrency of the perpendicular bisectors of the sides of the triangle. This
is also the centre of the circle, passing through the vertices of the given triangle.

Orthocentre of Triangle
This is the point of concurrency of the altitudes of the triangle.
Excentre
Excentre of a triangle is the point of concurrency of bisectors of two exterior and third
interior angle. Hence there are three excentres I1, I2 and I3 opposite to three vertices of a
triangle.
If A(x1, y1), B(x2, y2) and C(x3, y3) are the vertices of a triangle ABC,

coordinates of centre of ex-circle opposite to vertex A are given as


I1(x, y) = (ax1+bx2+cx3/a+b+c/a+b+c, ay1+by2+cy3/a+b+c).
Similarly co-ordinates of centre of I2(x, y) and I3(x, y) are
I2(x, y) = (ax1bx2+cx3/ab+c, ay1by2+cy3/ab+c)

I3(x, y) = (ax1+bx2cx3/a+bc, ay1+by2cy3/a+bc)

Area of a triangle
Let (x1, y1), (x2, y2) and (x3, y3) respectively be the coordinates of the vertices
A, B, C of a triangle ABC. Then the area of triangle ABC, is

|1/2[x1(y2 y3) + x2(y3 + y1) + x3(y1 y2)]| =

It follows that the three points (x1, y1), (x2, y2) and (x3, y3) will be collinear

if

= 0.

Area of a polygon of n sides


First of all we plot the points and see their actual order. Let A1(x1, y1),
A(x2, y2), , An(xn, yn) be the vertices of the polygon in anticlockwise order.
Then area of the polygon =
.
Illustration:

Calculate area of a triangle shown in figures given below.

Solution:

Using the just derived formula


Area of a triangle
1/2 [3(2 4) + (1) (4 6) + 5 (6 2)]
= 1/2 [ 6 2 + 20] = 6

Similarly, area of the triangle shown in the figure given above.


Area of a ?ABC is
= 1/2 [3(4 2) + (5) (2 6) + (1) (6 4)]
=1/2 [6 20 + 2] = 6
Caution:
Thus we observe that the area of a triangle is positive vertices are taken in
the anticlockwise direction and negative when the vertices are taken the
clockwise direction.

Note:

Area of a triangle can also be expressed as = 1/2


= 1/2 [x1 y2 y1 x2 + x2 y3 y2 x3 + x3 y1 y3 x1]
This form is important. It can be used to find area of a quadrilateral,
pentagon, hexagon and polygons.
Important:

If three points P1, P2 and P3 are collinear then the determinant at must vanish
i.e. the area of triangle formed must be zero.
Note:
If the vertices are in clockwise order then take modulus.
Illustration:

Prove that the area of the triangle with vertices at (p 4, p + 5),


(p + 3, p 2) and (p, p) remains constant as p varies.
Solution:

The area of the triangle is

which remains constant for all values of p.

Straight Line
Any equation of first degree of the form Ax + By + C = 0, where A, B, C are
constants always represents a straight line (at least one out of A and B is non
zero).
Slope
If is the angle at which a straight line is inclined to the positive direction of
the x-axis, then m = tan, (0 < < 180o) is the slope of the line.

Standard equations of the Straight Line


Slope Intercept From:
y = mx + c, where
m = slope of the line
c = y intercept

Intercept Form:
x/a + y/b = 1
x intercept = a
y intercept = b

Slope point form


(a)

One point on the straight line

(b)

The gradient of the straight line i.e., the slope m of the line

Equation:
y y1 = m(x x1), where (x1, y1) is a point on the straight line.
Illustration:

Pause:
Equation of line in figure (ii) is x = 3, because x-co-ordinate of each point on
the line is 3.
Equation of line in figure (iv) is y = 2, because y-co-ordinate of each point on
the line is 2.
Although every line satisfied the above given basic definition, a line can be
represented in many forms, some of which are given hereunder.
Two points form:
Let there be two points A(x1, y1) and B(x2, y2) in a co-ordinate plane. If any
point P(x, y) lies on the line joining A and b then m = tan = yy1/xx2 = y2
y1/x2 x1, (see figure given below).

y y1 = y2 y1/x2 x1 (x x1) which is the equation of the given line.


Equation of line can also be written as

y y2 = y2 y1/x2 x1 (x x2) or

= 0.

Different Forms of line


Illustration:
The ends of a rod of length l move on two mutually perpendicular lines. Find the locus of the
point on the rod, which divides it in the ratio 2 : 1.
Solution:
Suppose the two perpendicular lines are x = 0 and y = 0 and let the end of the rod lie at the
point (0, a) and (b, 0).

The point P has coordinates given by


h = b+2.0/2+1, k = 2.a+1.0/2+1 a = 3k/2, b = 3h.
Also l2 = a2 + b2.
l2 = (3k/2)2 + (3h)2.

Thus the required locus is


x2 + y2/4 = l2/9, which represents an ellipse.
Similarly, take the ratio AP : BP as 2 : 1 and proceed. We get the result as y 2 + x2/4 = l2/9.

Illustration:
Find the locus of the point of intersection of the liens xcos + ysin = a and xsin ycos =
b. where is a variable.
Solution:
Let P(h, k) be the point of intersection of the given lines.
Then hcos + ksin = a.
hsin kcos = b.

(1)
(2)

Here a is a variable. So we have to eliminate a.


Squaring and adding (1) and (2),
We get, (hcos + ksin)2 + (hsin kcos)2 = a2 + b2
h2 + k2 = a2 + b2.
Hence locus of (h, k) is x2 + y2 = a2 + b2.
Two intercept form:
If intercepts of a line on x and y-axis are known then equation of the line can also be found
in two-intercept form. Intercepts are OA and OB on x and y-axis respectively, where A(a, 0)
and B(0, b) are two points through which line is passing. Treating it as a special case of twopoint form, one can write a unique equation of the line as
y0/xa = 0b/a0, where P(x, y) is any point on the line (figure given below)

y/b = x/a + 1.
x/a + y/b = 1.
This is intercept from of the equation of a straight line.

Parametric form:
Consider line PQ with points Q(x1, y1). Then Co-ordinates of any points P(x, y) are
x = x1 + r cos (see figure given below)

y = y1 + r sin
Equation of the line is obtained as follows: xx1/cos = yy1/sin = r
This is parametric form of the equation of a straight line.
Note:
That tan = m = slope of line.
Normal form:
Consider line l as shown in figure given below

ON l and |ON| = p
We have in ?ONA
OA = p/cos

A and B are intercept points of line l. So intercepts on x and


y-axes are p/cos and p/sin respectively. So equation of line l will be (both intercept form)
x cos /p + y sin /p = 1
x cos + y sin = p
This is the equation of a straight line in normal form, where p is perpendicular distance of
the line from origin.
Caution:
tan a m (slope of line)
Note:
P is always measured away from the origin and is always positive in value, a is a positive
angle less than 360o measured from the positive direction OX of the x-axis to the normal
from the origin to the line.
General form:
The general form of the equation of a straight line is
Ax + By+ C = 0

This equation can be reduced to any of the above forms with some rearrangements.

Examples based on straight line


Illustration:
Reduce 3x 4y + 5 = 0 to all other forms.
Solution:
(a) Slope intercept form
y = 3/4 x + 5/4
where, m = slope = 3/4
c = 5/4 (y intercept)
(b)

Intercept form
3x 4y = 5
3x/5 + 4y/5 = 1

x/5/3 + y/5/4 = 1 (intercept form)


x intercept = 5/3
y intercept = 5/4
(c)

point slope form


Let x = 1, then y = 3/4 + 5/4 = 2
y 2 = 3/4 (x 1)

(d)

parametric form
x1/cos = r, where tan = 3/4

x1/4/5 = y2/3/5 = r
(e)

Normal form

3x 4y = 5
3x + 4y = 5
Dividing by ((3)2 + 4)
3/5 x + 4/5 y = 5/5 = 1
x cos + y sin = p, p > 0
Where cos = 3/5, sin = 4/5, p = 1
Illustration:
The straight line drawn through the point P(0, 3) and making an angle of
30o with positive x-axis, meets the line x + y = 6 at Q. Find the length PQ.
Solution:
Method 1.
Equation of the line through the point P is

x0/cos 30 = y 3/sin 30 = r
xQ = 3/r r, yQ = 3 + r/2, r = distance PQ
Point Q lies on x + y = 6
r3/2 + (3 + r/2) = 6
r = 6/3+1
Method 2.
Equation of the line through the point P is y = 1/3 x + 3
[because here m = tan 30o = 1/3, c = 3]
Solving this line, with x + y = 6, we get
xQ = 33/3+1, yQ = 33+6/3+1
distance PQ = (xQ xP)2 + (yQ yP)2
= 6/3+1
Illustration:
Find the equation of the line whose perpendicular distance from the origin is
4 units and the angle which the normal makes with positive direction of xaxis is 15o.
Solution:
Here, we are given p = 4 and = 15o.
Now cos 15o = 3+1/22
and sin 15o = 31/22
By the normal form, the equation of the line is
x cos 15o + y sin 15o = 4 or 3+1/22 x + 31/22 y = 4
or (3 + 1) x + (3 1)y = 82.
This is the required equation.

Illustration:
Given a line 2x 3y + 5 = 0. Write various forms of the line.
Solution:
Slope intercept form:
y = 2x/3 + 5/3, C = 5/3

and m = (coeffcient of x/coefficient of y) = 2/3


Intercept Form:
x/(5/3) + y/(5/3) = 1, a = 5/2; b = + 5/3.
Normal Form:
sin = 3/13, cos = 213
2/13 p = 5/13.
Illustration:
Find the equation to the straight line which passes through the point (5, 4)
and is such that the position of it between the axes is divided by the give
point in the ratio 1 : 2.
Solution:
Let the required straight line be (x/a) + (y/b) = 1.
Using the given conditions, P (2a+1.0/2+1, 2.0+1.b/2+1) is the point which
divides (a, 0) and (0, b) internally in the ratio 1 : 2.
But P is (5/4)
Hence 5 = 2a/3, 4 = b/3 a = 15/2, b = 12.
Hence the required equation is x/(15/2) + y/2 < < .
tan = 8 = slope of line.
We know that the equation of the straight line passing through the point (x1,
y1) having slope m is y y1 = m(x x1).
Therefore the equation of the required line is y 2 = 8 (x 1)
8 x + y 8 2 = 0.

Illustration:
Find the equation of the line joining the points (1, 3) and (4, 2).
Solution:
Equation of the line passing through the points (x1, y1) and (x2, y2) is
y y1 = y1y2/x1x2 (x x1)
Hence equation of the required line will be
y 3 = 3+2/14 (x + 1) x + y 2 = 0.

Illustration:
Represent the straight-line y = x + 2 in the parametric form.
Solution:
Slope of the given line is = 1 = tan /4.
Equation of the straight line can be written as y 2 = x.
or y2/1/2 = x/1/2= r.
Any point on the line is (r/2, 2 + r/2).
The point (x, y) is at a distance r from the point (0, 2).

Illustration:
A line joining two points A(2, 0) and B(3, 1) is rotated about A in the
anticlockwise direction through an angle of 15o. Find the equation of the line
in the new position. If B goes to C, what will be the coordinates of C, in the
new position?
Solution:
Slope of BAB(m) = 1 m = tan = 1 = 45o.
= tan (60o) (because angle between AB and AC = 15o).
Also AB = AC = 2 and A is (2, 0).

Hence equation of the line AC is


x2/cos60o = y0/sin 60o
or x2/1/2 = y/3/2 = r = 2
C is (2 + 2.1/2, 0 + 2.3/2) i.e. C is (2 + 1/2, 3/2).

Angle between two straight lines


Angle bisector of two lines (the line which bisects the angle between the two
lines) is the locus of a point which is equidistant (having equal perpendicular
distance) from the two lines.

Say, we have two lines


L1 : A1x + B1y + C1 = 0
L2 : A2x + B2y + C2 = 0
If point R(p, q) lies on the bisector, then length of perpendicular from P to be
both lines should be equal.

i.e.
generalizing for any point (x, y) the equation of the angle bisector is obtained
as:
A1x+B1y+C1/A12 + B12 = + A2x+B2y+C2/A22 + B22

Note:
1. This equation gives two bisectors: one-acute angle bisector and the other
obtuse bisector.

2. Rule to decide the particular bisector

To determine a bisector which lies in the same relative position with respect
to the lines as a given point S(x 3, y3) does, make the signs of the expressions
A2x3 +
B1y3+
C1 and
A2x3 +
B2y3 +
C2 identical.
(say
positive)
2
2
2
2
then A1x+B1y+C1/A1 + B1 = +A2x+B2y+C2/A2 + B2 gives the bisector
towards this point. If the signs are different multiply one of the equations
with 1 throughout, so that positive sign is obtained. Then above equation
with changed equations of lines will given the required bisector.

3. If (x3, y3) (0, 0) and A2A1 + B2B1 > 0 then the bisector towards the origin
is the obtuse angle bisector.

4. Alternative method: to determine whether the bisector is of the obtuse or


acute angle, determines both the bisectors and calculate angle between one
of them and the initial line. The bisector for which |tan | > 1 is the obtuse
angle bisector.

Locus
If a point moves according to some fixed rule, its co-ordinates will always
satisfy some algebraic relation corresponding to the fixed rule. The resulting
path (a curve) of the moving point is called the locus of the point. The locus
i.e. the curve now contains all the points satisfying the specified condition
and no point outside the curve satisfies the condition.
When a point moves in a plane under certain geometrical conditions, the
point traces out a path. This path of the moving point is called its locus.

Equation of Locus
The equation to the locus is the relation which exists between the
coordinates of all the point on the path, and which holds for no other points
except those lying on the path.
Procedure for finding the equation of the locus of a point
(i) If we are finding the equation of the locus of a point P, assign coordinates
(h, k) to P.
(ii) Express the given conditions as equations in terms of the known
quantities and unknown parameters.

(iii) Eliminate the parameters, so that the eliminant contains only h, k and
known quantities.
(iv) Replace h by x, and k by y, in the eliminant. The resulting equation is the
equation
of
the
locus
of
p.
The problem of determining the equation of locus of points every pair of
which has constant slope. (see figure given below)

Slope is the tangent (i.e. tan q) of the angle made by a line with the positive
x-axis (remember positive) taken in anticlockwise direction from x-axis to the
line. For any two points P(x1, y1) and Q(x2, y2).

Examples on Angle Between two straight


lines.
Illustration:
Draw the lines 3x + 4y 12 = 0 and 5x + 12y + 13 = 0. Find the equation of the bisector of
the angle containing the origin. Also find the acute angle bisector and obtuse angle bisector.

Solution:

Let us make the expression on the left-hand side of the given equations of the same sign
or + ve. After substituting x = 0 and
y = 0.
L.H.S. of (i) is 3.0 + 4.0 12 = 12 = ve
R.H.S. of (ii) is 5.0 + 12.0 + 13 = 13 = + ve
So, multiply equation (i) by (1), we get
3x 4y + 12 = 0

(1)

Equation of the bisector of the angle containing origin is given by +ve sign i.e. 3x
4y+12/5 = + 5x+12y+13/13
64x + 112y 91 = 0

(3)

Again, the given lines are


3x 4y + 12 = 0

(1)

5x + 12y + 13 = 0

(2)

To find out whether this is an acute angle bisector or obtuse angle bisector, let us find the
sign of a1 a2 + b1 b2 from equation (1) and equation (2).
a1 a2 + b1 b2
= (3) (5) + (4) (12) = 15 48 = 63 = ve
the bisector containing the origin is the acute angle bisector.
Now, For obtuse angle bisector, we take ve origin.
i.e. 3x 4y+12/5 = + 5x+12y+13/13
i.e. 14x 8y 221 = 0

(4)

Well, to confirm all this, let us find angle between one of the lines and one of the bisectors
i.e.
5x + 12y + 13 = 0

(2)

64x + 112y 91 = 0

(3)

Slope of line (2) is m2 = 5/12


Slope of line (3) is m3 = 64/112
Let q be the angle between these two lines

tan =

<1

64x + 112y 91 = 0 is an acute angle bisector.

If is the angle between two lines, then tan =

where m1 and m2 are the slopes of the two lines.


(i)

If the two lines are perpendicular to each other then m1m2 = 1.

Any line perpendicular to ax + by + c = 0 is of the form


bx ay + k = 0.
(ii)

If the two lines are parallel or are coincident, then m1 = m2.

Any line parallel to ax + by + c=0 is of the form ax ay + k=0.


Let there be two-lines l1 and l2 with slopes m1 and m2 respectively. So tan = m1, tan =
m2 Angle between them is either
or ( ) depending on the side on considers

Now, tan (a b) = tan tan /1+tan tan


tan () = m1+m2/1+m1m2

( = say)

Since lines can be taken in any order and


tan( ) = tan . So only the magnitude of can be obtained.
Further tan ( ) = tan .
Since magnitude also includes the other angle i.e.
Supplementary angle. So is given by

tan acute =
Important:
1.

If lines are parallel


tan = 0 m1 = m2

2.

If lines are perpendicular


tan = tan (/2) =
1 + m1 m2 = 0 m1 m2 = 1

3.
Equation of a line parallel to y = mx + c is y = mx + k, i.e. Equation of a line parallel to
ax + by + c = 0 is ax + by + k = 0
4.
Equation of a line perpendicular to y = mx + c is y = 1/m x + k i.e. Equation of a line
perpendicular to ax + by + c = 0 is
bx ay + k = 0

5.

Lines

a1x + b1y + c1 = 0
a2x + b2y + c2 = 0

(i)
(ii)

represents
(i)

intersecting lines if a1/a2 b1/b2

(ii)

parallel lines if a1/a2 = b1/b2

(iii)

Coincident lines if a1/a2 = b1/b2 = c1/c2

Length of the Perpendicular from a Point


on a Line
The distance of a point from a line is the length of the perpendicular drawn from the point on
the line. Given the equation of the line are different forms, the length of the perpendicular
can be obtained in different forms.
First form:
The normal equation helps us in finding the distance of a point from a straight line. Suppose
we have to find the distance of the point P(x 1, y1) from the line l1 whose equation is x cos +
y sin = p. Let l2 be the line through P parallel to the line l 1. Let d be the distance of P from
l1. Then, the normal from O to l2 is of length p + d. Hence the equation of l2 is
x cos + y sin = p + d.
Since P(x1, y1) lies on it.
x1 cos + y1 sin = p + d
d = x1 cos + y1 sin p.
Note:
1.
Rule to find the perpendicular distance of a given point from a given line in normal
form.
In the left side of the equation (right side being zero), substitute the coordinates of the
point. The result gives the perpendicular distance.

2.
Complete distance formula. If the point P and the origin O, instead of lying on the
opposite sides of l as in figure given above, lie on the same side of line l 1 it may be proved
by proceeding exactly in the same manner that

d = (x1 cos + y1 sin p)


Hence, the complete distance formula is
d = + (x1 cos + y1 sin p)

The distance of a point from a line is the length of the perpendicular drawn from the point to
the line. Let L : Ax + By + C = 0 be a line, whose distance from the point P(x 1, y1) is d. Draw
a perpendicular PM from the point P to the line L. If the line meets the x and y-axes at the
points Q and R respectively, then coordinates of the points are Q(C/A, 0) and R(0, C/B).
Thus the area of the triangle PQR is given by
area(?PQR) = 1/2 PM QR, which gives PM = 2 are (PQR)/QR
also, area (?PQR) = 1/2 |x1 (0 + C/B) + (C/A) (C/B y1) + 0(y1 0)|
= 1/2 |x1 C/B + y1 C/A + C2/AB|

(1)

Or 2 area (?PQR) = |C/AB| |Ax1 + By1 + C| and


QR = (0 + C/A)2 + (C/B 0)2 = |C/AB| A2 + B2
Substituting the values of area (?PQR) and QR in (1), we get
PM = |Ax1+By1+C|/A2+B2.
Or d = |Ax1+By1+C|/A2+B2.
Thus the perpendicular distance (d) of a line Ax + By + C = 0 from a point
(x1, y1) is given by d = |Ax1+By1+C|/A2+B2.

Distance between two parallel lines


We know that slopes of two parallel lines are equal. Therefore, two parallel lines can be
taken in the form
y = mx + c1

(1)

and y = mx + c2

(2)

Line (1) will intersect x-axis at the point A (c1/m, 0) as shown in figure.

Distance between two lines is equal to the length of the perpendicular from point A to line
(2). Therefore, distance between the lines (1) and 92) is
|(m)(c1/m)+(c2)|/1+m2 or d = |c1c2|/1+m2.
Thus the distance d between two parallel liens y = mx + c 1 and y + mx + c2 is given by d = |
C1C2|/A2+B2.

Illustration:
Find the distance of the point (3, 5) from the line 3x 4y 26 = 0.
Solution:
Given line is 3x 4y 26 = 0.

(1)

Comparing (1) with general equation of line Ax + By + C = 0, we get


A = 3, B = 4 and C = 26.
Given point is (x1, y1) = (3, 5). The distance of the given point from given line is d = |
Ax1+By1+C|/A2+B2 = 3/5.

Illustration:
Find the distance between the parallel lines 3x 4y + 7 = 0 and
3x 4y + 5 = 0.
Solution:
Here A = 3, B = 4, C1 = 7 and C2 = 5.

Therefore, the required distance is d = |75|/32+(4)2 = 2/5.

Illustration:
The coordinates of the vertices A, B, C of a triangle are (6, 3), (3, 5) and (4,2) respectively
and P is any point (x, y). Show that the ratio of the areas of the triangles PBC and ABC is |x +
y 2| : 7.
Solution:
Equation of the line BC is x + y 2 = 0. Let PG and AD be perpendiculars from p and A on
BC.

Ratio of areas of triangles PBC and ABC is


1/2.BC.PG/1/2.BC.AD = PG/AD. But PG is the length of the perpendicular form P(x, y) on
x + y 2 = 0 and AD is the length of the perpendicular from A(6, 3) on x + y 2 = 0.
Ratio of the areas of ?PBC and ?ABC.
|x+y2/2|/|6+32/2| = |x+y2/7|.

Illustration:
Find the distance from the line 3x 4y + 35 = 0 of the point (0, 0).
Solution:
Writing the given equation in the normal form, we get
3/5 x + 4/5 y 7 = 0
Substituting x = 0, y = 0 in it, d = 3/5 (0) + 4/5 (0) 7 = 7
Changing sign the required distance = 7.
Second form:

To find the perpendicular distance of the point (x1, y1) from the line
ax + by + c = 0.
Let us convert the given equation in the normal form.
a/a2+b2 x + b/a2+b2 y + c/a2+b2 = 0
The perpendicular Distance of (x1, y1) is
d = a/a2+b2 x1 + b/a2+b2 y1 + c/a2+b2
d = ax1+by1+c/a2+b2
This formula can also be obtained independently as under:
Let PM be the perpendicular form P on AB. Then coordinates of A and B are
(c/a, 0) and (0, c/b) respectively,
AB = c2/a2+c2/b2
= c/ab (a2+b2)
Area of ?PAB (Recall from page M4 M 4)
= 1/2 [x1 (c/b 0)+0(0 y1) c/a (y1 + c/b)]
= c/2ab (ax1 + by1 + c)

(1)

Also, area of ?PAB = PM.AB


= 1/2 PM.c/ab (a2+b2)
From (1) and (2), we have
1/2 PM.c/ab (a2+b2) (ax1 + by1 + c)

(2)

PM = ax1+by1+c/a2+b2
Neglecting the negative sign, as the length of a segment is always positive, we have

PM = |ax1+by1+c/a2+b2|

Note:
1.

Actually
d = + ax1+by1+c/a2+b2

2.
To find the distance of a point from the given line, in the left side of the equation (right
side being zero) substitute the co-ordinates of the point, and divide the result
by (coefficient of x)2 + (coefficient of y)2
3.
When to use complete perpendicular Distance formula? The complete perpendicular
Distance formula is used when the length of the perpendicular to the line is given.

Illustration:
Find the
x y = 5.

distance

of

the

point

P(2,

3)

from

the

line

AB

which

is

Solution:
The equation of the line is
x y 5 = 0 [Making right side zero (note this step)]
Perpendicular Distance of the point (2, 3)
= (2)35/(1)2+(1)2 = 10/2
= 52
Changing the sign, perpendicular Distance in magnitude = 52.

Enquiry:
We can now find the distance of a point form a line but how can we
determine as to which side of the line does the point lie?
From the figure, we observe that
ax0 + by0 + c = 0

(? point (x0, y0) lies on the line)

(1)

Consider, ax1 + by1 + c


= (ax0 + c) + by1

[x0 = x1 = x2]

= b(y1 y0)
= ve
Consider, ax2 + by2 + c
= (ax0 + c) + by2
= b(y2 y0)
= +ve
Thus we observe that the point is on one side of the line, if put in the expression of line is
gives one sign, while the point is on the other side of the line, if put in the expression of line
it gives opposite sign.

Illustration:
Final the condition so that the points (x 1, y1) and (x2, y2) lie on the same side, of the line ax +
by + c =
Solution:
Since, (ax1 + by1 + c) and (ax2 + by2 + c)
Should be of the same sign.
Their product should be positive i.e.
(ax1+by1+c)(ax2+by2+c) > 0, which is the required condition.

Family of lines
The general equation of the family of lines through the point of intersection of two given
lines is L + L = 0, where L = 0 and L = 0 are the two given lines, and is a parameter.
Illustration:
A
variable
line
through
the
point
of
intersection
of
the
lines
x/a + y/b = 1 and x/b + y/a = 1 meets the coordinate axes in A and B. Show that the locus
of the midpoint of AB is the curve 2xy(a + b) = ab(x + y).
Solution:
Let (h, k) be the midpoint of the variable line AB.
The equation of the variable line AB is
(bx + ay ab) + (ax + by ab) = 0

Coordinates of A are (ab(1+)b+a).


Coordinate of B are (0, ab(1+)b+a).
Mid point of AB is (ab(1+)2(b+a), ab(1+)2(a+b))
h = ab(1+)2(b+a); k = ab(1+)2(a+b) 1/2h = b+a/ab(1+); 1/2k =
a+b/ab(1+)
1/2h + 1/2k = a+b/ab (h + k)ab = 2hk (a + b).

Hence the locus of the midpoint of AB is (x + y) ab = 2xy (a + b).


To find the equation to the straight lines which pass through a given point (x 1, y1) and make
equal angles with the given straight line y = m1x + c.
If m is the slope of the required line and is the angle which this line makes with the given
line, then tana = + m1m/1+m1m.

(i)

The above expression for tan, gives two values of m, say mA and mB.

(ii)
The required equations of the lines through the point (x 1, y1) and making equal
angles with the given line are y y1 = mA (x x1), y y1 = mB(x x1).
Illustration:
Find the equations to the sides of an isosceles right-angled triangle, the equation of whose
hypotenuse is 3x + 4y = 4 and the opposite vertex is the point (2, 2).
Solution:
The problem can be restated as:
Find the equations to the straight lines passing through the given point (2, 2) and
making equal angles of 45o with the given straight line 3x + 4y 4 = 0
Slope of the line 3x + 4y 4 = 0 is m1 = 3/4

tan 45o = + m1m/1+m1m, i.e., 1 = + m+3/4/13/4m


So that mA = 1/7,

and mB = 7.

Hence the required equations of the two lines are


y 2 = mA (x 2) and y 2 = mB(x 2).
7y x 12 = 0 and 7x + y = 16.
Illustration:
The straight lines 3x + 4y = 5 and 4x 3y = 15 intersect at the point A. On these lines
points B and C are chosen so that AB = AC. Find the possible equations of the line BC
passing through (1, 2).
Solution:
The two given straight lines are at right angles.
Since AB = AC, the triangle is an isosceles right angled triangle.
The required equation is of the form y 2 = m(x 1)

(1)

with tan 45o = + m+3/4/13m/4 = + m4/3/1+4m/3


1 = + m+3/4/13m/4 = + m4/3/1+4m/3 m = 7, 1/7.
Substitute the value of m in (1). We get the required equations.
Illustration:
Find the equation of the straight line passing through (2, 7) and having intercept of length 3
units
between
the
straight
lines
4x + 3y = 12 and 4x + 3y = 3.

Solution:
Distance AC between the two given parallel lines
= |c1c2/a2+b2| = 123/16+9 = 9/5.

Let AB be the intercept of length 3 units.


BC = 12/5. If is the angle between BC and AB, then tan = 9/12 = 3/4.
Slope of the parallel lines = 4/3 = m2.
If m1 is the slope of the required line, then tan = m1m/1+m1m 3/4 = + m14/3/1+4/3m1
i.e. m1 + 4/3 = 3/4 (1 4/3 m1) and m1 + 4/3 = 3/4 (1 4/3 m1).
The slopes are
(i) m1 = 7/24
(ii) m1 =
(the line is parallel to the y-axis).
The required
x + 2 = 0.

equations

of

the

lines

are

7x

24y

182

and

Alternative solution:
Equation of the line, through P(2, 7) and making angle with the x-axis, is x+2/cos =
y+7/sin = r.
If this line intersects the given lines at A and B, with AB = 3, the points A and B are A(2 +
r1 cos , 7 + r1 sin ) and B (2 + (r1 + 3) cos , 7 + (r1 + 3) sin ).
Since A and B lie on the lines 4x + 3y = 3 and 4x + 3y = 12, we have

4r1 cos + 3r1 sin = 32 and


4r1 cos + 3r1 sin + 12 cos + 9 sin = 41, so that
12 cos + 9 sin = 0 or 4 cos + 3 sin = 3.
Solving this equation we find that = /2 and tan = 7/24.
Hence the required lines are x + 2 = 0

and y + 7 = 7/24 (x + 2) i.e. 7x + 24y + 182 = 0.

Concurrency of Straight Lines


The condition for 3 lines a1x + b1y + c1 = 0, a2x + b2y + c2 = 0,
a3x + b3y + c3 = 0 to be concurrent is

(i)

= 0.

(ii) There exist 3 constants l, m, n (not all zero at the same time) such that IL 1 + mL2 +
nL3 = 0, where L1 = 0, L2 = 0 and L3 = 0 are the three given straight lines.
(iii) The three lines are concurrent if any one of the lines passes through the point of
intersection of the other two lines.
Illustration:
Check if lines
a1 x + b1 y + c1 = 0

(1)

a2 x + b2 y + c2 = 0

(2)

(2a1 3a2)x + (2b1 3b2)y + (2c1 3c2) = 0

(3)

are concurrent?
Solution:
We can try to find , and by observation as follow:
L3 2L1 + 3L2 = 0
Enquiry: many lines can pass through the intersection of two lines. Can we find
a general equation of these lines?

If L1 = 0 and L2 = 0 are two lines then equation of family of lines passing through their
intersection is given by
L1 + L2 = 0

(A)

Where is any parameter. (Equation A is satisfied by the point of intersection of L 1= 0 and


L2 = 0)
Note:
To determine a particular line one more condition is required so as to determine or
eliminate .

Illustration:
If x (2q + p) + y(3q + p) = 0
(x + y 1) + q/p (2x + 3y 1) = 0, p 0
Solution:
This equation represents the family of lines passing through the intersection of lines x + y
1 = 0 and 2x + 3y 1 = which is fixed point i.e. (2, 1).
If p = 0 then equation becomes
q(2x + 3y 1) = 0
this also represents a line which passes through fixed points (2, 1).
Hence the given equation represents family of lines passing through a fixed point (2, 1) for
variable p, q.
Illustration:
Find the equation of a line, through the intersection of 2x + 3y 7 = 0 and x + 3y 5 = 0
and having distance from origin as large as possible.

Point of intersection of two lines is A(2, 1)


Now, with OA as radius and O itself as centre draw a circle.
There will be infinitely many lines through A and each except one of them produces a
chord of circle and hence their distance from origin i.e. centre of circle is less than OA i.e.
radius of circle.
But the exceptional one which infact is a tangent to circle at A will be at a distance OA
from O.
Thus, tangent to circle at A will be the line through A and is farthest from origin.
Now, OA tangent at A.
(slope of OA) (slope of tangent at A) = 1
Or, 10/20 (slope of tangent at A) = 2
equation of required line is
(y 1) = 2(x 2)
Or

2x + y 5 = 0

Illustration:
Find the point of concurrency of the altitudes drawn from the vertices (at 1t2, a(t1 + r2)), (at2t3,
a2t2 + t3)) and (at3t1, a(t3 + t1)) respectively of a triangle ABC.
Solution:

Slope of AD = t3.
Equation of AD is y a(t1 + t2) = t3(x + at1t2).

(1)

Equation of CF is y a(t3 + t1) = t2(x at3t1).

(2)

Subtracting (1) from (2), we get


x = a y = a(t1 + t2 + t1t2t3).
Hence the point of concurrency of the altitudes is
(a, a(t1 + t2 + t3 + t1t2t3)).

Position of two points with respect to a


given line
Let the line be ax + by + c = 0 and P(x1, y1), Q(x2, y2) be two points.
Case 1:
If P(x1, y1) and Q(x2, y2) are on the opposite sides of the line
ax + by + c = 0, then the point R on the line ax + by + c = 0 divides the line PQ internally in
the ratio m1 : m2, where m1/m2 must be positive.
Co-ordinates of R
are (m1x2+m2x1/m1+m2, m1y2+m2y1/m1+m2).
Point R lies on the line ax + by + c = 0.
m1/m2 = ax1+by1+c/ax2+by2+c > 0

So that ax1 + by1 + c and ax2 + by2 + c should have opposite signs.
Case 2:
If ax1 + by1 + c and ax2 + by2 + c have the same signs then m1/m2 = ve, so that the point R
on the line ax + by + c = 0 will divide the line PQ externally in the ratio m 1 : m2 and the
points P(x1, y1) and Q(x2, y2) are on the same side of the line ax + by + c = 0.
Illustration:
Find the range of in the interval (0, ) such that the points (3, 5) and (sin, cos) lie on the
same side of the line x + y 1 = 0.
Solution:
3 + 5 1 =7 > 0 sin + cos 1 > 0
sin(/4 + ) > 1/2
/4 < /4 + < 3/4
0 < < /2.
Illustration:
Find a, if (, 2) lies inside the triangle having sides along the lines
2x + 3y = 1, x + 2y 3 = 0, 6y = 5x 1.
Solution:
Let A, B, C be vertices of the triangle.

A (7, 5), B (5/4, 7/8),


C (1/3, 1/9).
Sign of A w.r.t. BC is ve.

If p lies in-side the ABC, then sign of P will be the same as sign of a w.r.t. the line BC

5 62 1 < 0.
Similarly

2 + 32 1 > 0.

And,

+ 22 3 < 0.

(1)
(2)
(3)

Solving, (1), (2) and (3) for and then taking intersection,
We get

? (1/2, 1) (3/2, 1).

Illustration:
The equations of the perpendicular bisectors of the sides AB and AC of a triangle ABC are
respectively x y + 5 = 0 and x + 2y = 0. If the co-ordinates of A are (1, 2), find the
equation of BC.
Solution:
From the figure,
E (x1+1/2, y12/2),
F (x2+1/2, y22/2).

Alt text : equations of the perpendicular bisectors of sides of triangle


Since E and F lie on OE and OF respectively,
x1 y1 + 13 = 0

(1)

and x2 + 2y2 3 = 0

(2)

Also, slope of AB = 1 and slope of AC is 2, so that


x1 + y1 + 1 = 0.

(3)

And 2x2 y2 4 = 0

(4)

Solving these equations, we get the co-ordinates of B and C as


B (7, 6) and C (11/5, 2/5)
Equation of BC is 14x + 23y 40 = 0.
Illustration:
Two fixed points A and B are taken on the co-ordinate axes such that OA = a and OB = b.
Two variable points A and B are taken on the same axes such that OA + OB = OA + OB.
Find the locus of the point of intersection of AB and AB.
Solution:
Let A (a, 0), B (0, b), A (a, 0), B (0, b).
Equation of AB is x/a' + y/b' = 1.

. (1)

and equation of AB is x/a + y/b' = 1.

. (2)

Subtracting (1) from (2), we get, x (1/a 1/a') + y(1/b' 1/b) = 0.


x(a'a)/aa' + y(bb')/bb' = 0.
x/a(bb'+a) + y/bb', 0 b = a(a+b)y/aybx.
From (2) bx + ay = (4) we get x + y = a + b

[Using a a = b b]
.. (3)

which is the required locus.

Angle Bisectors
To find the equations of the bisectors of the angle between the lines
a1x + b1y+ c1 = 0 and a2x + b2y + c2 = 0.
A bisector is the locus of a point, which moves such that the perpendiculars drawn from it to
the two given lines, are equal.
The equations of the bisectors are
a1x+b1y+c1/a12+b12 = + a2x+b2y+c2/a22+b22.

AP is the bisector of an acute angle if,


Tan (PAN) = tan (/2) is such that |tan /2| < 1.
AP is an obtuse angle bisector if,
Tan (PAN) = tan (/2) is such that |tan /2| > 1.
Notes :
When both c1 and c2 are of the same sign, evaluate a1a2 + b1b2. If negative, then acute
angle bisector is a1x+b1y+c1/a12+b12 = + a2x+b2y+c2/a22+b22.
When both c1 and c2 are of the same sign, the equation of the bisector of the angle which
contains the origin is a1x+b1y+c1/a12+b12 = + a2x+b2y+c2/a22+b22.

Bisectors of the angle containing the point (, ) is


a1x+b1y+c1/a12+b12 = +a2x+b2y+c2/a22+b22 if a1 + b1 + c1 and a2 + b2 + c2 have the
same sign.
Bisectors of the angle containing the point (, ) is
a1x+b1y+c1/a12+b12 = +a2x+b2y+c2/a22+b22 if a1 + b1 + c1 and a2 + b2 + c2 have the
opposite sign.
Illustration:
For the straight lines 4x + 3y 6 = 0 and 5x + 12y + 9 = 0 , find the equation of the
(i)

bisector of the obtuse angle between them,

(ii)

bisector of the acute angle between them,

(iii)

bisector of the angle which contains (1, 2)

Solution:
Equations of bisectors of the angles between the given lines are
4x+3y6/42+32 = + 5x+12y+9/52+122

9x 7y 41 = 0 and 7x + 9y 3 = 0.

If is the angle between the line 4x + 3y 6 = 0 and the bisector 9x 7y 41 = 0, then


tan = > 1.
Hence
(i)

The bisector of the obtuse angle is 9x 7y 41 = 0.

(ii)

The bisector of the acute angle is 7x + 9y 3 = 0.

For the point (1, 2)


4x + 3y 6 = 4 1 + 3 2 6 > 0.
5x + 12y + 9 = 12 12 + 9 > 0.
Hence equation of the bisector of the angle containing the point (1, 2) is 4x+3y6/5 =
5x+12y+9/13 9x 7y 41 = 0.

Pair of Straight Lines

The equation ax2 + 2hxy + by2 + 2gx + 2fy + c = 0. Represents a second degree equation
where a, h, b doesnt variables simultaneously.
Let a 0.
Now, the above equation becomes
a2 x2 + 2ax (hy + g) = aby2 2afy ac
on completing the square on the left side, we get,
a2 x2 + 2ax (hy + g) = y2 (h2 ab) + 2y (gh af) + g2 ac.
i.e.

(ax + hy + g) = + y2(h2ab)+2y(ghaf)g2ac

We cannot obtain x in terms of y, involving only terms of the first degree, unless the quantity
under the radical sign be a perfect square. The condition for this is,
(gh af)2 = (h2 ab) (g2 ac)
i.e. g2h2 2afgh + a2f2 = g2h2 abg2 abg2 ach2 + a2bc
cancelling and diving by a, we have the required condition
abc + 2fgh af2 af2 bg2 ch2 = 0
Illustration:
What is the point of intersection of two straight lines given by general equation ax 2+ 2hxy +
by2 + 2gx + 2fy + c = 0?
Solution:
The general solution is
ax2 + 2hxy + by2 + 2gx + 2fy + c = 0

(1)

Let (, ) be the point of intersection we consider line paralleled transformation.


x = x + ,

y = y +

From (1) we have


a(x + )2 + 2h(x + ) (y + ) + b(y + )2 + 2g(x + ) + 2f(y + ) + c = 0

ax2 + 2hxy + by2 + a 2 + 2h + b2 + 2g + 2f + 2x(a + h + g) + 2y + 2y


(h + b + f) = 0

ax2 + 2hxy + by2 + 2x(a + h + g) + 2g + 2y (h + b + f) = 0

Which must be in the form


ax'2 + 2hxy + by = 0
This cannot be possible unless
a + h + g = 0
h + b + f = 0
Solving
/hfbg = /hgaf = 1/abh2
= hfbg/abb2, = hgaf/abh2

Illustration:
Represent lines y = 2x and y = 3x by a homogeneous equation of second degree
Solution:
(y 2x) (y 3x) = 0
Or 6x2 5xy + y2 = 0

Illustration:
Represent lines parallel to y = 2x and y = 3x by a second degree equation
Solution:
(y 2x c1) (y 3x c2) (where c1 and c2 are constants)
= 6x2 5xy + y2 + (3c1 + 2c2) x + ( c1 c2) y + c1 c2 = 0

Note:
1.
Homogeneous part is same as for the equation of above illustration. Therefore, the
homogeneous part of a general second degree equation determines the slope of the lines
i.e. lines parallel to ax2 + 2hxy + by2 + c = 0 and through the origin are represented by the
equation ax2 + 2hxy + by2 = 0
2.
The equation ax2 + 2hxy + by2 + 2fy + c = 0 represents a pair of parallel straight lines
if h/a = b/h = f/g or bg2 = af2
The distance between them is given by 2g2ac/a(a+b) or f2bc/b(a+b)

Illustration:
Does the second degree equation x2 + 3xy + 2y2 x 4y 6 = 0 represents a pair of
lines. If yes, find their point of intersection.
Solution:
We observe that
a = 1, h = 3/2, b = 2, g = 1/2, f = 2, c = 6
\ abc + 2fgh af2 bg2 ch2 = 12 + 3 4 1/2 + 27/2 = 0
Therefore the given second-degree equation represents a pair of lines, x 2 + 3xy + 2y2 x
4y 6 = (x + 2y + 2) (x + y 3).

Consider the equations formed by first two rows of


i.e. ax + hy + g = 0 and hx + by + f = 0
i.e. x + 3/2 y 1/2 and 3/2 x + 2y 2 = 0
Solving these, we get the required point of intersection.
i.e. 2x + 3y 1 = 0
3x 4y 4 = 0
Solving the above equation, we get x = 8, y = 5.
Note:

(2x + 3y 1)(3x + 4y 4) x2 + 3xy + 2y2 x 4y 6.

Angle between pair of lines represented by ax

+ 2hxy

+ by = 0
Comparing the coefficients of x2, y2 and xy, we get
b(y m1x) (y m2x) = ax2 + 2hxy + by2
m1 + m2 = 2h/b and
m1 m2 = a/b
tan acute = |m2m1/1+m1m2|
= |(m1m2)2 4m1m2/1+m1m2|
= |2h2ab/a+b|
Caution:
1.

If two lines through the origin are represented by


y = mx and y = m2x, we cannot write

(y m1x) (y m2x) ax2 + 2hxy + by2

Because coefficient of y2 on left hand side is one on right hand side, it is b.


2.

The given equation represents real lines only when h2 ab > 0

Note:
1.

If two lines are coincident then tan = 0 h2 = ab

2.

If two lines are perpendicular then


m1m2 = 1 a + b = 0

i.e. x2 + 2hxy y2 always represents pair of mutually perpendicular lines through origin.
3.
Two lines are equally inclined to axes but are not parallel. For such a case let us take a
line l1 which is inclined at an angle , then l2 is inclined at ( ).
tan ( ) = tan .
which is the condition for two lines inclined equally to axes.
4.

m1 = h+h2ab/2 and m2 = hh2ab/b

Illustration:
What is the equation of the pair of lines through origin and perpendicular to ax 2 + 2hxy +
by2 = 0
Solution:
Let ax2 + 2hxy + by2 = 0 represents the lines y = m1x (i) and y = m2x (ii)
Lines perpendicular to the lines (i) and (ii) are y = 1/m1 x and y = 1/m2 x respectively and
passing through origin
i.e. m1y + x = 0 and m2y + x = 0
Their combined equation is given by
(m1y + x) (m2y + x) = 0
m1m2 y2 + (m1 + m2) xy + x2 = 0
a/2 y2 2h/b xy + x2 = 0
bx2 2hxy + ay2 = 0 is the equation of the pair of lines perpendicular to pair of lines ax 2 +
2hxy + by2 = 0
Note:

Coefficient of x2 and y2 are interchanged and the sign of xy term is reversed.

Combined equations of the angle


bisectors of the lines represented by ax + 2hxy +by
2

Let ax2 + 2hxy + by2 = 9 represent lines


y m1x = 0 and y m2x = 0
Let P(, ) be any point on one of bisectors.
(m1)/1+m12 = + m2/1+m22
(1 + m22) ( m1)2 (1 + m12) ( m2)2 = 0
Locus of P(, ) is
x2 y2 = 2hxy 1m1m2/m1+m1
x2y2/ab = xy/h; is required equation of angle bisectors (1)
Note:

=0

1.
2.

If a = b, then bisectors are x2 y2 = 0 i.e. x y = 0, x + y = 0

If h = 0, the bisectors are xy = 0 i.e. x = 0, y = 0

3.
If in (i), coefficient of x2 + coefficient of 2 = 0, then the two bisectors are always
perpendicular to each other

Illustration:
Prove that the angle between one of the lines given by
ax2 + 2hxy + by2 = 0 and one of the lines given by ax2 + 2hxy+ by2 + K(x2 + y2) = 0 is equal
to the angle between the other two lines of the system.
Solution:
Let L1 and L2 be one pair and L3 and L4 be the other pair of lines.
If the angle between L1 and L3 is equal to the angle between L2 and L4 then pair of
bisectors of L1 and L2 would be same as that of L3 and L4. Pair of bisectors of L3 and L4 is
x2y2/(a+k)(b+k) = xy/h
x2y2/ab = xy/h
Which is the same as the bisector pair of L1 and L2.
6.
Angle bisectors of ax2 + 2hxy + by2 + 2gx + 2fy + c = 0 (i) are given by (xx0)2(y
2
y0) /ab = (xx0)(yy0)/h where (x0, y0) is the point of intersection of (i)
Enquiry: If we shift the origin of the coordinate system how can the coordinates
of a point be known in the new system? What will happen if we rotate the axis?
(i)

Linear Transformation:
CP = x, DP = y; AP = x, BP = y

If origin is shifted to (h, k) then the coordinates of a point


P(x, y) in the new system are

X=xh
Y=yk
You can check it by putting (h = 0, k = 0) that it gets reduced to the same original
coordinate system.
(ii)

Rotation of Axes:

If axes are rotated anticlockwise by angle q then the coordinates of a point P(x, y) changes
to say P(x, y):
OL = x, PL = y
OM = X, PM = Y
OP = R (say)
cos ( + ) = x/R

(i)

sin ( + ) = Y/R

(ii)

cos = x/R

(ii)

sin = Y/R

(iv)

Eliminating from (i), (ii), (iii) and (iv) we get


X = x cos + y sin
Y = x sin + y cos
It can be checked that if = 0; then coordinates remain unchanged.
Illustration:
Find the equation of the line 2x + y = 7 when co-ordinates system is shifted to the point (3,
1)
Solution:
x = 3 + X and y = 1 + Y

Equation of line becomes


2(3 + X) + (1 + Y) = 7
or 2X + Y = 0
Note:
Slope of the line remains the same.
The general equation of second degree ax2 + 2hxy + by2 + 2gx + 2fy + c=0 represents a

pair of straight lines if

= 0.

abc + 2fgh af2 bg2 ch2 = 0 and h2 > ab.


The homogeneous second degree equation ax2 + 2hxy + by2 = 0 represents a pair of
straight lines through the origin.
If lines through the origin whose joint equation is ax 2 + 2hxy + by2 = 0, are y = m1x and y =
m2x, then y2 (m1 + m2)xy + m1m2x2 = 0 and y2 + 2h/b xy + a/b x2 = 0 are identical. If is
the angle between the two lines, then tan = + (m1+m2)24m1m2/1+m1m2 = + 2h2
ab/a+b.

The lines are perpendicular if a + b = 0 and coincident if h 2 = ab.

Joint Equation of Pair of Lines Joining the


Origin and the Points of Intersection of a
Line and a Curve
If the line lx + my + n = 0, (n 0) i.e. the line not passing through origin) cuts the curve
ax2 + by2 + 2gx + 2fy + c = 0 at two points A and B, then the joint equation of straight lines
passing through A and B and the origin is given by homogenizing the equation of the curve
by the equation of the line. i.e.

ax2 + 2hxy + by2 + (2gx + 2fy) (kx+my/n) + c (lx+my/n)2 = 0.


is the equation of the lines OA and OB.
Illustration:
Prove that the straight lines joining the origin to the points of intersection of the straight line
hx + ky = 2hk and the curve (x k)2 + (y y)2 = c2 are at right angles if h2 + k2 = c2.
Solution:
Making the equation of the curve homogeneous with the help of that of the line, we get
x2 + y2 2(kx + hy) (hx+ky/2hk) + (h2 + k2 c2) (hx+ky/2hk) = 0
or 4h2k2x2 + 4h2k2y2 4h2x(hx+ky) 4h2ky(hx + ky) + (h2 + k2 c)(h2x2 + k2y2 + 2hxy)
= 0.
This is the equation of the pair of lines joining the origin to the points of intersection of the
given line and the curve. They will be at right angles if
Coefficient of x2 + coefficient of y2 = 0 i.e.
(h2 + k2) (h2 + k2 c2) = 0 h2 + k2 = c2

(since h2 + k2 0).

Circle
Circle holds a high pedestal in the entire Syllabus of Co-ordinate
Geometry in Mathematics.
In this chapter, we discuss the algebraic equations representing
a circle and the lines associated with it i.e. a tangent, a pair of

tangents and a chord of contact. The concepts of family of


circles and that of common tangents to two circles under
various configurations are given due importance.
Whole of the analysis is illustrated by appropriate examples, while
giving the student ample scope to test him skills through
assignments of different levels.
Circle is one of the most scoring and hence an important
chapter of Co-ordinate Geometry in the Mathematics syllabus
of IIT JEE, AIEEE and other engineering examinations. The
preliminary knowledge of the concept of Straight Lines is
prerequisite to study Circles.
The chapter is not vast and it fetches 2-3 questions in most of the
engineering examination.
Index

1. Basic Concepts
2. Chord of a Circle
3. Chord of Contact
4. Common Tangents
5. Family of Circle
6. Power of a Point with Respect to a Circle
7. Radical Axis
8. Solved Examples

Circle is most important from the perspective of scoring high in


IIT JEE as there are few fixed pattern on which a number Multiple

Choice Questions are framed on this topic. You are expected to do


all the questions based on this to remain competitive in IIT JEE
examination. It is very important to master these concepts at
early stage as this forms the basis of your preparation for IIT JEE,
AIEEE, DCE, EAMCET and other engineering entrance
examinations.

Basic Concepts
Locus has been defined as the path of a point satisfying some geometrical condition; i.e.
constraint equations. The path represents a curve, which includes all the points satisfying
the given condition.
Similarly a circle can be defined as: The locus of a point which moves in such a way that its
distance from a fixed point is always constant and positive. The fixed point is called the
centre of the circle and the given distance the radius of the circle. In real life, when you
rotate a stone tied with one end of a string then the path followed by stone is exactly a circle
whose centre is your finger an radius is length of the string.
The equation of a circle with its centre at C(xc, yc) and radius r is:
(x xc)2 + (y yc)2 = r2
Proof:
Let P(x, y) be any point on the circle. Then by the definition of the locus the constant
distance is (see figure given below)

|PC| = r ((x-xc) )2+(y-yc) )2 ) = r


(x xc)2 + (y yc)2 = r2
which is the required equation of the circle
Note:
(1) If xc = yc = 0 (i.e. the centre of the circle is at origin) then equation of the circle reduce
to x2 + y2 = r2
(2) If r = 0 then the circle represents a point or a point circle.

Equation of the circle in Various Forms


(i) The simplest equations of the circle is x2 + y2 = r2 whose centre is (0, 0) and radius r.
(ii) The equation (x a)2+ (y b)2 = r2 represents a circle with centre (a, b) and radius r.
(iii) The equation x2 + y2 + 2gx + 2fy + c = 0 is the general equation of a circle with centre
(g, f) and radius (g2+f2-c).
(iv) Equation of the circle with points P(x1, y1) and Q(x2, y2) as extremities of a diameter is (x
x1) (x x2) + (y y1)(y y2) = 0.

Equation of a circle under Different Conditions

Parametric Equation of a circle


Let us consider a circle of radius r and centre at C(xc, yc) we have:
(y-yc)/r = sin (see figure given below)

y = yc + r sin
Similarly x = xc + r cos

This gives the parametric from of the equation of a circle.


General equation of a circle in polar co-ordinate system
Let O be the origin, or pole, OX the initial line, C the centre and a the radius of the
circle.
Let the polar co-ordinates of C be R and , so that OC = R and XOC = .
Let a radius vector through O at an angle with the initial line cut the circle at P and Q.
Let OP be r.

Then we have
CP2 = OC2 + OP2 2OC . OP cos COP
i.e. a2 = R2 + r2 2 Rr cos ( )
i.e. r2 2 Rr cos ( ) + R2 a2 = 0 (1)
This is the required polar equation.
Particular cases of the general equation in polar coordinates.
Note:
1. Let the initial line be taken to go through the centre C. Then = 0, and the equation
becomes
r2 2Rr cos + R2 a2 = 0.
2. Let the pole O be taken on the circle, so that
R = OC =
The general equation the becomes

r2 2ar cos ( ) = 0,
i.e. r = 2a cos ( ).
3. Let the pole be on the circle and also let the initial line pass through the centre of the
circle. In this case
= 0, and R = a
Now, the general equation reduces to the simple form r=2a cos

This is at once evident from the figure given above.


For, if OCA were a diameter, we have
OP = OA cos ,
r = 2a cos .
Let us consider a circle such that points P(x 1, y1) and Q(x2, y2) are on it and PQ is one of
the diameters of the circle.
If R(x, y) is any point on the circle then

Recall:
PRQ = /2 (Angle subtended by diameter at any point on the circle is a right angle).

QR PR
(Slope of QR) x (Slope of PR) = 1
(y-y2)/(x-x2 )(y-y1)/(x-x1 ) = 1
(x x1) (x x2) + (y y1) (y y2) = 0
Which gives the required equation.
Note:
This equation can also be obtained considering
PR2 + QR2 = PQ2
The general from of the equation of a circle is:
x2 + y2 + 2gx + 2fy + c = 0 (1)
(x + g)2 + (y + f)2 = g2 + f2 c
Comparing this equation with the standard equation (x xc)2 + (y yc)2= r2
We have:
Centre of the circle is (g, f), Radius = (g2+f2-c).
Equation (1) is also written as S = 0.
Note:
1. If g2 + f2 c > 0, circle is real
2. If g2 + f2 c = 0, circle is a point circle.
3. If g2 + f2 c < 0, the circle is imaginary.
4. Any second-degree equation ax2 + 2hxy + by2 + 2gx + 2fy+c=0 represents a circle
only when h = 0 and a = b i.e. if there is no term containing xy and co-efficient of x 2 and
y2 are same, provided abc + 2fgh af2 bg2 ch2 0
Illustration:
Find the centre and the radius 3x2 + 3y2 8x 10y + 3 = 0.
Solution:
We write the given equation as x2 + y2 8/3 10/3 y + 1 = 0.
g = -4/3, f = -5/3 , c = 1

Hence the centre is (4/3,5/3) and the radius is


(16/9+25/9-1)=(32/9)=(42)/3.
Illustration:
Find the equation of the circle with centre (1, 2) which passes through the point (4,
6).
Solution:
The radius of the circle is ((4-1)2+(6-2)2 )=25 = 5.
Hence the equation of the circle is (x 1)2 + (y 2)2 = 25
x2 + y2 2x 4y = 20.
Illustration:
A circle has radius 3 units and its centre lies on the line y = x 1. Find theequation of
the circle if it passes through (7, 3).
Solution:
Let the centre of the circle be (, ). It lies on the line y = x 1
= 1. Hence the centre is (, 1).
The equation of the circle is (x )2 + (y + 1)2 = 9. It passes through (7, 3)
(7 )2 + (4 )2 = 9 22 22 + 56 = 0
2 11 + 28 = 0 ( 7) = 0 = 4, 7.
Hence the required equations are
x2 + y2 8x 6y + 6 = 0 and x2 + y2 14x 12y + 76 = 0.

Illustration:
Find the equation of the circle whose diameter is the line joining the points (4, 3) and
(12, 1). Find also the intercept made by it on the y-axis.
Solution:
The equation of the required circle is
(x + 4) (x 12) + (y 3) (y + 1) = 0.

On the y-axis, x = 0 48 + y2 2y 3 = 0.
y2 2y 51 = 0 y = 1 52.
Hence the intercept on the y-axis = 2252 = 413.

Illustration:
Find the equation of the circle passing through (1, 1), (2, 1) and (3, 2).
Solution:
Let the equation be x2 + y2 + 2gx + 2fy + c = 0.
Substituting the coordinates of three points, we get
2g + 2f + c = 2,
4g 2f + c = 5,
6g + 4f + c = 13.
Solving the above three equations, we obtain:
f = 1/2; g = 5/2, c = 4.
Hence the equation of the circle is
x2 + y2 5x y + 4 = 0.
Illustration:
Write general equation of a circle centered at a point on x-axis.
Solution:
Circle is: x2 + y2 + 2gx + c = 0, g2 c 0
Its centre is (g, 0) and radius (g2-c)
Or
(x + g)2 + (y 0)2 = r2

Its centre is (g, 0) and radius r. (figure given above)


Illustration:
Write general equation of a circle passing through the origin.
Solution:
Point (0, 0) must satisfy x2 + y2 + 2gx + 2fy + c = 0
C=0

circle is : x2 + y2 + 2gx + 2fy = 0 (figure given above)


Illustration:
Write the equation of a circle centered at x-axis at (x1, 0) and touching y-axis at the
origin. (figure given below)
Solution:

(x x1)2 + (y 0)2 = (x1)


Illustration:
Write the equation of a circle passing through O (0, 0) A (a, 0) and B (0, b)? Obviously

AB is the diameter of the circle. (Figure given below)

Solution:
(x a) (x 0) + (y 0) (y b) = 0
Illustration:
Find the equation of circle shown in figure given below in polar form.

Solution:
OP = OA cos
r = 2a cos , /2 /2, a is raius of circle
Illustration:
Find the co-ordinates of the centre of the circle represented by
r = A cos + B sin .
Solution:
r = A cos + B sin
= [A/(A2+B2 ) cos +B/(A2+B2 ) sin ] (A2+B2 )
= cos ( ) ((A2+B2 ))
centre is (1/2 (A2+B2 ),tan-1 (B/A) )
Note:
1. The equation of the circle through three non-collinear points

2. The circle x2 + y2 + 2gx + 2fy + c = 0 makes an intercept on x-axis if x2 + 2gx + c = 0


has real roots i.e. if g2 > c. And, the magnitude of the intercept is 2(g2-c).
The Position of a Point with respect to a Circle
The point P(x1, y1) lies outside, on, or inside a circle S x2 + y2 + 2gx + 2fy + c = 0,
according as S1 x12 + y12 + 2gx1 + 2fy1 + c > = or < 0.

Chord of a circle

The equation of the chord of the circle x2 + y2 + 2gx + 2fy +c=0 with M(x1, y1) as the
midpoint of the chord is
xx1 + yy1 + g(x + x1) + f(y + y1) = x12 + y12 + 2gx1 + 2fy1
i.e. T = S1

Illustration:
Find the equation of the circle whose centre is (3, 4) and which touches the line 5x +
12y = 1.
Solution:
Let r be the radius of the circle. Then
r = distance of the centre i.e. point (3, 4) from the line 5x + 12y = 1
= |(15+48-1)/(25-44)|=62/13.
Hence the equation of the required circle is (x 3)2 + (y 4)2 = (62/13)2.

x2 + y2 6x 8y + 381/169 = 0.
Illustration:
Find the co-ordinates of the point from which tangents are drawn to the circle x 2 + y2
6x 4y + 3 = 0 such that the mid-point of its chord of contact is (1, 1).
Solution:
Let the required point be (P(x1, y1). The equation of the chord of contact of P with
respect to the given circle is
xx1 + yy1 3(x + x1) 2(y + y1) + 3 = 0. (1)
The equation of the chord with mid-point (1, 1) is
x + y 3(x + 1) 2(y + 1) + 3 = 1 + 1 6 4 + 3
2x + y = 3.
Equating the ratios of the coefficients of x, y and the constant terms and solving for x 1,
y1 we get x1 = 1, y1 = 0.

Chord of contact
Let AP and AQ be tangents to circle from point P(x 1, y1). Then equation of PQ is known as
equation of chord of contact.

For circle x2 + y2 = a2, it is


x1x + y1y = a2
For general circle, it is
x1x + y1y + g(x1 + x) + f(y1 +y) + c = 0
Note:

1. It is also written as T = 0
2. The equation of chord AB [A (R cos , R sin ); B (R cos , R sin )] of the circle x2 +
y2 = R2 is given by
x cos (( + )/2) + y sin (( - )/2) = a cos (( - )/2)
3. If a line y = mx + c intersects the circle x2 + y2 = a2 in two distinct points A and B then
length of intercept AB = 2((a2 (1+m2 )-c2)/(1+m2 ))
Caution:
The equation of a chord of contact and the equation of the tangent on a point of the circle
and both given by T = 0. The difference is that while in the case of a tangent the point (x 1,
y1) lies on the circle. In the case of a chord of contact (x1, y1) lies outside the circle.

Illustration:
Write the equations of tangents to the circle x2 + y2 = 9 and having slope 2.
Solution:
Tangents with slope m are given by y = mx + a (1+m 2 ) i.e.
3 (1+4).
Note:
These are two parallel tangents to the circle at the end of the diameter.
Illustration:
Write the equation of tangents to the circle x2 + y2 = 25 at the point (3, 4)?
Solution:
Point (3, 4) lies on the circle.
Required equation of the tangent is : 3x + 4y= 25 using
x1x + y1y = a2, where (x1, y1) (3, 4)
Illustration:
Write the equation of normal to x2 + y2 = 25 at (3, 4)?
Solution:
Recall:

y = 2x

A normal to the circle passes through the centre of the circle.


Normal is: y 0 = 4/3 (x 0) (using two point form of straight line) i.e. 3y 4x = 0

Common Tangents
(a) Direct common tangents:
(i) The direct common tangents to two circles meet on the line of centres and divide it
externally in the ratio of the radii.
(ii) The transverse common tangents also meet on the line of centres and divide it
internally in the ratio of the radii.

Notes:
When one circle lies completely inside the other without touching, there is nocommon
tangent.
When two circles touch each other internally 1 common tangent can be drawn to the
circles.
When two circles intersect in two real and distinct points, 2 common tangentscan be
drawn to the circles.
When two circles touch each other externally, 3 common tangents can be drawn to the
circles.
When two circle neither touch nor intersect and one lies outside the other, then 4 common
tangents can be drawn.
P is the point of intersection of two direct common tangents to the circles with centres
C1 and C2 and radii r1, r2 respectively. C1A1, C2A2 are perpendiculars from C1 and C1 to one of
the tangents (figure given below)

PC1A1 and PC2A2 are similar


(C1 P)/(C2 P)=(C1 A1)/(C2 A2 )=r1/r2 i.e. P is a point dividing C1C2 externally in the ratio r1 :
r2 For finding direct common tangents of two circles, find the point P dividing the line
joining the centre externally in the ratio of the radii. Equation ofdirect common
tangents is SS1 = T2 where S is the equation of one circle.
Caution:
Length C1C2 > |r1 r2|
(b) Transverse Common tangents
P is the point of intersection of two transverse tangents to two non-intersecting circles
with centres C1 and C2 and radii r1, and r2 respectively. Then P lies on the line joining the
centres. C1A1 and C2A2 are perpendiculars from C1 and C2 to one of these tangents. (Figure
given below)

Since triangles C1A1P and C2A2P are similar.


So (C1 P)/(C2 P)=(C1 A1)/(C2 A2 )=r1/r2
i.e. P divides the line joining C1 and C2 internally in the ratio r1:r2
Equation of transverse Common tangents is SS1 = T2 where S is the equation of one of
the circle.

Common Tangents

(a) Direct common tangents:


(i) The direct common tangents to two circles meet on the line of centres and divide it
externally in the ratio of the radii.
(ii) The transverse common tangents also meet on the line of centres and divide it
internally in the ratio of the radii.

Notes:
When one circle lies completely inside the other without touching, there is nocommon
tangent.
When two circles touch each other internally 1 common tangent can be drawn to the
circles.
When two circles intersect in two real and distinct points, 2 common tangentscan be
drawn to the circles.
When two circles touch each other externally, 3 common tangents can be drawn to the
circles.
When two circle neither touch nor intersect and one lies outside the other, then 4 common
tangents can be drawn.

P is the point of intersection of two direct common tangents to the circles with centres
C1 and C2 and radii r1, r2 respectively. C1A1, C2A2 are perpendiculars from C1 and C1 to one of
the tangents (figure given below)

PC1A1 and PC2A2 are similar


(C1 P)/(C2 P)=(C1 A1)/(C2 A2 )=r1/r2 i.e. P is a point dividing C1C2 externally in the ratio r1 :
r2 For finding direct common tangents of two circles, find the point P dividing the line
joining the centre externally in the ratio of the radii. Equation ofdirect common
tangents is SS1 = T2 where S is the equation of one circle.

Caution:
Length C1C2 > |r1 r2|
(b) Transverse Common tangents
P is the point of intersection of two transverse tangents to two non-intersecting circles
with centres C1 and C2 and radii r1, and r2 respectively. Then P lies on the line joining the
centres. C1A1 and C2A2 are perpendiculars from C1 and C2 to one of these tangents. (Figure
given below)

Since triangles C1A1P and C2A2P are similar.


So (C1 P)/(C2 P)=(C1 A1)/(C2 A2 )=r1/r2
i.e. P divides the line joining C1 and C2 internally in the ratio r1:r2
Equation of transverse Common tangents is SS1 = T2 where S is the equation of one of
the circle.

Enquiry: What do we understand by co-axial circles and limiting points?


A system of circles every pair of which has the same radical axis is called a coaxial
system.

The centres of circles of a coaxial system, which are of zero radiuses, are called the
limiting points o the coaxial system.
Let the equation of a system of coaxial circles be
x2 + y2 + 2gx + c = 0
Where g is a parameter and c is a constant.
Its radius (g2-c)and centre is (g, 0)
If g2 c = 0 or g = + c, then radius become zero and for these two values of g we
have two circles of zero radius whose centres are ( c, 0).

These circles of zero radius are just points and according to definition given above are
the limiting points of the co-axial system.
1. If the system of circles is intersecting one, then c is negative and these limiting points are
two imaginary points.
2. If the system of circles are non intersecting then c is positive and these limiting points are
both real.
3. If c = 0, points of intersection are coincident to (0, 0) i.e.

Circle touch each other at (0, 0).


Limiting points coincide at (0, 0).

Enquiry: When do two circles intersect orthogonally?


The angle of intersection between two curves intersecting at a point is the angle
between their tangents drawn at that point. The curves are said to be intersecting
orthogonally, if the angle between their tangents are common point is a right angle.
Consider two circles
S1 x2 + y2 + 2g1x + 2f1y + C1 = 0
S2 x2 + y2 + 2g2x + 2f2y + C2 = 0
They intersect at point P such that tangent PT1 and PT2 are at right angle (see figure
given below)

Since radius of a circle is perpendicular to the tangent. So C 1P and C2P are also
perpendicular.
(C1C2)2 = (C1P)2 + (C2P)2 (g1 g2)2 + (f1 f2)2 = r12 + r22
g12 + g22 2g1g2 + f12 + f22 2f1f2=g12 + f12 C1 + g22 + f22 c2
2g1g2 + 2f1f2 = C1 + C2
which is the required condition for the orthogonal intersection of two circles.

At what angle do the circles shown in figure intersect?

From the triangle C1C2P it is clear that angle can be written as:
cos = (r12+r22-d2)/(2r1 r2 )

Enquiry: What do we understand by pole and polar?


Let P(x1, y1) be a fixed point and chords be drawn through this point to a fixed circles S
(see figure given below). The locus of the point of intersection of tangents drawn at the end
points of chords is a line which is called the polar of point P(x 1, y1) (Point P is called the pole)
w.r.t. the circle. So there is a fixed polar for a fixed point and a fixed pole for a fixed line.

The equation of polar of a fixed point P(x1, y1) with respect to the circle x 2 + y2 +
2gx + 2fy + c = 0 is
xx1 + yy1 + g(x + x1) + f(y + y1) + c = 0
i.e. of the form T = 0

Family of Circles
Enquiry: If the numbers of conditions for a circle to be drawn are less than three
then what shall we get?

The general equation of a circle is x2 + y2 + 2gx + 2fy + c = 0. Since this equation


involves three unknowns i.e. g, f and c so we need at least three conditions to get a unique
circle.
For example, if we are given two circles and we want to determine the third circle
touching both of them. We shall need one more condition. Without the condition we get the
equation of family of circles which satisfies the two given conditions. Imposition of a third
condition will result in the equation representing a particular circle.
Let us now see some of the ways of the providing only two condition and equations of
the family of circles resulting under these conditions.

1. Family of circles having a fixed centre


This equation is given by (x h)2 + (y k)2 = r2 (Figure given below)

Since (h, k) is fixed, so only parameter varying is r. This is one parameter family of
circles, and is the equation of the family of concentric circles. Fixation of the radius will give
a particular circle.

2. Equation of Family of circles passing through intersection of two circles S 1= 0


and S2 = 0.
The general equation of the family of circles passing through the intersection of
S1 and S2 in given by S1 + kS2 = 0. Here again we have the one-parameter (k) equation of
family of circles. The particular value of the parameter gives a unique circles.

Caution:
If k = 1, we get equation of common chord i.e. straight line instead of circle.
Let S1 x2 + y2 + 2g1x + 2f1y + c1 = 0
S2 x2 + y2 + 2g2x + 2f2y + c2 = 0
Since, point lies on both the circles,
x2A + y2A + 2g1xA + 2f1yA + c1 = 0
x2A + y2A + 2g2xA + 2f2yA + c2 = 0
x2A + y2A + 2g1xA + 2f1yA + c1 + (x2A + y2A + 2g1xA + 2f1yA + c1)=0
Point A(xA, yA) lies on S1 + S2 = 0 R
Similarly point B(xB, yB) lies on S1 + S2 = 0 R
S1 + S2 = 0 is the family of circles through the intersection of S 1 = 0 and S2 = 0

3. Family of Circles passing through intersection of line L and circle S:


This equation is given by S + L = 0
The particular value of the parameter gives a unique circle.
4. Family of circles touching the circle S = 0 and line L = 0 at their point of
contact
Equation S + L = 0, where is the required family.
5. Family of circle passing through two given points A(x 1, x1) and B(x2, y2)

Required family of circles is

6. Family of circles touching a given line L = 0 at a point (x1, x1) on the line is (x x1)2 + (y
x1)2 + L = 0, the particular value of the parameters gives a unique circle.

Family of Circles
(i) If S x2 + y2 + 2gx + 2fy + c=0 and S x2 + y2 + 2gx + 2fy + c=0 are two intersecting
circles, then S + S = 0, 1, is the equation of a family of circlespassing through the
points of intersection of S = 0 and S = 0.
(ii) If S = x2 + y2 + 2gx + 2fy + c = 0 is a circle which is intersected by the straight line =
ax + by + c = 0 in two real and distinct points, then S + = 0 is the equation of a family
of circles passing through the points of intersection of S = 0 and = 0.
(iii) The equation of a family of circles passing through two given points (x 1, x1) and (x2, y2)
can be written in the form

where is a parameter.
(iv) The equation of the family of circles which touch the line y x1 = m
(x x1) at (x1, x1) for any values of m is (x x1)2 + (y x1)2 +
[(y x1) m(x x1)] = 0.

Notes:

The two circles are said to intersect orthogonally if the angle of intersection of the circles
i.e., the angle between their tangents at the point of intersection is 90o.
The condition for the two circles to cut each other orthogonally is 2gg 1+ 2ff1 = c + c1 where
(g, f) and (g1, f1) are the centres of the respective circles, S = 0 and S1 = 0.

Illustration:
Find the equation of the circle described on the common chord of the circles x 2 + y2
4x 5 = 0 and x2 + y2 + 8y+ 7 = 0 as diameter
Solution:
Equation of the common chord is S1 S2 = 0
x + 2y + 3 = 0
Equation of the circle through the two circles is S1 + S2 = 0
x2 + y2 -4/(1+ ) x+8y/(1+ )+(7 -5)/(1+ ) = 0.
Its centre (2/(1+ ),-4/(1+ )) lies on x + 2y + 3 = 0
2/(1+ )-8/(1+ ) + 3 = 0 2 8 + 3 + 3 = 0 = 1.
Hence the required circle is x2 + y2 2x + 4y + 1 = 0.

Illustration:
The line Ax + By + C = 0 cuts the circle x2 + y2 + ax + by + c = 0 in P and Q. The Line A?
x + B?y + C? = 0 cuts the circle x2 + y2 + a'x + b'y + c' = 0 in R and S. If P, Q, R, S are
concyclic, prove that

Solution:
The equation of the circle through the first line and the first circle, i.e. through P and Q
is
x2 + y2 + ax + by + c + 1 (Ax + By + C) = 0. ........................ (1)

The equation of the circle through R and S is


x2 + y2 + a'x + b'y + c + 2 (A'x + B'y + C') = 0.................... (2)
Since P, Q, R, S are concyclic, (1) and (2) are identical.
1 = (a+1 A)/(a'+2 A' )=(b+1 B)/(b'+2 B' )=(c+1 C)/(c'+2 C' )
1A-2A' + a - a' = 0,
1B -2B' + b - b' = 0,

C- 2C' + c - c' = 0.

Illustration:
Show that the circle passing through the origin and cutting the circles x 2 + y2 - 2a1x 2b1y + c = 0 and x2 + y2 - 2a2x - 2b2y + c = 0 orthogonally is

Solution:
Let the equation of the circle passing through the origin be
x2 + y2 + 2gx + 2fy = 0. ..................... (1)
It cuts the given two circles orthogonally
-2ga1 - 2fb1 = c1 c1 + 2ga1 + 2fb1 = 0 ? (2)
and -2ga2 - 2fb2 = c2 c2 + 2ga2 + 2fb2 = 0 ? (3)

Illustration:
Find that member of the family of circles having centre at (2, 3) which has radius of 5
unit.
Solution:
Family of circles having fixed centre (2, 3) is
(x - 2)2 + (y - 3)2 = r2
here we want that particular circle that has radius 5 units i.e. r = 5, the required circle is,
(x - 2)2 + (y - 3)2 = 25

Illustration:
Find a circle passing through the intersection of x 2 + y2 - 4 = 0 and x2 + y2 - 6x + 5 = 0
which passes through the point (2, 1)?
Solution:
Family of required circles is S1 + S2 = 0
(x2 + y2 - 4) + (x2 + y2 - 6x + 5) = 0
Since the required circle passes through the point (2, 1), the previous equation is satisfied
for the point (2, 1)
(4 + 1 - 4) + (4 + 1 - 12 + 5) = 0
1 - 2 = 0 = 1/2
Equation of the required circle is
(x2 + y2 - 4) + 1/2 (x2 + 2y - 6x + 5) = 0
x2 + y2 - 2x - 1 = 0
External and Internal Contacts of Circles
If two circles with centres C1(x1, y1) and C2(x2, y1) and radii r1 and r2 respectively, touch each
other externally, C1C2 = r1 + r2. Coordinates of the point of contact are A ((r1 r2+r2 r1)/
(r1+r2 ),(r1 y2+r2 y1)/(r1+r2 )).

The circles touch each other internally if


C1C2 = r1 r2.

Coordinates of the point of contact are


T ((r1 x2+r2 x1)/(r1+r2 ),(r1 y2+r2 y1)/(r1+r2 )).

Illustration 11:
Examine whether the two circles x2 + y2 2x 4y = 0 and x2 + y2 8y 4 = 0 touch
each other externally or internally.
Solution:
Let C1 and C2 be the centres of the circles.
C1(1, 2) and C2(0, 4). Let r1 and r2 be the radii of the circles
r1 = 5 and r2 = 25. Also C1C2 = (1+4)=5.
But r1 + r2 = 35 and r2 r1 = 5 = C1C2.
Hence the circles touch each other internally.

Illustration 12:

Find all the common tangents to the circles x2 + y2 2x 6y + 9 = 0 and x2 + y2 6x


2y + 1 = 0.
Solution:
The centres and the radii of the circles are
C1 (1, 3) and r1 = (1+9-9) = 1, C2(3, 1) and r2 = (9+1-1) = 3,
C1C2 = 20, r1 + r2 = 4 = 16 and C1C2 > r1 + r2.
Sense the circles are non-intersecting. Thus there will be four common tangents.
Transverse common tangents are tangents drawn from the point P which divides
C1C2 internally in the ratio 1 : 3.
Direct common tangents are tangents drawn from the point Q which divides
C1C2 externally in the ratio 1 : 3.
Coordinates of P are
((1(-3)+3.1)/(1+3),(1.1+3.3)/(1+3)) i.e. (0,5/2).
and coordinates of Q are (3, 4).
Transverse tangents are tangents through the point (0,5/2).
Any line through (0,5/2) is
y 5/2 = mx (1)
mx y + 5/2 = 0
Applying the usual condition of tangency to any of the circle, we get
(m.1 3 + 5/2 = 0)/((m2+1) ) = 1 (m-1/2)2 = m2 + 1
m 3/4 = 0 or 0.m2 m 3/4 = 0.
m = 3/4 and as coefficient of m2 is zero.
Therefore from (1),
(y-5/2)/x = m = and -3/4 x = 0 is a tangent and
3x + 4y 10 = 0 is another tangent.
Direct tangents are tangents drawn from the point Q(3, 4).

Now proceeding as for transverse tangents their equations are


y = 4, 4x 3y = 0.

Power of a point with respect to a circle


The power of a point P(x1, y1) with respect to a given circle defined as
Power = PA PB
Where A and b are the points on the circle where the line PAB intersects it. (See figure
given below).

Clearly when A and B coincide to T then we have


Power = PT2 = Square of the length of tangent from point P
(x1, y1) to the circle.

Radical Axis
This radical axis of two circles is the locus of a point from which the tangent segments to
the two circles are of equal lengths.
Equation of the Radical Axis
In general S S = 0 represents the equation of the Radical Axis to the two circles
i.e. 2x(g g) + 2y(f f) + c c = 0
where S x2 + y2 + 2gx + 2fy + c = 0
and S x2 + y2 + 2gx + 2fy + c = 0

(i) If S = 0 and s = 0 intersect in real and distinct points then S S = 0 is the equation of
the common chord of the two circles.
(ii) If S = 0 and S = 0 touch each other, then S S = 0 is the equation of the common
tangent to the two circles at the point of contact.

Properties of the Radical Axes


The radial axis of two circles is perpendicular to the line joining the centres.
The radical axis of three circles taken two at a time are concurrent and the point of
concurrency is known as the radical centre.
The radical axis of two circles bisects their direct common tangents.
If two circles cut a third circle orthogonally, then the radical axis of the two circles will
pass through the centre of the third circle.

Solved Examples
Example 1:
Find the equation of the circle circumscribing the triangle formed by the lines x + y =
6, 2x + y = 4, x + 2y = 5.
Solution:
Method 1.
Consider the equation
(x + y 6) (2x + y 4) + 1 (2x + y 4) (x + 2y 5) + 2

(x + 2y 5) (x + y 6) = 0 (1)
This equation is satisfied by the points of intersection of any two of the given three
lines, i.e. it is satisfied by the vertices of the triangle formed by the given lines.

Result:
Now if (i) represents a circle then
(a) coefficient of x2 = coefficient of y2 and
(b) coefficient of xy = 0
And
3 + 51 + 32 = 0 1 = 6/5
Substituting these values in (i) and simplifying
We get
x2+ y2 17x 19y + 50 = 0 (Ans.)
Which is the equation of the required circle.

Method 2.
Solve the lines in pair to find the vertices of the triangle and then obtain theequation
of the circle through these three points.

Example 2:
Find the locus of the point of intersection of perpendicular tangents to the circle x 2 +
y = 4.
2

Solution:
Method 1.
y = mx + 2 (1+m2 ) is tangent to x2 + y2 = 4 for all values of m.
It passes through (h, k) if
k = mh + 2 (1+m2 )

(k mh)2 = 4 (1 + m2)
m2 (h2 4) 2h km + (k2 4) = 0 (i)

The roots m1 and m2 of this quadratic equation are the slopes of PT and PT. If PT and
PT are at right angle then m1m2 = 1.
From (i)
m1m2 = (k2-4)/(h2-4)
(k2-4)/(h2-4) = 1
h2 + k2 = 8
Locus of P(h, k) is x2 + y2 = 8
Method 2:
Equation of tangent at the point T(2 cos , 2 sin )

is 2 cos x + 2 sin y = 4
i.e. x cos + y sin = 2 (i)

Tangent at T would be perpendicular to the tangent at T


If TOT = 90o
i.e. AOT = 90 +
Co-ordinates of T are (2 cos (90 + ), 2 sin (90 + ))
Equation of tangent at the point T is
2 sin x + 2 cos y = 4
or, x sin + y cos = 2 (ii)
Think:
How to get the locus of point P(h, k)?
Caution:
Do not simply square and add (i) and (ii). Though we get the required result, but that it
not the right approach.
Well, lines (i) and (ii) both pass through the point P(h, k)
h cos + k sin = 2 and
h sin + k cos = 2
.Now we want to get a relation between h and k and also eliminate
The best way to do this is to square and ad above two
Equations and we get h2 + k2 = 8
Locus of P(h, k) is x2 + y2 = 8

Method 3:
OP2 = OT2 + TP2 ( OTP is a right angled triangle)
OP2 = OT2 + OT2
OP2 = 2R2
h2 + k2 = 2(4) = 8

Locus of P(h, k) is x2 + y2 = 8.
Note:
x2 + y2 = 8 is director circle of the circle x2 + y2 = 4

Example 3:
Find the condition that the line 3x + 44y p = 0 is tangent to the circle x 2 + y2 4x 6y
+9=0
Solution:
Radius of the given circle is 2 and centre is (2, 3). So for line 3x + 4x p = 0 to be
tangent to the circle we have,
|(3.2+4.3-p)/(32+42 )| = 2.
|18 p| = 10
18 p = 10 p = 8
18 p = 10 p = 28
Hence the required condition is p = 8 or p = 28. (Ans.)
Example 4
A circular plot of land in the form of a unit circle is to be divided into two equal parts by
the arc of a circle whose centre is on the circumference of the circular plot. Show that the
radius of the circular arc is 2 cos , where is given by sin 2 2 cos 2 = /2.
Solution:
Let O be the centre of the given circular plot of radius 1 i.e. OA = OB = OC = 1 and A
be any point on its circumference. Again BDC be the arc of the circle with centre A and
dividing the given circle into two equal parts. Let r the radius of the new circle, then AB = AC
= AD = r.

Let AOB =
Then OBA = OCA = , and AOB = ( 2).
Now area ABDCA must be = 1/2 area of unit circle = ( (1) 2)/2 = /2 (i)
r/sin( -2 ) =1/(sin ) r = 2 cos .
Required Area APBDCQA = Area of sector ABDCA + area of sector OCQAPB 2 area of
OAB
/2 = 1/2 r2 (2) + 1/2 (1)2 (2 4) 2 1/2 (1)2 sin ( 2)
sin 2 2 cos 2 = /2 (Ans.)

Example 5:
Find the equation of the circle which passes through the point (2, 0) and whose
centre is the limit of the point of intersection of lines 3x + 5y = 1, (2 + c)x + 5c 2 y = 1.

Solution:
Solving, 3x + 5y = 1, (2 + c)x + 5c2y = 1
We get,

when c 1
Pause:
We will study limits in detail in module 5.
Now, we want to find out the equation of the circle which passes through (2, 0) and
has its centre at (2/5), 1/25).
Equation of the circle is
(x-2/5)2+(y+1/25)2=(2/5-2)2+(1/25)2
This is the required equation of the circle.

Example 6:
Find the equation of circle having the lines x2 + 2xy + 3x + 6y = 0 as its normals
and having size just sufficient to contain the circle, x(x 4) + y(y 3) = 0
Solution:
The combined equation of two normal of the circle is given by
x2 + 2xy + 3x + 6y = 0
(x + 3)(x + 2y) = 0
x = 3, x = 2y
Recall:
A normal to a circle always passes through the centre of the circle. Now solving these,
we get the co-ordinates of the centre of the circle as (3, 3/2); because the two normal
intersect at the centre of the circle
The required circle just contains the circle
x(x 4) + y(y 3) = 0
i.e. x2 + y2 4x 3y = 0 (i)
Hence the required circle will touch the circle given by (1) internally.
Let r be the radius of the required circle. Now the two circles given by (1)
= ([22+(3/2)2 ] )=5/2 and centre = (2,3/2)

Now the required circle will touch the circle (i) internally, if
We have distance between the centre of the two circles = difference between their radii.
((-3-2)2+(3/2-3/2)2 )=(r-5/2)
r = 15/2
Hence the equation of the required circle is given by
(x + 3)2 + (y-3/2)2=(15/2)2
x2 + y2 + 6x 3y 45 = 0 (Ans.)

Example 7:
A tangent is drawn to each of the circles x2 + y2 = a2, x2 + y2 = b2. Show that if these
two tangents are perpendicular to each other, the locus of their point of intersection is a
circle concentric with the given circles.
Solution:
Method 1:
Let P (x1, y1) be the point of intersection of the tangents PA and PB where A, B are
points of contact with the circles respectively.
As PA is perpendicular to PB, the corresponding radii OA and OB are also perpendicular.
Let AOX =

BOX = 90 +
Using the parametric from of the circle we can take

A (a cos , a sin )
B (b sin , b cos )
The equation of PA is
x (a cos) + y (a sin ) = a2
y cos x sin = b
Since P(x1, y1) lies on these tangents
x1 cos + y1 sin = a and y1 cos x1 sin = b
. Squaring and adding above equation (we get).as is a variable quantity; we eliminate
x12 + y12 = a2 + b2
locus of p is x2 + y2 = a2 + b2, which is concentric with given circles.

Method 2:
OAPB is a rectangle
OP2 = OA2+ AP2
x1 2 + y 1 2 = a 2 + b 2
Locus of P(x1, y1) is x2 + y2 = a2 + b2

Example 9:
The circle x2 + y2 = 1 cuts x-axis at P and Q. Another circle with centre at Q and
variable radius intercepts the first circle at R above x-axis and the line segment PQ at S. Find
the maximum area of the triangle QSR.
Solution:
Method 1.
Equation of circle centred at Q is (x + 1)2 + y2 = 2

Since point R (cos , sin ) lies on this circle


2 = (cos + 1)2 + (sin )2 = 2 + 2 cos
= 2 (2cos2 /2) = 2 cos /2
A = Area of QSR = 1/2 base altitude
= 1/2 sin
= cos /2 sin (i)
dA/d = cos cos /2-1/2 sin /2 sin

Pause:
We will study maxima and minima in module 5.
For max./min. dA/d = 0

tan = 2 cot /2
2t/(1-t2 )=2/t (where t=tan /2)
t = tan /2=1/2
(d2 A)/(d2 ) = sin cos /2-1/2 cos sin /2-1/2 sin /2 cos 1/4 cos /2 sin

(A"( )(at tan /2=1/2) ) = ve


From (i)
Maximum area = cos /2 sin
= 2 sin /2 (cos /2)2
4/(33) sq. units. (Ans.)

Method 2.
Equation of circle I is
x2 + y2 = 1 It cuts x-axis
where P (1, 0) and
Q (1, 0)
Let QR = then equation of the circle II.
Centred at Q(1, 0) and radius = is given by
(x + 1)2 + y2 = 2 (1)
Solving it with x-axis; we get S ( 1, 0).
Also solving the two circles, we get the co-ordinates of R
as [(2/2)-1,/2 ((4-2 ) )]
The area of QRS = 1/2 QS RL
= 1/2 /2 ((4-2 ) )
= A (say)
Now A is max./min. means A2 is max./min.
Let A2 = Z.
Then Z = 4/16 (4 )2
dz/d=1/4. 43 (65)/16

(d2 z)/d2 = 33 30 (4/16)


For max./min. of A i.e. max./min. of A2 or Z. we get
dZ/d = 0 = (8/3) and then
(d2 z)/d2 = 3.8/3-30/10.64/9 = ve
For = ((8/3))
Thus area is max when = ((8/3))
Also max. area of QRS
1/4.8/3 ((4/3))=(4/33) sq. units.
Example 10:
Distances from the origin to the centres of three circles x2 + y2 2x = c2(where c is
constant and is variable) are in G.P. Prove that the lengths of tangents drawn from any
point on the circle x2 + y2 = c2 to the three circles are in G.P.
Solution:
The equation of the three circles is
x2 + y2 2x = c2 (1)
where = 1, 2, 3. Their centres are:
(

, 0), (2, 0) and (3, 0)

Now distances of these points from the origin are 1, 2 and 3


13 = 22 (2)
Now, let P(h, k) be any point on the circle
x2 + y2 = c2, then h2 + k2 = c2 (3)
If r1, r2, r3 are the lengths of the tangents from P(h, k) on
The three circles, we then obtain
r12 = h2 + k2 21h c2 = c2 h c 22, using (3)

r12 = 2h
Similarly r22 = 22h
r32 = 23h
= r12 r32 = 41 2 h2 = 422 h2 = (r22)2
= r2r3 r22 r1, r2, r3 are in G.P.

(Proved).

Example 11:
Show that the equation x2 + y2 4x ky 5 = 0 represents (for variable k) a family of
circles passing through two fixed points A and B. Find the equation of the circle belonging to
this family and cutting circle x2 + y2 6x 5y = 0 at right angles.
Solution:
x2 + y2 4x ky 5 = 0
(x2 + y2 4x 5) + k ( y) = 0
This is the equation of family of circles passing though the intersection point of x 2+ y2 4x
5 = 0 (a circle) and a straight line putting y = 0, in x 2 + y2 4x 5 = 0 gives
x2 4x 5 = 0
x = 1, 5
Hence the given circle passes through two fixed points (1, 0) and (5, 0)
For given family of circles
x2 + y2 4x ky 5 = 0 (1)
g = 2, f = k/2, c = 5
One member of family (1) and circle x2 + y2 6x 5y 0 (2)
Intersect orthogonally
For circle (2)
g = 3, f = 5/2, c = 0
For two circle to be orthogonal

2(gg + ff) = (c + c)
2[(2) (3) + (k/2)(5/2)] = 5 + 0
2 [6+5K/4] = 5
12 + 5K/2 = 5
5K/2 = 17
K = 34/5
Required equation of circle is
x2 + y2 4x + 34/5 y 5 = 0 (Ans.)

Example 12:
Lines 5x + 12y 10 = 0 and 5x 12y 40 = 0 touch a circle C 1 of diameter 6. If the
centre of C1 lies in first quadrant, find the equation of circle C2 which is concentric with
C1 and cuts intercept of length 8 on these lines.
Solution:
Recall:
If a circle touches two lines L1 and L2 then the centre of the circle lies on the angle
bisectors of the lines.
Angle bisector of given lines are
(5x+12y-10)/13=(5x-12y-40)/13
Taking +ve sign: y = (-5)/4
Taking ve sign: x = 5
Given lines L1 and L2 intersect at (5,-5/4)
Since the centre of C1 lies in the first quadrant, it can lie on x = 5 only.
Let the centre of C1 be (5, y1)
|(5(5)+12y1-10)/(52+122 )| = 3
15 + 12y1 = 39

y1 = 2 or y1 = -54/12 (Neglect)
Centre of C1 is (5, 2)
Since C2 is concentric with C1, its centre is also (5, 2)

C2 cuts intercept of length 8 on lies


5x + 12y 10 = 0 and 5x 12y 40 = 0
AB = CD = 8
Recall:
Perpendicular from centre bisects the chord.
AN = 4
C2 N = 3 (given)
Radius of C2 = r (say)
r2 = (AN)2 + (C2N)2 = 16 + 9
r=5
Equation of C2 is (x 5)2 + (y 2)2 = 25

Example 13:
Find the equation of the circle which passes through the point (2a, 0) and whose radical
axis with respect to the circle x2 + y2 = a2 is the lines x = a/2.
Solution:
Recall:

Radical axis of two circles is S1 S2= 0.


Let equation of circle is
x2 + y2 + 2gx + 2fy + c = 0 (i)
radical axis of circle (i) and circle x2 + y2 a2 = 0 is given by
x2 + y2 + 2gx + 2fy + c = x2 + y2 a2
2gx + 2fy + c + a2 = 0 (ii)
It is given that radical axis is x a/2 = 0 (iii)
Comparing (ii) and (iii)
We get f = 0, (c+a2)/2g=-a/2
ag + a2 + c = 0 (iv)
Circle (i) passes through (2a, 0)
4ag + 4a2 + c = 0 (v)
From (iv) and (v)
3ag + 3a2 = 0
g = a
c = (ag + a2) from (iv)
= (a2 + a2) = 0
Equation of circle is
x2 + y2 2ax = 0 (Ans.)
Example 14:
Show that x2 + y2 + 4y 1 = 0, x2 + y2 + 6x + y + 8 = 0 and x2 + y2 4x 4y 37 = 0
touch each other.
Solution:
S1 x2 + y2 + 4y 1 = 0

S2 x2 + y2 + 6x + y + 8 = 0
S3 x2 + y2 4x 4y 37 = 0
C1 (0, 2), C2 (3, 1/2), C3 (2, 2)
r1 = (4+1) = 5
r2 = (9+1/4-8)=5/2
r3 = (4+4+37)=45=35
C2C3 = ((2+3)2+(2+1/2)2 )=(25+25/4)=5/2 5
C1C3 = ((2-0)2+(2+2)2 )=25

r1 + r2 = 3/2 5 = C1C2
r3 r2 = 5/4 5 = C2C3
r3 r1 = 25 = C1C3
Recall:
Two circles touch each other.
(i) externally if C1C2 = r1 + r2
(ii) internally if C1C2 = r1 r2

Example 15:

Find the four common tangents to the circles x2 + y2 22x + 4y + 110 = 0 and x2 + y2
22x 4y 100 = 0
Solution:
S1 x2 + y2 22x + 4y + 100 = 0
S2 x2 + y2 + 22x 4y 100 = 0
C1 (11, 2), C2 (11, 2)

r1 = ((11)2+(-2)2-100) = 5
r2 = ((-11)2+(2)2+100) = 15
Out of four common tangents are transverse tangents and other two direct tangents. (1)
and (2) are direct common tangents while (3) and (4) are transverse common tangents.
Recall:
Transverse common tangents divide line joining centres in ratio of radii internally while
direct tangents divides line joining centres in ratio of radii externally.
Let T1, T2 divide C1C2 in ratio of r1 : r2 internally and externally respectively.
Co-ordinates of T1 are (1511+5(-11))/(15+5) and (15(-2)+52)/(15+5)
That is T1 is the point (11/2,-1)
Co-ordinates of T2 are (1511-5(-11))/(15-5) and (15(-2)-52)/(15-5)
that is T2 is the point (22, 4)
Let the equation to either of the tangents, passing through T1 be
y + 1 = m (x 11/2) (A)

Then the perpendicular from the point (11, 2) on it is equal to + 5 and hence
(m(11-11/2)-(-2+1))/(1+m2 ) = 5
On solving, we have m = -24/7 or 4/3
The required tangents through T1 are therefore
24x + 7y = 125, and 4x 3y = 25
Similarly the equation to the tangents through T2 is
y + 4 = m (x 22) (B)
where (m(11-22)-(-2+4))/(1+m2 ) = 5
On solving, we have m = 7/24 or 3/4
On substitution in (B) the required equations are therefore
x 24y = 250 and 3x + 4y = 50
The four common tangents are therefore found. (Ans.)
Example 16:
The circle x2 + y2 4x 4y + 4 = 0 is inscribed in a triangle which has two of its sides
along the co-ordinate axes. The locus of the circumcentre of the triangle is x + y xy + k
(x2 + y2)1/2 = 0 Find k?

Solution:
Method 1.
The equation of the incircle can be put in the from (x 2)2 + (y 2)2 = 4
This implies that the inradius r = 2 (i)
Let the hypotenuse of the triangle meet OX and OY at A (a, 0) and B (0, b) respectively.
r = (Area of AOB) / ((1/2)(Sum of sides of AOB)) = ((1/2)ab) / ((1/2)
(a+b+(a2+b2 )))
2 = ab/(a+b+(a2+b2 ))
Let M (x1, y1) be the circumcentre of OAB. Since OAB is right angled, its circumcentre is
the mid point of hypotenuse.

2 = (4x1 y1) / (2x1+2y1+2(x12+y12 ))


x1 + y1 + (x12+y12 ) = x1y1
Hence the equation of the locus of M(x1, y1) is
x + y xy + (x2+y2 ) = x1y1
Comparing it with the given equation of the locus, we find that
k = 1. (Ans.)

Method 2.
Equation of AB is x/a + y/b = 1 (1)
(x1, y1) (a/2,b/2), where M(x1, y1) is the circumcentre of OAB i.e. midpoint of the
hypotenuse.
(1) becomes: x / (2x1 )+y / (2y1 ) = 1 (2)
In circle (x 2)2 + (y 2)2 = 4 touches line (2)
(2/(2x1 )+2 / (2y1 )-1) / ((1/2x1 )2+ (1/2y1 )2 ) = 2
x1 + y1 x1 y1 + (x12+y12 ) = 0
Locus of M(x1, y1) is
x + y xy + (x2+y2 ) = 0
Comparing it with the given equation of the locus, we find that k = 1.

Note:
Distance from (2, 2) to the line x/2x1 +y/2y1 -1 = 0 has been taken 2, because origin and
this point lies on the same side of the origin.

Parabola
Parabola Definition

Parabola is the chief and easiest chapter in the Conic Sections of Co-ordinate Geometry in
Mathematics. Let us start analyzing the thought A point always moves such that the ratio of
its distances from a fixed point and a fixed line is constant. Can we get some meaningful
result? Yes we get a parabola, which follow the above rule/thought. We can represent the
curve mathematically using the co-ordinates and this curve is useful in finding out the many
physical/practical phenomenons.
If we take a Right Circular Cone and cut it by a horizontal plane, we get a cross-section
which is circular. If we cut this cone by planes of different orientations, different planes
produce different type of curves. As all these curves are sections of a right circular cone, we
call them conic sections. When we analyze these curves in detail we find that when the
ratio of the distances of a point on the curve from a fixed point to its distance from a fixed
line, is equal to 1 we have one type of curve, for ratio less than 1, we have second type of
curve and for ratio more than 1, we have third type of curve. With this basis we can analyze
these three curves.
In this sections well discuss the curve of 1st type for which the ratio is equal to 1 i.e. the
distance from the fixed point is always equal to the distance from the fixed line. We call this
a Parabola.
Parabola is one of the easiest and important chapters of Conic Sections of Co-ordinate
Geometry in the Mathematics syllabus of IIT JEE, AIEEE and other engineering examinations.
The chapter is important because it fetches 1-2 questions in most of the engineering
examination
Index

1. Conic Sections- Parabola Definition


2. Focal Chord
3. Chord
4. Tangent to a Parabola
5. Normal to a Parabola
6. Propositions on the Parabola
7. Solved Examples

Parabola is important from the perspective of scoring high in IIT JEE as there are few fixed
pattern on which a number Multiple Choice Questions are framed on this topic. You are
expected to do all the questions based on this to remain competitive in IIT JEE examination.

It is very important to master these concepts at early stage as this forms the basis of your
preparation for IIT JEE, AIEEE, DCE, EAMCET and other engineering entrance examinations.

Conic Sections
Conics or conic sections are the curves corresponding to various plane sections of a
right circular cone by cutting that cone in different ways.
Each point lying on these curves satisfies a special condition, which actually leads us
towards the mathematical definition of conic sections.
If a point moves in plane in such a way that the ratio of its distance from a fixed point to
its perpendicular distance from a fixed straight line, always remains constant, then the locus
of that point I called a Conic Section.
The fixed point is called the focus and the fixed line is called directrix of the conic. The
constant ratio is called the eccentricity and is denoted by e.
According to the value of there are three types o conic i.e. for e = 1, e < 1 and e > 1 the
corresponding conic is called parabola, ellipse and hyperbola respectively.
A conic section or conic is the locus of a point, which moves so that its distance from a
fixed point is in a constant ratio to its distance from a fixed straight line, not passing through
the fixed point.
The fixed point is called the focus.
The fixed straight line is called the directrix.
The constant ratio is called the eccentricity and is denoted by e.
When the eccentricity is unity i.e. e = 1, the conic is called a parabola; when e < 1, the
conic is called an ellipse; and when e > 1, the conic is called a hyperbola.
The straight line passing through the focus and perpendicular to the directrix is called the
axis of the parabola.
The point of intersection of a conic with its axis is called vertex.
The chord passing through focus and perpendicular to axis is called latus rectum.
Any chord of the parabola which is perpendicular to the axis is called double ordinate.
The straight line perpendicular to axis of the parabola passing through vertex is called
tangent at the vertex.

Axis of the conic:


The line through focus and perpendicular to the directrix is called the axis of the conic. The
intersection point o conic with axis is known as the vertex of the conic.
Enquiry: How do we mathematically define a parabola and what are its various
features?
The locus of the point, which moves such that its distance from a fixed point (i.e. focus)
is always equal to its distance from a fixed straight line (i.e. directrix), is called parabola.
Equation of Parabola:
Let S be the focus, V be the vertex, ZM be the directrix and x-axis be the axis
of parabola. We require therefore the locus of a point P, which moves so that its distance
from S, is always equal to PM i.e. its perpendicular distance from ZM. After appropriate
configuration let S = (a, 0)

We have ten SP2 = PM2


(x a)2 + y2 = (a + x)2
y2 = 4ax This is the standard equation of a parabola.

There are four common forms of parabola according to their axis, with their vertex at origin
(0, 0).

Let us understand some other features of a parabola.


(a) Focal Distance:
The distance of a point on the parabola from its focus is called the focal distance of the
point Focal distance of P = SP = x + a.
(b) Focal Chord:
A chord of the parabola, which passes through its focus, is called Focal chord.
(c) Latus Rectum:
The chord through focus and perpendicular to the axis of the parabola is called the latus
rectum.

The co-ordinates of the end point of the latus rectum are (a, 2a) and (a, 2a) and length
of latus rectum = |4a|.

For horizontal parabola


Let us consider origin (0, 0) as the vertex A of the parabola and two equidistant points
S(a, 0) as focus and Z(a, 0) a point on the directrix now let P(x, y) be the moving point.
Draw SZ perpendicular from S on the directrix. Then SZ is the axis of the parabola. Now the
middle point of SZ, that is A, will lie on the locus of P.
i.e. AS = AZ.

The x-axis along AS, and the y-axis along the perpendicular to AS, as A, as in the figure. Now
by definition PM = PS MP2 = PS2
So, that, (a + x)2 = (x a)2 + y2.
Hence, the equation of horizontal parabola is y2 = 4ax.

Similarly for the vertical parabola


Let us consider origin (0, 0) as the vertex A of the parabola and two equidistant points
S(0, b) as focus and Z(0, b) a point on the directrix now let P(x, y) be the moving point.
Draw SZ perpendicular from S on the directrix. Then SZ is the axis of the parabola. Now the
middle point of SZ, that is A, will lie on the locus of P i.e. AS = AZ.
The y-axis along AS, and the x-axis along the perpendicular to AS at A, as in the figure.

Now by definition PM = PS
MP2 = PS2
So that, (b + y)2 = (y b)2 + x2.
Hence, the equation of vertical parabola is x2 = 4by.

Finding the end points of latus Rectum


For finding the end points of latus rectum LL of the parabola y2 = 4ax, we put x = a as latus
rectum passes through focus (a, 0) therefore we have

y2 = 4a2
y = + 2a
Hence the end points are (a, 2a) and (a, 2a).
Also LSL = |2a (2a)| = 4a = length of double ordinate through the focus S.

Note:
Two parabolas are said to be equal when their latus recta are equal.
The important points & lines related to standard Parabola

Note:
1. The points and lines of two parabolas can be interchanged by transformations.
2. If a > 0 & a < 0 the parabola will be forward opening and backward opening
respectively.
3. If b > 0 & b < 0 the parabola will be forward opening and downward opening
respectively.

The important points & lines related to shifted Parabola


The forms of the horizontal and vertical parabola having vertex at (h, k) can be obtained by
shifting the origin at (h, k) as below

Illustration:
Find the vertex, axis, directrix, tangent at the vertex and the length of thelatus
rectum of the parabola 2y2 + 3y 4x 3 = 0.
Solution:
The given equation can be re-written as (y-3/4)2= 2 (x+33/32)
Which is of the form Y2 = 4aX.
Hence the vertex is (-33/32,-3/4).
The axis is y + 3/4 = 0 y = 3/4.
The directrix is X + a = 0.
x + 33/32+1/2 = 0 x = -49/32.
The tangent at the vertex is x + 33/32 = 0 x = 33/32.
Length of the latus rectum = 4a = 2.

Illustration:
The extreme points of the latus rectum of a parabola are (7, 5) and (7, 3). Find the

equation of the parabola and the points where it meets the coordinate axes.
Solution:
Focus of the parabola is the mid-point of the latus rectum.
S is (7, 4). Also axis of the parabola is perpendicular to the latus rectum and passes
through the focus. Its equation is
y 4 = 0/(5-3) (x 7) y = 4.
Length of the latus rectum = (5 3) = 2.
Hence the vertex of the parabola is at a distance 2/4 = .5 from the focus. We have two
parabolas, one concave rightwards and the other concave leftwards. The vertex of the first
parabola is (6.5, 4) and its equation is
(y 4)2 = 2(x 6.5) and it meets the x-axis at (14.5, 0).
The equation of the second parabola is (y 4)2 = 2 (x 7.5).
15).

It meets the x-axis at (0.5, 0) and the y-axis at (0, 4 +

Parametric Form of a Parabola


Suppose that the equation of a tangent to the parabola y2 = 4ax. (i)
is y = mx + c. (ii)
The abscissa of the points of intersection of (i) and (ii) are given by the equation (mx +
c) = 4ax. But the condition that the straight line (ii) should touch the parabola is that it
should meet the parabola in coincident points hence discriminant should be zero
2

(mx 2a)2 = m2c2 (iii)


c = a/m.
Hence, y = mx + a/m is a tangent to the parabola y2 = 4ax, whatever be the value of m.
Equation (mx + c)2 = 4ax now becomes (mx a/m)2 = 0.
x = a/m2 and y2 = 4ax
y = 2a/m.
Thus the point of contact of the tangent y = mx + a/m is (a/m2 ,2a/m).

Taking 1/m = t where t is a parameter, i.e., it varies from point to point. Theparabola y2 =
4ax as a parametric form is given by the co-ordinate (at 2, 2at) and we refer to it as point t.

Illustration:
Prove that the area of the triangle inscribed in the parabola y2 = 4ax is a2 |(t1 t2) (t2
t3) (t3 t1)| where t1, t2 and t3 are the vertices.
Solution:
The three points on the parabola are (at12, 2at1), (at22, 2at2) and (at32, 2at3).

THE GENERAL EQUATION OF A PARABOLA


We shall now obtain the equation of a parabola when the focus is any point and the
dircectrix is any line.
Let (h, k) be the focus S and lx + my + n = 0 the equation of the directrix ZM of a parabola.
Let (x, y) be the coordinates of any point P on the parabola. Then the relation, PS = distance
of P from ZM, gives
(x h+) + (y k)2 = (lx + my + n)2/(l2 + m2)
(mx ly)2 + 2gx + 2fy + d = 0.

This is the general equation of a parabola. It is clear that second-degree terms in the
equation of a parabola form a perfect square.
The converse is also true, i.e. if in an equation of the second degree, the second-degree
terms from a perfect square then the equation represents a parabola, unless it represents
two parallel straight lines.

Note:
The general equation of second degree i.e. ax2 + 2hxy + by2 + 2gx + 2fy + c = 0
represents a parabola if 0 and h2 = ab. ( = abc + 2fgh af2 bg2 ch2).
Special case:
Let the vertex be (, ) and the axis to be parallel to the x-axis. Then the equation of
parabola is given by (y )2 = 4a (x ) which is equivalent to x = Ay 2 + By + C.
If three points are given we can find A, B and C.
Similarly, when the axis is parallel to the y-axis, the equation of the parabola is y = Ax2 +
Bx + C.

Illustration:
Find the equation of the parabola whose focus is (3, 4) and directrix
x t + 5 = 0.
Solution:
Let P(x, y) be any point on the parabola. Then
((x-3)2 (y+4) )=|x-y+5|/(1+1)
(x 3)2 + (y + 4)2 = (x-y+5)2/2
x2 + y2 + 2xy 22x + 26y + 25 = 0.
(x + y)2 = 22x 26y 25.

Illustration:
Find the equation of the parabola having focus (6, 6) and verte (2, 2).
Solution:

Let S(6, 6) be the focus and A(2, 2) the vertex of the parabola. On SA take a point K
(x1, y1) such that SA = AK. Draw KM perpendicular on SK. Then KM is the directrix of
the parabola.
Since A bisects SK, ((-6+x1)/2,(-6+y1)/2) = (2, 2)
6 + x1 = 4, and 6 + y1 = 4
Or (x1, y1) = (2, 10).

Hence the equation of the directrix KM is y 10 = m (x + 2). (1)


Also gradient of
SK = (10-(-6))/(2-(-6) )=16/8 = 2; m = (-1)/2
So that equation (1) becomes y 10 = 1/2 (x 2)
or x + 2y 22 = 0 is the directrix.
Next, let PM be a perpendicular on the directrix KM from any point
P(x, y) on the parabola.
Form SP = PM, the equation of the parabola is
({(x+6)+(y+6)2 } )=(x+2y-22)/((12+22 ) )
or 5 = (x2 + y2 + 12x + 12y + 72) = (x + 2y 22)2
or 4x2 + y2 4xy + 104x + 148 y 124 = 0.
or (2x y)2 + 104x + 148y 124 = 0.

Illustration:

If the point (2, 3) is the locus and x = 2y + 6 is the directrix of a parabola, find
(i) the equation of the axis,
(ii) the co-ordinates of the vertex,
(iii) length of the latus rectum,
(iv) equation of the latus rectum.

Solution:
(i) We know that the axis of a parabola is the line through the focus
And perpendicular to the directrix.
The equation of any line passing through the focus (2, 3) is
y 3 = m (x 2) mx y = 3 2m
If the line be perpendicular to the directrix x 2y = 6, we have,
m (1/2) = 1 m = 2.
Hence the equation of the axis is y 3 = 2 (x 2) 2x + y = 7.
(ii) The co-ordinates of the point of intersection (say) A of the directrix x 2y = 6 and the
axis 2x + y = 7 are obtained by solving the two equations; thus they are (4, 1). Since the
vertex is the middle point of A (4, 1) and the focus S(2, 3); the co-ordinates of the vertex
are ((4+2)/2,(3-1)/2), i.e. (3, 1).
(iii) Since OS = ((3-2)2+(1-3)2 )=5,
The length of the latus rectum = 4OS = 45.
(iv) Since the latus rectum is the line through the focus parallel to the directrix, its
equation is x 2y + c = 0, where c is given by 2 2.3 + c = 0, i.e. c = 4.

Focal Chord
Any chord to y2 = 4ax which passes through the focus is called a focal chord of
the parabola y2 = 4ax.
Let y2 = 4ax be the equation of a parabola and (at2, 2at) a point P on it. Suppose the
coordinates of the other extremity Q of the focal chord through P are (at12, 2at1).

Then, PS and SQ, where S is the focus (a, 0), have the same slopes
(2at-0)/(at2-a)=(2at1-0)/(at12-a)
tt12 t = t1 t2 (tt t1

+ 1) (t1 t) = 0.

Hence t1 = 1/t, i.e. the point Q is (a/t2, 2a/t).


The extremities of a focal chord of the parabola y2 = 4ax may be taken as the points t and
1/t.
Illustration:
Prove that the circle with any focal chord of the parabola y2 = 4ax as its diameter
touches its directrix.
Solution:
Let AB be a focal chord. If A is (at2, 2at), then B is (a/t2 ,-2a/t).
Equation of the circle with AB as diameter is
(x at2) (x-a/t2 ) + (y 2at) (y+2a/t) = 0.
For x = a, this gives (a2 (1+t2 )2)/t2 + y2 2ay (t-1/t) 4a2 = 0.
a2 (t-1/t)2 + y2 2ay(t 1/t) = 0
[y a(t 1/t)]2 = 0, which has equal roots.
Hence x + a = 0 is a tangent to the circle with diameter AB.
Illustration:

Find the locus of the centre of the circle described on any focal chord of
aparabola as diameter.
Solution:
Let the equation of the parabola be y2 = 4ax.
Let t1, t2 be the extremities of the focal chord. Then t1 . t2 = 1.
The equation of the circle on t1, t2 as diameter is
(x at22) (x at22) + (y 2at1) (y 2at2) = 0
or x2 + y2 ax (t12 + t22) 2ay (t1 + t2) + a2 t12 t12 + 4a2 t1t2 = 0
x2 + y2 ax (t12 + t22) 2ay (t1 + t2) 3a2 = 0. ( t1t2 = 1)
If (,) be the centre of the circle, then = a/2 (t 12+t22 ) If (, ) be the centre of the circle,
then = a/2 (t12+t22 )
= a (t1 + t2) (t1 + t2)2 =2/a2 t12 + t22 + 2t1t2 =2/a2 2/a-2= 2/a2
2a 2a2 = 2 2 = 2a ( a).
Hence locus of (, ) is y2 = 2a(x a).
Focal Distance of a Point
The focal distance of a point P on the parabola

y2 = 4ax is the distance between the point P and the focus S, i.e. PS. Thus thefocal
distance of P = PS = PM = ZN = ZA + AN = a + x.
or

PS = a + at2 = a(1 + t2).


Position of a point relative to a Parabola
Consider the parabola y2 = 4ax.
If (x1, y1) is a given point and y12 4ax1 = 0, then the point lies on the parabola. But when
y12 4ax1 0, we draw the ordinate PM meeting the curve in L. Then P will lie outside
the parabola if PM > LM, i.e., PM2 LM2 > 0.

Now, PM2 = y12 and LM2 = 4ax1 by virtue of the coordinates of L satisfying the equation of
the parabola. Hence, the condition for P to lie outside the parabolabecomes y12 4ax1 >
0.
Similarly, the condition for P to lie inside the parabola is y12 4ax1 < 0.

Chord
Intersection of a Straight Line with a Parabola
The combined equation of straight line y = mx + c and parabola
y2 = 4ax gives us the co-ordinates of point(s) of their intersection. The combined equation
m2x2 + 2x (mc 2a) + c2 = 0 will give those roots. The straight line therefore meets
the parabola at two points.
Points of Intersection of a straight line with the parabola y2 = 4ax
Points of intersection of y2 = 4ax and y = mx + c are given by (mx+c)2=4ax

i.e. m2x2 + 2x(mc 2a) + c2 = 0. (i)


Since (i) is a quadratic equation, the straight line meets the parabola in two points, real,
coincident, or imaginary. The roots of (i) are real or imaginary according as {2(mx 2a)} 2
4m2c2 is positive or negative, i.e. according as amc + a 2 is positive or negative, i.e.
according as mc is less than or greater than a, (taking a as positive).
Note:
When m is very small, one of the roots of equation (i) is very large; when m is equal to zero,
this root is infinitely large. Hence every straight line parallel to the axis of
the parabola meets the curve in one point at a finite distance and in another point at an
infinite distance from the vertex. It means that a line parallel to the axis of
theparabola meets the parabola only in one point.

Length of the chord


As in the preceding article, the abscissae of the points common to the straight line y = mx +
c and the parabola y2 = 4ax are given by the equation m2x2 + (2mx 4a) x + c2 = 0.

Hence, the required length of chord

Illustration:
Find the Length of the chord intercepted by the parabola y2 = 4ax from the line y =
mx + c. Also find its mid-point. Solution:
Simply by applying the formula o length of the joining (x1, y1) and (x2, y2) we get,
Length of the chord = ((x1-x2 )2+(y1-y2 )2 )
= ((x1-x2 )2+m2 (x1-x2 )2 )
= |x1 x2| (1+m2 ) = 4 (a(a-mc) ) (1+m2 )
[ x1+x2=(-2(m-2a) )/m2 and x1 x2=c2/m2 ]
The midpoint of the chord is ((2a-mc)/m2 ,2a/m)

Tangent to a Parabola
Let P(x1, y1) and Q(x2, y2) be two neighbouring points on the parabola y2 4ax. Then
the equation of the line joining P and Q is
y y1 = (y2-y1) / (x2-x1 ) (x x1) (1)
Since, points P and Q lies on the parabola, we have
y12 = 4ax1 (2)
y22 = 4ax2 (3)
(equation 3 and 2) give

y22 y12 = 4a(x2 x1)


(y2-y1)/(x2-x1 )=4a/(y1+y2 )
Equation of chord PQ (i.e. equation (1) becomes):
y y1 = 4a/(y1+y2 ) (x x1) (4)
Our aim is to in the equation of tangent at point P. For that, let point Q approach point P i.e.
x2 x1 and y2 y1.
y y1 = 4a/(2y1 ) (x x1)
y1y = 2a (x1 + x) (using equation (2))
This is the required equation of the tangent to the parabola y2 4ax at P(x1, y1).

Note:
The angle between the tangents drawn to the two parabolas at the point of their
intersection is defined as the angle of intersection of two parabolas.
Tangent at the point (x1, y1)
Let the equation of the parabola be y2 = 4ax.
Hence, value of dy/dx at P(x1, y1) is 2a/y1 and the equation of the tangent at P is
y y1 = 2a/y1 (x x1) i.e. yy1 = 2a(x x1) + y12.

yy1 = 2a(x + x1).

Alternatively, we write the equation of the chord joining the points P(x1, y1) and Q(x2, y2)
on the parabola y2 = 4ax. Equation of the chord is (x-x1)/(x2-x1 )=(y-y1)/(y2-y1 )
or (x-x1)/(x2-x1 ) = (y-y1 )(y2+y1 )/(y22-y12 )= (y-y1 )(y2+y1 )/((x2-x1 ) )
or 4a(x x) = (y y1) (y2 + y1).
When the two points P and Q tend to coincide, y 2 y1 and the line PQ becomestangent
to the parabola. Its equation is 4a (x x1) = (y y1) (2y1) = 2yy1 2y12= 2yy1 8ax1 or
yy1 = 2a(x + x1).

Tangent in terms of m
Suppose that the equation of a tangent to the parabola y2 = 4ax (i)
is y = mx + c. (ii)
The abscissae of the points of intersection of (i) and (ii) are given by the equation (mx +
c)2 = 4ax. But the condition that the straight line (ii) should touch the parabola is that it
should meet the parabola in coincident points
(mx 2a)2 = m2c2 (iii)
c = a/m. (iv)
Hence, y = mx + a/m is a tangent to the parabola y2 = 4ax, whatever be the value of m.
Equation (mx + c)2 = 4ax now becomes (mx a/m)2 = 0.
x = a/m2 and y2 = 4ax y = 2a/m.
Thus the point of contact of the tangent y = mx + a/m is (a/m2 ,2a/m).

Illustration:
Find the condition that the line y = mx + c may touch the Parabola y2 = 4ax and also
find its point of contact.
Solution:

Equation of Parabola is yy2 = 4ax (1)


Slope of tangent at any point is
dy/dx=2a/y = m (say) (2)
y = 2a/m
from (1), x = a/my2
point of contact is (a/my2 ,2a/m)
Equation of tangent is
y 2a/m = m (x-a/my2 )
or y = mx + a/m
Therefore, the condition that y = mx + c touches the Parabola
yy2 = 4ax is c = a/m.

Illustration:
Find the equation of normal to the Parabola yy2 = 4ax, having slope m.
Solution:
Slope of tangent at any point is
dy/dx=2a/y
Slope of normal at that point is
-y/2a = m (say)
Point of contact of a normal having slope m with the Parabola
yy2 = 4ax is (amy2, 2am)
So, equation of normal at this point is
y + 2am = m (x amy2)
or y = mx 2am amy3.

Illustration:
If the line 2x + 3y = 1 touches the Parabola yy2 = 4ax, find the length of its latus
rectum.
Solution:
Equation of any tangent to yy2 = 4ax is
y = mx + a/m my2x my + a = 0.
Comparing it with the given tangent 2x + 3y 1 = 0, we find
my2/2=(-m)/3=a/(-1), m = (-2)/3, a = m/3 = -2/9.
Hence the length of the latus rectum.
= 4a = 8/9, ignoring the negative sign for length.
Alternative Solution:
Writing the given equation as
y = -2/3 x + 1/3=-2/x x-(2/9)/(-2/3), we find that a = 2/9.
Hence the length of the latus rectum = 4a = 8/9.
tangent at the point t
Equation of the tangent at t is ty = x + aty2. T
the point of intersection of the tangents at t1 and t2 is (at1t2, a(t1 + t2)).

Illustration:
One the Parabola yy2 = 4ax, three points E, F, G are taken so that their ordinates are
in G.P. Prove that the tangents at E and G intersect on the ordinate of F.
Solution:
Let the points E, F, G be (at1y2, 2at1), (at2y2, 2at2), (at3y2, 2at3) respectively. Since the
ordinates of these points are in G.P., t22 = t1t3. tangents at E and G are t1y = x + at12 and
t3y = x + at32. Eliminating y from these equation, we get x = at1t3 = at22. Hence the point lies
on the ordinates of F.

Illustration:
Prove that the area of the triangle formed by three points on a parabola is twice the
area of the triangle formed by the tangents at these points.

The intersection of the tangents, at these points, are the points


(at1, t2, a(t1 + t2)}, {at2t3, a(t2 + t3)}, {at3 t1, a(t3 + 1)}.
The area of the formed by these points=1/2 a2(t1 t2) (t2 t3) (t3 t1).

Equation of the Tangents from an External Point


Let y = mx + a/m be any tangent to y2 = 4ax passing through the point (x1, y1).
Then, we have
y1 = mx + a/m or m2x1 m1 + a = 0
If m1 and m2 are to roots of (i) then
m1 + m2 = y1/x1 and m1m2 = a/x1
Also the two tangents are y = m1x + a/m1 , and y = m2x + a/m2
Their combined equation is
(y m1x a/m1 ) (y m2x a/m2 ) = 0
On solving this we get
(y2 4ax) (y12 4ax1) = [yy1 2a (x + x1)]2
SS1 = t2.
Where S = y2 4ax, S1 = y12 4x1
Let y2 = 4ax be the equation of a parabola and (x1, y1) an external point P. Then, equation
of the tangents is given by
SS1 = t2, where S = y2 4ax, S1 = y12 4ax1, T = yy1 2a(x + x1).

If the tangents from the external point (x1, y1) touch the parabola at P and Q, then PQ is
the chord of contact of the tangents.

Illustration:
Prove that through any given point (x1, y1) there pass, in general, twotangents to
the parabola y2 = 4ax.
Solution:
The equation to any tangent is y = mx + a/m. (1)
If this passes through the fixed point (x1, y1), we have
y1 = mx1 + a/m, i.e. m2x1 m y1 + a = 0. (2)
This is a quadratic equation and gives two values of m (real or imaginary). Corresponding to
each value of m we have, two different tangents. The roots of (2) are real and different if
y12 4ax1 > 0, i.e. If the point (x1, y1) lies outside the curve. The roots are equal, if y12
4ax1 = 0 i.e. if the point (x1, y1) lies on the curve. In this case the two tangent merge into
one. The two roots are imaginary if y12 4ax1 < 0, i.e. if the point (x1, y1) lies within the
curve.

Chord of Contact
The chord joining the points of contact of the tangents on the parabola from an external
point is called the chord of contact.
Let the tangent drawn from the point P(x1, y1) touch Parabola at Q(x2, y2) and R(x3, y3)
then QR is the chord of contact of the point P(x1, y1) with respect to y2= 4ax.
The equation of tangents at Q and R are
yy2 = 2a(x + x2) (1)
yy3 = 2a(x + x3) (2)

Since (ii) and (iii) pass through (x1, y1) so we have


y1y2 = 2a(x1 + x2) (3)
y1y3 = 2a(x1 + x3) (4)
From (ii) and (iv) we find that the points Q(x2, y2) and R(x3, y3) lie on yy1 = 2a (x2+ x1), which
being of first degree in x and y represents a straight line. Hence the equation of the chord
of contact of P(x1, y1) is
yy1 = 2a (x + x1) and is of the form T = 0.
Equation of the chord of contact of the tangents drawn from a point (x1, y1) to
theparabola y2 = 4ax is T = 0, i.e. yy1 2a(x + x1) = 0.

Note:
The equation of the chord of the parabola y2 = 4ax with mid point
(x1, y1) is T = S1.

Illustration:
Find the equation of the chord of the parabola y2 = 12x which is bisected at the
point (5, 7).

Solution:
Here (x1, y1) = (5, 7), and y2 = 12x = 4ax a = 3.
The equation of the chord is S1 = T

or y12 4ax1 = yy1 2a(x + x1) or (7)2 12.5 = y(7) 6 (x + 5).


Or 6x + 7y + 19 = 0.

Normal to a Parabola
Normal at the point (x1, y1)
The equation of the tangent at the point (x1, y1) is yy1 = 2a(x + x1). Since the slope of the
tangent = 2a/y1, slope of the normal is y1/2a. Also it passes through (x1, y1).
Hence, its equation is y y1 = -y1/2a (x x1). (i)
Normal in terms of m
In equation (i), put -y1/2a = m so that y1 = 2a and x1 = (y12)/4a = am2, then the equation
becomes y = mx 2am am3
where m is a parameter. Equation (ii) is the normal at the point (am 2, 2am) of
theparabola.

Note:
If this normal passes through a point (h, k), then k = mh 2am am3.
For a given parabola and a given point (h, k), this cubic in m has three roots say m1, m2,
m3 i.e. from (h, k) three normals can be drawn to the parabola whose slopes are m1, m2, m3.
For this cubic, we have m1+ m2+ m3 = 0, m1 m2 + m2 m3 + m3 m1 = (2a h)/a, m1 m2 m3 =
k/a.
If we have an extra condition about the normals drawn from a point (h, k) to a
given parabola y2 = 4ax then by eliminating m1, m2, m3 from these four relations between
m1, m2, m3, we can get the locus of (h, k).

Since the sum of the roots is equal to zero, the sum of the ordinates of the feet of
the normals from a given point is zero. These points are called Co-Normal Points.

Illustration:
Find the locus of the point of intersection of two normals to a parabolawhich are at
right angles to one another.
Solution:

The equation of the normal to the parabola y2 = 4ax is


y = mx 2am am3.
It passes through the point (h, k) if
k = mh 2am am3 => am3 + m(2a h) + k = 0. (1)
Let the roots of the above equation be m1, m2and m3. Let the
perpendicularnormals correspond to the values of m1 and m2 so that m1 m2 = 1.
From equation (1), m1 m2 m3 = -k/a. Since m1 m2 = 1, m3 = k/a.
Since m3 is a root of (1), we have a (k/a) 3+k/a (2a h) + k = 0. k2 + a(2a h) + a2 =
0
k2 = a(h 3a).
Hence the locus of (h, k) is y2 = a(x 3a).

Normal at the point t


The normal, being perpendicular to the tangent at (at, 2at) is given by y = tx + 2at + at 3.
Note:
If normal at the point at1 meets the parabola again at the point at2, then at2 = at1
2/at1 .
Point of intersection of the normals to the parabola y2 = 4ax (at12, 2at1) and (at2t2,
2at2) is (2a+a(t12+t22+t1t2), a t1t2(t1+t2)).

Illustration:
Prove that the normal chord to a parabola at the point whose ordinate is equal to
the abscissa subtends a right angle at the focus.
Solution:
If the normal to the parabola y2 = 4ax at P(at1t2, 2at1) meets it again at the point t2,
then we have t2 = t1 2/t1 .
If the abscissa and the ordinates of P be equal, then at 12 = 2at1

t1 = 2 (rejecting t1 t= 0)

=21=3

The co-ordinates of P and Q are therefore (4a, 4a) and (9a, 6a) respectively.
The focus is the point S (a, 0).
Slope of PS = and slope of QS = .
PSQ = right angle. Hence the result.

Illustration:
Find the locus of the middle points of the normal chords of the parabolay2 = 4ax.
Solution:
Equation of the normal chord at any point (at2, 2at) of the parabola is
y + tx = 2at + at3. (1)
Equation of the chord with mid point (x1, y1) is T = S1
or yy1 2a(x + x1) = y12 4ax1 or yy1 2ax = y12 2ax1. (2)
Since equations (1) and (2) are identical, 1/y1 =t/(-2a)=(2at+at3)/t = 2a + ((2a)/y1 )2
or -(y12)/2a + x1 = 2a + 4a3/(y12 ) or x1 2a = (y12)/2a+4a3/(y12 )
Hence the locus of the middle point (x1, y1) is
x 2a = y2/2a+4a3/y2 .

Illustration:
P and Q are the points t1 and t2 on the parabola y2 = 4ax. If thenormals to
the parabola at P and Q meet at R, (a point on the parabola), show that t1t2 = 2.
Solution:
Let the normals at P and Q meet at R(at2, 2at).
Then t = t1 2/t1 and t = t2 2/t2 .
Therefore t1 + 2/t1 = t2 + 2/t2 (t1 t2) = 2(t1-t2 )/(t1 t2 ) t1t2 = 2.

Illustration:
Find the equations of the normals to the parabola y2 = 4ax at the extremities of
its latus rectum. If the normals meet the parabola, again at P and Q, prove that PQ = 12a.
Solution:
The ends of the latus rectum are (a, 2a) and (a, 2a). The equations of the normals
to the parabola at these points are (put t = 1 and 1)
y + x = 3a and y x = 3a.
These

lines meet the parabola again at P(9a, 6a) and Q(9a, 6a) respectively.

PQ = 6a + 6a = 12a.

Propositions on the Parabola

(i) The tangent at any point P on a parabola bisects the angle between the focal chord
through P and the perpendicular from P on the directrix.
The tangent at P (at2, 2at) is ty = x + at2.
It meets the x-axis at T(at2, 0).
Hence ST = a (1 + t2).

Also, SP = (a2 (1+t2 )2+4a2 t2 ) = a(1 + t2) = ST, so that


MPT = PTS = SPT TP bisects SPM.

(ii) The portion of a tangent to a parabola cut off between the directrix and the
curve subtends a right angle at the focus.
Let P(at2, 2a), be a point on the parabola y2 = 4ax.
The tangent at P is ty = x + at2.
Point of intersection of the tangent with the directrix x + a = 0 is (a, at a/t).
Now, slope of SP is (2at-0)/(at2-a)=2t/(t2-1)
and slope of SK is (at-a/t-0)/(-a-a)=-(t2-1)/2t
(Slope of the SP).(Slope of SK) = 1.
Hence SP is perpendicular to SK i.e. KSP = 90.

(iii) Tangents at the extremities of any focal chord intersect at right angles on the
directrix.
Let P(at2, 2at) and P(at12, 2at1) be the end points of a focal part on the parabola.
Then t.t1 = 1. Equations of the tangents at the point P and the point P are ty = x + at 2 and
t1y = x + at12 respectively.
Let these tangents intersects at a point (h, k). Then h = att1 and k = a(t + t1).
Since the tangents are perpendicular, tt1 = 1 h a.
Hence the locus of the point (h, k) is x = a which is the equation of the directrix.

(iv) Any tangent to a parabola and the perpendicular on it from the focus meet on
the tangent at the vertex.
Equation of the perpendicular to the tangent ty = x + at 2 (1)
From the focus (a, 0) is tx + y = at. (2)
and (2) intersect at x = 0 which is the equation of the tangent at the vertex.

Pole and polar of a conic


The locus of the point of intersection of tangents drawn at the extremities of the chord
of the conic drawn through a point is called the polar of that point with respect to the conic.
This point itself is called the pole.
Equation of the polar of a point (x1, y1) with respect to the parabola y2 = 4ax.
Let us draw the chord QR from the point P(x1, y1) and if the tangents drawn from point
Q and R meet at the point T(h, k), required locus of (h, k) is polar. Since QR is the chord of
contact of tangents from (h, k), its equation is
ky = 2a(x + h)
This straight line passes through the point (x1, y1), we have
ky1 = 2a(x1 + h) (1)
Since the relation (1) is true, it follows that point (h, k) always lies on the line.
yy1 = 2a(x + x1) (2)
Hence (2) is the equation to the polar of pole (x1, y1)

Co-normal Points:
The three points on the parabola, the normals at which pass through a common point,
are called the co-normal points.

Diameter:
The locus of the middle point of a system of parallel chords of a parabola is called its
diameter.

Let the parabola be


y2 = 4ax. (i)
let y = mx + c (ii)
be a system of parallel chords to (i) for different chords, only c varies, m remains constant.
y2 = 44a (y c)/m
my2 4ay + 4ac = 0
let y2 and y3 be the roots of (iii), then y2 and y3 are the ordinates of the points where (ii) cuts
(i)
from (iii), y2 + y3 = 4a/m
Also, if (x1, y1) be the midpoint of the chord them
y1 = (y2-y3)/2=2a/m
Locus of (x1, y1) is y = 2a/m, which is the equation of the diameter.
Note:
y = 2a/m is a straight line parallel to the axis of the parabola.

Solved Examples
Example 1:
Find the equation of the parabola whose focus is (3, 4) and directrix is the line

parallel to 6x 7y + 9 = 0 and directrix passes through point (3/2,2).


Solution:
Let (x, y) be any point on the parabola. Then by definition, the distance between (x, y)
and the focus (3, 4) must be equal to the length of perpendicular from (x, y) on directrix.
So first we will find the equation of the directrix.
The line parallel to 6x 7y + 9 = 0 is
6x 7y + 6 = 0 (1)
Since directrix passes through (3/2,2), this point wil satisfy equation 91) hence
6 (3/2) 7 (2) + k = 0
K = 9 + 14 = 5.
Equation of directrix is 6x 7y + 5 = 0
Now by definition of parabola,
({(x-3)2+(y+4)2 } )= (6x-7y+5)/(62+72 )
85 {(x 3)2 + (y + 4)2} = (6x 7y + 5)2
49x2 + 36y2 + 84xy 570x + 750y + 2100 = 0

Example 2:
Find the equation of the parabola whose directrix makes an isosceles right angled
triangle of area 4 square units with the axis in the 3rd quadrant and focus is on the line y =
x, 2 units away from the origin.
Solution:
First we find the equation of directrix. Let the directrix form the isosceles triangle OAB
with OA = OB = a.

Then according to the given condition,


( OAB) = 4
1/2 a2 = 4
a = 22

{the triangle in 3rd quadrant=-22}

Therefore the co-ordinate, of A and B are (22, 0) and (0, 22) respectively.
So, equation of directrix
(y 0) = ((0+22)/(-22-0)) (x + 22) x + y + 22 = 0
Now the focus S is on line y = x and 2 units away from the origin i.e.
OS = 2 point (2, 2) by definition of parabola, we have
((x-2)2+(y-2)2 )=|(x+y+22)/(12+12 )|
x2 + y2 82x 82y 2xy = 0

Example 3:
The x and y co-ordinate of any point P are expressed as x = (v cos ) t, y = (v sin )
t 1/2 gt2, where t is parameter and v, , g are constants, show that the locus of the point
P(x, y) is a parabola. Find the coordinate of the vertex of thisparabola.
Solution:
We are given x = (v cos ) t (1)
y = (v sin ) t 1/2 gt2 (2)

We will eliminate t from above two relations,


From (1) : = x/(v cos )
Put this value in (2)

From equation (3) it is clear that this is a equation of parabola whose vertex is the point
with the co-ordinates
((v2 sin cos )/g,(v2 sin2 )/2g)

Example 5:
Find the equation of the common tangents to the parabola y2 = 32x and x2 = 108y.
Solution:
The equation of the tangent to the parabola y2 = 4ax, is

y = mx + a/m (1)
The equation of tangent to the parabola
y2 = 32x (2) is
y = mx + 8/m (3)
If this line given by (3) is also a tangent to the parabola
x2 = 108y, then (3) meets x2 = 108y (4)
in two coincident points
Substituting the value of y from (3) in (4) we get
x2 = 108 [mx+(8/m) ]
mx2 108m2x 864 = 0
The roots of this quadratic are equal provided b2 = 4ac.
i.e. ( 108 m2)2 = 4m (864)
m = (-2)/3 (m 0, from geometry of curves)
Substituting their value of m in (3), the required equation is
y = (-2)/3 x + 8/(2/3)
y = (-2)/3 x 12
2x + 3y + 36 = 0

Example 6:
Find the equation of the tangent to the parabola y2 = 6x, which is parallel to the line
cutting intercepts 3 and 4 on x and y axis respectively.
Solution:
The line cutting intercepts 3 and 4 on x and y-axis respectively is,
x/3+y/4 = 1
4x + 3y = 12 (1)

Slope of line (1) is 4/3


The equation of the parabola is y2 = 4 (3/2) x (2)
The equation of any tangent to (2) is
y = mx + (3/2m) (3)
for all value of m
If this tangent is parallel to line (1), then
m = (-4)/3
Hence from (3) required equation of the tangent is
y = (-4)/3 x + (3/2(-3)/4)
y = (-4)/3 x-9/8
32x + 24y + 27 = 0

Example 7:
Find the angle of intersection of the parabola y2 = 8x and x2 = 27y.
Solution:
The given parabolas are
y2 = 8x (1)
and x2 = 27y (2)
Solving (1) and (2) we get
(x2/27)2 = 8x
x4 = 5832x
x4 5832x = 0
x(x3 5832) = 0
x = 0, x = 18

Substituting these values of x in (2) we get y = 0, 12


The point of intersection are (0, 0), (18, 12)
At the point (0, 0)
y2 = 8x dy/dx=4/y dy/dx|(0,0) =
x2 = 27y dy/dx=2x/27 dy/dx|(0,0) = 0
the two curves intersect at the point (0, 0) at right angle.
At the point (18, 12)
y2 = 8x dy/dx|(18,12) =4/12=1/3 = m1 (say)
x2 = 27y dy/dx=2x/27 dy/dx|(18,12)= (218)/27=4/3 = m2 (say)
Let be the angle at which the two curves intersect at the point (18, 12)
Then tan acute = |(m2-m1)/(1+m1 m2 )|=3/13
acute = tan-1 (3/13)

Example 8:
Prove that (x + a)2 = (y2 4ax), is the locus of the point of intersection of the
tangents to the parabola y2 /4.= 4ax, which includes an angle
Solution:
Let two tangent to the parabola y2 = 4ax (1)
be yt1 = x + at12 (2)
and yt2 = x + at22 (3)
Let the point of intersection of the tangent be (x1, y1) then solving equation (1) and (2) we
get,
x1 = at1 t2
y1 = a(t1 + t2)
Also the slope of these tangents are 1/t1 and 1/t2
If be the angle between these two tangents then

tan = + ((m1-m2)/(1+m1 m2 ))=((1/t1 -1/t2 ))/(1+1/t1 1/t2 ).


tan = ((t2-t1)/(1+t2 t1 ))
we are given = /4
tan /4 = 1 = ((t2-t1)/(1+t2 t1 ))
(1 + t1t2)2 = (t2 t1)
{1+(x1/a)2 } = (t1 t2)2 4t1t2 = (y1/a)2-4x1/a
(x1 + a)2 = y12 4ax1
Required locus of (x1, y1) is (x + a)2 = (y2 4ax)

Example 9:
Prove that normal at one end of latus rectum of a parabola is parallel to the tangent
at the other end.
Solution:
Let the parabola be y2 = 4ax (1)
The end points of latus rectum are (a, 2a) & (a, 2a) and
The equation of the normal to (1) at (a, 2a) is
(y 2a) = (x a)
y = x + 3a (2)
The equation of the tangent to (1) at (a, 2a)
y + 2a = (x a)
y=xa
and from (2) and (3), we find that the slope of normal to one end of the latus rectum is equal
to the slope of tangent at other end of tangent to the other end. Hence the required fact is
proved.

Example 10:

Find the length of the focal chord of parabola y2 = 4ax whose one end point is P(at2,
2at)
Solution:
Let Q (at12, 2at) be the other end of this focal chord.
The equation of the line joining (at2, 2at) and (at12, 2at1) is
(y 2at) = (2at1-2at)/(at12-at2 ) (x at2)
If this passes through the focus (a, 0), then
2at = 2/(t1+t) (a at2)
t (t + t1) = 1 t2
tt1 = 1
t1 = 1/t
Then Q is the point (a/t2 ,(-2a)/t)
The length of the focal chord PQ
= a |t+1/t| ((t-1/t)2+4)
a (t-1/t)2

Example 11:
Prove that the locus of the point of intersection of two mutually perpendicular tangents
one to each of the parabola y2 = 4a (x + a) and y2 = 4b (x + b), is a line parallel to y-axis.
Solution:
The given parabolas are
y2 = 4a(x + a) (1)
and y2 = 4b(x + b) (2)
Any tangent to (1) is y = m (x + a) + a/m (3)
Similarly any tangent to (2) is y = m (x + b) + b/m (4)
mm = 1

or m = -1/m
(4) becomes y = {(x + b)/m} bm (5)
The required locus is obtained by eliminating m between (3) and (5).
For this subtracting (5) from (3), we get
0 = x (m+1/m)+a(m+1/m)+b(m+1/m)
x+a+b=0
This is the required locus which is parallel to y axis.

Example 12:
If two tangents to a parabola intercept a constant length on any fixed tangent, find
the locus of their point of intersection.
Solution:
Let yt = x + at2 (1)
be a fixed tangent to the parabola y2 = 4ax (2)
Let the other two tangent to (2) be
yt1 = x + at12 (3)
And yt2 = x + at22 (4)
The point of intersection of (1) with (3) and (4) are P {att1, a(t1 + t2)} and Q{at t2, a (t +
t2)}
Given PQ is constant or PQ2 = constant
a2t2 (t1 t2)2 + a2(t1 t2)2 = constant
a2(t2 + 1) (t1 t2)2 = constant
(t1 t2)2 = constant since t is constant as (1) is a fixed tangent.
(t1 t2)2 = c (say) (5)
Let (x1, y1) be the point of intersection of (3) and (4),

then x1 = at1t2 and y1 = a(t1 + t2) (6)


We know that
(t2 t1)2 = (t1 + t2)2
From equation (5), (6) and 97) we get
c = (y1/a)2- 4x1/a
or y12 = 4x1 a a2c = 4a (x1+1/4 ac)
the locus of (x1, y1) is
y2 = 4a (x+1/4 ac)
The required locus is a parabola whose latus rectum is 4a i.e. equal to latus rectum of
y = 4ax.
2

Example 13:
Find the locus of the poles of normals to parabola y2 = 4ax.
Solution:
Any normal to the parabola y2 = 4ax is (1)
y = mx 2am am3 (2)
Let (x1, y1) be the pole of (2) with respect to (1), then (2) is the polar of (x 1, y1) w.r.t (1)
i.e.
yy1 = 2a (x + x1)
comparing (2) & (3), we get
2a/m=y1/1=(2ax1)/(-2am-am3 )
Hence we get
x1 = 2a am2 (4)
and y1 = 2a/m (5)
Eliminating m between (4) & (5) we get

y12 (x1 + 2a) + 4a3 = 0


The required locus of (x1, y1) is
(x + 2a) y2 + 4a3 = 0

Example 14:
Find the locus of the point P if the perpendicular from that point P upon its polar with
respect to parabola, is of constant length.
Solution:
Polar of P (x1, y1) with respect to the parabola y2 = 4ax is
y1y = 2a (x1 + x)
or y1y = 2ax 2ax1 = 0 (1)
We are given that the distance of P (x1, y1) from line (1) is constant, say
|y12-2ax1-2ax1 |/(y12+(2a)2 ) = (constant)
Locus of (x1, y1) is
(y2 4ax)2 = 2 (y2 + 4a2)

Example 15:
Find the locus of the point, from which the three normals to the parabola y 2 = 4ax
cut the axis at points whose distance from the vertex are in A.P.
Solution:
Any normal to the parabola y2 = 4ax is y = mx 2am am3 (1)
If (1) passes through (x1, y1) then
y1 = mx1 2am am3
am3 + m (2a x1) + y1 = 0 (2)
m1 + m2 + m3 = 0 (3)

m1m2 + m2m3 + m3m1 = ((2a-x1 ))/a (4)


m1 m2 m3 = (-y1)/a (5)
Also, the normal with slope m1, i.e. y = m1x 2am1 am13 cuts axis of theparabola at
the point A(2a + am12, 0) similarly, normals with slope m2 and m3 cut the axis at B(2a +
am22, 0) and c(2a + am32, 0).
OA, OB and OC are in A.P. (given) where O is origin or vertex of the parabola.
2OB = OA + OC (6)
2m22 = m12 + m32
(m1 + m2 + m3)2 2 (m1 m2 + m2 m3 + m3 m1)
= 0 2 ((2a-x1)/a)
m22 = 2/3 ((x1-2a)/a) (7)
From equation (3), we have
m22 = (m1 + m3)2
= m12 + m32 + 2 m1 m3
= 2m22 + 2 (-y1/am2 ) (using 6 and 5)
m32 = -2y1/a (8)
Cubing equation (7) and squaring equation (8) we get
27 ay12 = 2(x1 2a)3
Locus of point (x1, y1) is
27 ay2 = 2(x 2a)3
Example 16:
Show that the locus of the mid point of all focal chords, of a parabola is also a
parabola.
Solution:
Method 1:
Let the parabola be given by

y2 = 4ax
Then its focus is (a, 0). Let (x1, y1) be the mid point of a chord of the given parabola. Then
it equation is
S1 = T
or y12 4ax1 = y.y1 2a (x + x1)
It passes through the focus (a, 0) of (1), then
y12 4ax1 = 0.y1 2a (a + x1)
y12 = 2a(x1 a)
The required locus of (x1, y1) is
y2 = 2a (x a), which is a parabola.
Method 2:
Equation of chord AB is
y (t1 + t2) = 2(x + at1t2) (1)
It passes through the focus (a, 0) (2)
0 = 2(a + at1 + t22) t1t2 = 1
2h = a(t12 + t22) and k = a (t1 + t2) (3)
2h = a((t1 + t2)2 2t1t2)
= a ((k/a)2+2) using (2) and (3)
k2 = 2a (h a)
Locus of M (h, k) is
y2 = 2a (x a)

Example 17:
A tangent to the parabola y2 + 12x = 0 cuts the parabola y2 = 4ax at P and Q. Find
the locus of middle points of PQ.

Solution:
Any tangent to the parabola y2 = 4bx is y = mx b/m
y = mx 3/m
Let (x1, y1) be the mid point of PQ, where P and Q are point of intersection o line (1) and
y = 4ax
2

Equation of chord PQ is
S1 = T
y12 4ax1 = y1y 2a(x + x1)
y1y 2ax y12 + 2 ax1 = 0 (2)
Equation (1) can be written as
my m2 x + 3 = 0 (3)
equation (2) and (3) represent the same line
m/y1 =m2/2a=3/(2ax1-y12 ) (4)
m = 2a/y1 (from 4)
Again, from (4), we get
m/y1 =3/(2ax1-y12 )
2a/y1 =3/(2ax1-y12 )
Locus of (x1, y1) is
4a2x = y2 (3 + 2a)

Example 18:
The normal at any point P of the y2 = 4ax meets the axis in G and to thetangent at
the vertex at H. If A be the vertex and the rectangle AGQH be completed, prove that the
locus of Q is x3 = 2ax2 + ay2.
Solution:

Any normal to the parabola y2 = 4ax (1)


is y = mx 2am am3 (2)
This normal (2) meets the axis y = 0 of (1) in G and the tangent at the vertex i.e. x = 0
in H
The coordinates of G and H are (2a + am2, 0) and (0, 2am am3) respectively. Also the
vertex A is (0, 0). Let Q be (x1, y1)
Given that AGQH is a rectangle. AQ and GH are its diagonals and therefore there mid
points are same. Now the mid point of AQ is (1/2 x1,1/2 y1 ) and that of GH is
[1/2 (2a+am2+0) 1/2 (0-2am-am3 ) ]
i.e. [1/2 (2a+am2 )-1/2 (2am+am3 ) ]
The mid points coincide so we have
1/2 x1 = 1/2 (2a + am2), 1/2 y1 = -1/2 (2am + am3)
or x1 = 2a + am2, y1 = (2am + am3)
The required locus of Q is obtained by eliminating m between these.
Now y12 = (m2a2) (2 + m2)2
= a (m2a) [2+am2/a]2
= a (x1 2a) (x1/a)2
ay12 = (x1 2a) x12
So the locus of Q is
ay2 + 2ax2 = x3

Ellipse
Ellipse is one of the easiest topics in the Conic Sections of Co-ordinate Geometry in
Mathematics.
"Ellipse" is defined as the locus of a point which moves such that the ratio of its distance
(Eccentricity) from a fixed point (Locus) and a fixed line (Directrix) is less than one i.e. a
point moves such that its distance from a fixed point is always less than the distance from a
fixed line, we get a different types of curve for one value of eccentricity, which are similar for

all values of eccentricity less than one. Thus curve looks like a circle but it is not exactly a
circle. Rather it is more like the edges of an egg. And if we plot the movement of the Earth
and other planets around the Sun, it is the same curve satisfying the above condition of
eccentricity less than one. This beautiful curve has been named as "Ellipse".
In this chapter we will discuss in detail the nature/properties of this beautiful and important
curve. As you will see, the curve is symmetrical about two axes. We will study the standard
form of ellipse where the X and y-axes will be taken as these axes. The main emphasis in
this chapter should be on learning the properties ofellipse. The judgement of using
parametric co-ordinates, which can reduce the complexity of the problem, should also be
learnt.
Topics Covered under Ellipse are:-

1. Basic Concepts
2. Tangent and Normal
3. Propositions of an Ellipse
4. Solved Examples of Ellipse

Basic Concepts
If we slice an egg obliquely there will appear a typical curve by its edge. We find the similar
but larger curve if we trace the curve of the movement of earth around the sun. Our
mathematicians and scientist, named this curve as the ellipse. Ellipse one of the conic
sections is obtained by cutting one nappe of cone with a plane that does not pass through
the vertex.
Definition
An ellipse is locus of a point, which moves in a plane such that the ratio of its distances
from a fixed point and a fixed line is constant and always less than one.
In other words "Ellipse" is a conic for which the eccentricity
e < 1. Let S be the focus of ellipse, P any point on the ellipse and PM perpendicular
distance of the directrix from P, then
SP/PM = e < 1
Let S be the focus and ZM be the directrix of the ellipse. Let be itseccentricity.

We draw SZ perpendicular to the directrix and divide SZ internally and externally in the
ratio e : 1 and let A and A' be the internal and external point of division.

Then we have SA = e AZ
And SA' = e A'Z

...... (1)
...... (2)

.. A and A' lie on the ellipse.

Let AA' = 2a and take O the midpoint of AA' as origin. Let P(x, y) be any point on
theellipse referred to OA and OB as co-ordinate axis.
Then from figure it is evident that
AS = AO - OS = a - OS
AZ = OZ - OA = OZ - a
A'S = A'O + OS = a + OS
A'Z = OZ + OA' = OZ + a
Substituting these values in (1) and (2), we have
a - OS = e (OZ - a)
a + OS = e (OZ + a)

...... (3)
...... (4)

Adding (3) and (4), we get


2a = 2 e OZ
Or

OZ = a/e

...... (5)

Subtracting (3) from (4), we get


2 OS = 2ae => OS = ae

...... (6)

.. The directrix MZ is x = OZ = a/e and the co-ordinate of the focus S are (OS, 0)
i.e. (ae, 0). Now as P(x, y) lies on the ellipse.
So we get
SP = e PM or SP2 = e2 PM2
(x - ae)2 + y2 = e2 [OZ - x co-ordinate of P]2
=> (x - ae)2 + y2 = e2 [a/e - x]2 = (a - ex)2

...... (7)

=> x2 + a2e2 - 2axe + y2 = a2 + e2x2 - 2aex


or x2/a2 + y2/a2(1-e2) = 1

[Dividing each term by a2 (1 - e2)]

or x2/a2 + y2/b2 = 1 where b2 = a2 (1 - e2)


This is the standard equation of an ellipse, O is called the centre of the ellipse, AA' and
BB' are called the major and minor axes of ellipse (where b < a).
There exists a second focus and second directrix for the curve. On the negative side of
the origin take a point S', which is such that SO = S'O = ae and another point Z' such that
ZO = OZ' = a/e.

Draw Z'K' perpendicular to ZZ' and PM' perpendicular to Z'K'


The equation (7) may also be written in the form
(x + ae)2 + y2 = (a + ex)2
=>

S'P2 = e2 (PM')2

Hence, any point P on the curve is such that its distance from S' is e times to its
distance from Z'K' so we should have obtained the same curve, if we had started with S' as
focus, a Z'K' as directrix and the same eccentricity.

Pause:
We have considered a > b, now if we consider b > a, what will be the shape of
the ellipse x2/a2 +y2/b2? In this case the major axis AA' of the ellipse is along the y-axis and
is of length 2b. See figure.

The minor axis of BB' = 2a. The foci S and S' are (0, be) and (0, -be) respectively.
The directrix are MZ and M'Z' given by y = + b/e, respectively. Also here a 2 = b2 (1 - e2).
Note:
Let P(x1, y1) be any point. This point lies outside, on or inside the ellipse (8) according as
x21/a2 + y21/b2 = 1 > 0 or = 0 or < 0.
Central Curve
A curve is said to be a central curve if there is a point, called the centre, such that every
chord passing through it is bisected at it.
Latus rectum:
The length of a chord through the focus and at right angle to the major axis of
the ellipse is known as the latus rectum of the ellipse.

There being two foci of an ellipse, there are two rectum, which are of equal length.
yL = b2/a
.. The length of latus rectum LSL' = 2b2/a

Notes:

The major axis AA' is of length 2a and the minor axis BB' is of length 2b.

The foci are (-ae, 0) and (ae, 0).

The equations of the directrices are x = a/e and x = -a/e.

The length of the semi latus rectum = b2/a.

Circle is a particular case of an ellipse with e = 0.


Focal Distance of a Point
Since S'P = ePN', SP = ePN,
S'P + SP = e(PN + PN')

e (NN') = e(2a/e) = 2a

=>

the sum of the focal distances of any point on the ellipse is equal to its major axis.

Another definition of an ellipse


Let ellipse be x2/a2 +y2/b2 = 1

...... (i)

Its foci S and S' are (ae, 0) and (-ae, 0). The equation of its directrices MZ and M'Z'
are x = a/e and x = -a/e respectively. Let P(x 1, y1) be any point on (i)
Now SP = e PM = e NZ = e (OZ - ON) = e[(a/e)-x 1] = a - ex1

and S'P = ePM' = e (Z'N) = e (OZ' + ON) = e[(a/e) + x1] = a + ex1

.. SP + S'P = 2a = AA'
So by this property an ellipse can also be defined as "the locus of a point which moves such
that the sum of its distances from two fixed point is always constant.
Other Forms

If in the equation x2/a2 +y2/b2 = 1, a2 < b2, then the major and minor axis of
theellipse lie along the y and the x-axis and are of lengths 2b and 2a respectively. The foci
become (0, + be), and the directrices become y = + b/e where e = (1-a2/b2 ). The length of
the semi-lactus rectum becomes a2/b2.

If the centre of the ellipse be taken at (h, k) and axes parallel to the x and the yaxes, then the equation of the ellipse is (x-h)2/a2 +(y-k)2/b2 = 1.

Let eh equation of the directrix of an ellipse be ax + by + c = 0 and the focus be


(h, k).

Let the eccentricity of the ellipse be e(e < 1).


If P(x, y) is any point on the ellipse, then
PS2 = e2 PM2

=> (x - h)2 + (y - k)2 = e2(ax+by+c)2/(a2 + b2 ), , which is of the form


ax2 + 2hxy + by2 + 2gx + 2fy + c = 0 ... (*) where
= abc + 2fgh + af2 - bg2 - ch2 0, h2 < ab.
These are the necessary and sufficient conditions for a general quadratic equation given by
(*) to represent an ellipse.

Position of a Point Relative to an Ellipse


The point P(x1, y1) is outside or inside the ellipse x2/a2 + y2/b2 = 1, according as the
quantity ((x12)/a2 +(y12)/b2 -1) is positive or negative.

Parametric Equation of an Ellipse


Clearly, x = a cos, y = b sin satisfy the equation x2/a2 +y2/b2 = 1 for all real values of .
Hence, the parametric equations of the ellipse x2/a2 +y2/b2 = 1 are x = a cos,
y = b sinq where is the parameter.
Also (a cos , b sin ) is a point on the ellipse x2/a2 +y2/b2 = 1 for all values of
(0 < < 2).
The point (a cos, b sin) is also called the point . The angle is called the eccentric angle
of the point (a cos, b sin) on the ellipse.
Draw a circle with AA' (the major axis) as the diameter. This circle is called the auxiliary
circle of the ellipse. The equation of the circle is x2 + y2 = a2. Any point Q on the circle is (a
cos, a sin). Draw QM as perpendicular to AA' cutting the ellipseat P. The x-co-ordinate of
P = CM = a cos.

=> y-co-ordinate of P is b sin


=> P (a cos, b sin).

Illustration:
Find the centre, the length of the axes and the eccentricity of the ellipse 2x2 + 3y2 - 4x 12y + 13 = 0.
Solution:
The given equation can be written as 2(x - 1)2 + 3(y - 2)2 = 1
=>(x-1)2/(1/2)+(y-2)2/(1/3) = 1 => The centre of the ellipse is (1, 2).
The major axis = 2. 12 = 2.
The minor axis = 2.1/3=2/3 => e2 = 1 1/2 = 1/3 => e = 1/3.

Illustration:
Find the equation of the ellipse whose foci are (2, 3), (-2, 3) and whose semi minor axis is
of length 5.
Solution:
Here S is (2, 3), S' is (-2, 3) and b = 5.
=> SS' = 4 = 2ae ae = 2.
But b2 = a2 (1 - e2) => 5 = a2 - 4 => a = 3.
Centre C of the ellipse is (0, 3).

Hence the equation of the ellipse is (x-0)2/9+(y-3)2/5 = 1


=> 5x2 + 9y2 - 54y + 36 = 0.

Illustration:
Find the equation of the ellipse having centre at (1, 2), one focus at (6, 2) and passing
through the point (4, 6).
Solution:
With centre at (1, 2) the equation of the ellipse is (x-1)2/a2 +(y-2)2/b2 = 1. It passes
through the point (4, 6).
=>

9/a2 +16/b2 = 1.

...... (1)

Distance between the focus and the centre = (6 - 1) = 5 = ae


=>

b2 = a2 - a2e2 = a2 - 25.

...... (2)

Solving for a2 and b2 from the equations (1) and (2), we get a2 = 45 and b2 = 20.
Hence the equation of the ellipse is (x-1)2/45+(y-2)2/20 = 1.

Illustration:
Find the equation of the ellipse (in standard form) having latus rectum 5
andeccentricity 2/3.
Solution:
Let the ellipse be x2/a2 +y2/b2 = 1 with a > b.
Latus rectum = 5 = 2b2/a => 2b2 = 5a.

...... (1)

Also b2 = a2 (1 - e2) = a2(1-4/9) = 5a2/9


=> 5a/2 = 5a2/9 => a = 9/2 and hence b2 = 5/2a = 45/4.
The equation of the ellipse, in the standard form, is thus x2/(81/4)+y2/(45/4) = 1.

Illustration:

Find the equation of the ellipse, which cuts the intercept of length 3 and 2 on positive x
and y-axis. Centre of the ellipse is origin and major and minor axes are along the positive xaxis and along positive y-axis.
Solution:
x2/a2 +y2/b2 = 1

...... (1)

According to the given condition the ellipse (1) passes through (3, 0) and (0, 2), so
we have.
9/a2 = 1 => a2 = 9
And 4/b2 = 1 => b2 = 4
Therefore, the equation of the ellipse is x2/9 + y2/4 = 1

Illustration:
Obtain the equation of an ellipse whose focus is the point (-1, 1) whose directrix is the
line passing through (2, 5) having the unit gradient and whose eccentricity is .
Solution:
Let P(x, y) be any point on ellipse.
Its focus is S (-1, 1).
Let the directrix be y = x + c

...... (1)

(. gradient m = 1)
Line (1) passes through (2, 5) so,
5 = 2 + c => c = 3
The directrix is y = x + 3
=> x - y + 3 = 0

...... (2)

Now let PM be the perpendicular from P, drawn to its directrix


(2). By definition of ellipse SP = e PM
or SP2 = e2 PM2
=> (x + 1)2 + (y - 1)2 = (1/2)2 [(x-y+3)/((12+12 ))]2

=> 8[(x + 1)2 + (y - 1)2] = (x - y + 3)2


=> 7x2 + 7y2 + 2xy + 10x - 10y + 7 = 0,
This is the required equation of ellipse.

Tangent and Normal


The ellipse is x2/a2 +y2/b2 = 1

...... (1)

Let P(x1, y1) and Q(x2, y2) be two points on the ellipse. The equation of the line PQ is,
y - y1 = ((y2-y1)/(x2-x1 )) (x - x1)

...... (2)

since points P and Q lies on (1), we get


(y2-y1)/(x2-x1 )=(-b2 (x2-x1 ))/(a2 (y2-y1 ) )
So (2) becomes
y - y1 =(-b2 (x2-x1 ))/(a2 (y2-y1)) (x - x1)

As point Q approaches towards point P along the ellipse, the line PQ tends to thetangent at
P. So, by substituting x1 and y1 for x2 and y2 in the above equation, we have the equation of
the tangent at P as
y - y1 = (-b2 (2x1 ))/(a2 (2y1)) (x - x1)
=> (xx1)/a2 + yy1/b2 =(x12)/a2 +(y`1)/b2 = 1

[as P lies on (1)]

Hence the equation of the tangent to x2/a2+y2/b2= 1 at P(x1,y1) is xx1/a2 + yy1/b2= 1.


...... (3)
Equation of the tangent in terms of 'm'
Let the line y = mx + c

....... (4)

Touch the ellipse x2/a2 +y2/b2 = 1

...... (5)

Eliminating y between (4) and (5), we get;


x2 (b2 + a2m2) + 2a2mcx + a2(c2 - b2) = 0

...... (6)

If (4) touches (5) then the roots f (6) must be coincident i.e. D = 0

i.e. (2a2mc)2 = 4(b2 + a2m2) a2(c2 - b2)


solving this we get c = +(a2 m2+b2)
So the equation of tangent is y = mx + (a2 m2+b2) for all real m

......(7)

From the equation (6) and (7) we get the point of contact as ((+_a2 m)/(a2
m2+b2 ),(b2)/(a2 m2+b ))

Equation of the Tangent to an Ellipse


* Let the equation of the ellipse be x2/a2 +y2/b2 = 1 . Slope of tangent at
(x1, y1) = dy/dx(x1,y1)=-b2/a2 x2/y1
Hence the equation of the tangent at (x1, y1) is y - y1 = (-b2 x1)/(a2 y1)(x - x1)
=>

(xx1)/a2 +(yy1)/b2 = 1.

* Equation of tangent at the point q i.e. (a cos, b sin) is obtained by putting x 1= a cos,
y1 = b sin
=> (x cos )/a+(y sin )/b = 1.

Equation of the Tangent in Terms of its slope; Condition of Tangency


To find the condition that the line y=mx+ c may touch the ellipse x2/a2 +y2/b2 = 1.
Tangent to the ellipse at (a cos, b sin) is (x cos)/a+(y sin)/b = 1.
If y - mx = c is also a tangent to the given ellipse at 'q', then comparing coefficients, we
get
(cos)/am = (sin)/b=1/c or 1/c =(sin)/b=(cos)/(-am)=((sin2+cos2)/(b2+(am)2))=1/(a2 m2+b2 )
=> c = (a2 m2+b2 ).
Therefore, the equation of a tangent to the ellipse x2/a2 +y2/b2 = 1 is
y = mx (a2 m2+b2 ) for all values of m.

Illustration:
Find the locus of the point of intersection of the tangents to the ellipse x2/a2+y2/b2 = 1
(a > b) which meet at right angles.
Solution:
The line y = mx (a2 m2+b2) is a tangent to the given ellipse for all m. Let is passes
through (h, k).
=> k - mh = (a2 m2+b2 ) => k2 + m2h2 - 2hkm = a2m2 + b2
=> m2 (h2 - a2) - 2hkm + k2 - b2 = 0.
If the tangents are at right angles, then m1m2 = -1.
=> (k2-b2)/(h2-a2 ) = - 1 => h2 + k2 = a2 + b2.
Hence the locus of the point (h, k) is x2 + y2 = a2 + b2 which is a circle. This circle is called
the Director Circle of the ellipse.
Note:
The locus of the point of the intersection of two perpendicular tangents to anellipse is a
circle known as the director circle.

Illustration:
Prove that the locus of the mid-points of the intercepts of the tangents to
theellipse x2/a2 + y2/b2 = 1 = 1, intercepted between the axes, is a2/x2 +b2/y2 = 4.
Solution:
The tangent to the ellipse at any point (a cos, b sin)(x cos)/a+(y sin )/b = 1.
Let it meet the axes in P and Q, so that P is (a sec, 0)
and Q is (0, b cosec). If (h, k) is the mid-point of PQ, then h = (a sec)/2.
=> cos = a/2h

and k = (b cosec)/2 => sin = b/2k.

Squaring and adding, we get a2/4h2 +b2/(4k2 ) = 1


Hence the locus of (h, k) is a2/x2 +b2/y2 = 4.

Illustration:
Prove that the product of the lengths of the perpendiculars drawn from the foci to
any tangent to the ellipse x2/16+y2/9 = 1 is equal to 9.
Solution:
For the given ellipse a = 4, b = 3 and hence 9 = 16 (1 - e2)
=> e = 7/4. The foci are thus located at (7,0) and (-7,0).
Equation of a tangent to the given ellipse is
y = mx + (16m2+9)

(as a = 4, b = 3).

Lengths p1 and p2 of the perpendiculars drawn from the foci are


p1 = ((16m2+9)+7 m)/(1+m2 ) and p2 = ((16m2+9)-7 m)/(1+m2)
=>

p1p2 = (16m2+9-7m2)/(1+m2) = 9(1+m2)/(1+m2) = 9.

Note:
The product of lengths of the perpendiculars drawn from the foci to any tangent to
the ellipse x2/a2 +y2/b2 = 1 is b2.

Equation of the Normal to an Ellipse


The normal to a curve is a line perpendicular to the tangent to curve through eh point of
contact.
.. The slope of normal at point (x1, y1) = a2y1/b2x1 and so its equation is (x-x1)/((x1/a2))=(yy1)/((y1/b2 ) ).

* To find the equation of the normal to the ellipse x2/a2 +y2/b2 = 1 at (x1, y1):
Equation of the tangent at (x1, y1) is (xx1)/a2 +(yy1)/b2 = 1
=>

Slope of the normal is a2/b2 +y1/x1; => equation of the normal is


y - y1 = a2/b2 +y1/x1 (x - x1) =>(x-x1)/(x1/a2)=(y-y1)/(y1/b2 ).

* Equation of the normal at (a cos, b sin) is (x-a cos)/((a cos)/a2) = (y-b sin )/((b
sin)/b2).
=> ax sec - by cosec = a2 - b2.

ax1 sec - by1 cosec = a2 - b2.


or ax1((1+t2)/(1-t2)) - by1 ((1+t2)/2t) = a2 - b2, where t = tan /2.
On simplification, this equation gives
by1t4 + 2(ax1 + a2 -b2)t3 + 2(ax1 - a2 + b2)t - by1 = 0.
This is a 4th degree equation in t which gives, in general, four values fo t. Hence from a fixed
point four normals can be drawn to the given ellipse.

Illustration:
If the normals to the ellipse x2/a2 + y2/b2 = 1 at the points (x1, y1), (x2, y2) and (x3, y3) are
concurrent, prove that
= 0.

Solution:
The equation of the normal to the given ellipse at (x1, y1) is
a2xy1 - b2yx1 - (a2 - b2)x1y1 = 0.

...... (1)

Similarly the normals at (x2, y2) and (x3, y3) are


a2xy2 - b2yx2 - (a2 - b2)x2y2 = 0.

...... (2)

a2xy3 - b2yx3 - (a2 - b2)x3y3 = 0.

...... (3)

Eliminating a2x, b2y and (a2 - b2) from (1), (2) and (3), we find that the three lines are

concurrent if

= 0.

Tangent and Normal


Illustration:
If the normals at the end of a latus rectum of the ellipse x2/a2 +y2/b2 = 1 passes through
the extremity of a minor axis, prove that e4+e2-1=0.
Solution:
Equation the normal to te given ellipse at
(ae,b2/a) is (x-ae)/(ae/a2) = (y-b2/a)/(b2/ab2).

Since it passes through (0, - b).


- a2 = - ab - b3
=> a2 = ab + a2 (1 - e2)
or b = ae2 => b2 = a2e4
or a2 (1 - e2) = a2e2 a4 + e2 - 1 = 0.

Enquiry:

How many tangents can be drawn from a point on an ellipse?

We know that y = mx + (a2 m2+b2) is a tangent to the ellipse x2/a2 +y2/b2 = 1 for real
values of m. If this tangent passes through a point (x1, y1) we have y1 = mx1 +
((a2 m2+b2)).
or m2(x12 - a2) - 2x1y1m + (y12 - b2) = 0, which being a quadratic equation in m gives two
values of m. Thus from a point two tangents corresponding two values of m) can be drawn
to an ellipse.

Pair of Tangents and Chord of Contact


From a fixed point (x1, y1) in general two tangents can be drawn to an ellipse. The equation
of the pair of tangents drawn to the ellipse x2/a2 +y2/b2 = 1 is given by (x2/a2 +y2/b2-1)
((x12)/a2 +(y12)/b2-1) = (xx1/a2 + yy1/b2-1)2.

Tangent and Normal


In symbols we write SS1 = T2, where
S x2/a2 +y2/b2 = 1, S1 (x12)/a2 +(y12)/b2 1 and T xx1/a2 + yy1/b2 1

* If from the point P(x1, y1) tangents PQ and PR be drawn to the ellipse x2/a2+y2/b2 = 1,
then the line joining the points of contact Q and R is called the chord of contact. Equation of
the chord of contact is xx1/a2 + yy1/b2 - 1 = 0 or T = 0.
* Equation of the Chord Joining the Points ( cos, sin),
( cos, sin) is
x/a cos((+)/2)+ y/b sin((+)/2) = cos((-)/2)
* Equation of a chord which is bisected at the point (x1, y1) is xx1/a2 + yy1/b2 -1 = (x12)/a2 +
(y12)/b2 - 1 or = S1
* Length of the chord ... (from package).

To find the length of the chord intercepted by the ellipse x2/a2 +y2/b2 = 1 on the
straight line y = mx + c.
Points of intersection of the ellipse and the line are given by x2/a2 +(mx+c)2/b2 = 1
i.e. (a2 m2 + b2)x2 + 2a2 cmx + a2(c2 - b2) = 0

...... (1)

Therefore the straight line meets the ellipse in two points (real, coincident or imaginary).
If (x1, y1) and (x2, y2) be the points of intersection, the length of the chord is
((x1-x2 )2+(y1-y2)2 )=(1+m2 )= |x1 - x2|

...... (2)

(since y1 - y2 = m (x1 - x2))


where x1 and x2 are the roots of the equation (1), and
x1 + x2 = -(2a2cm)/(a2 m2 + b2), x1 x2 (a2 (c2 - b2))/(a2 m2 + b2) so that
(x1 - x2)2 = (x1 + x2)2 - 4x1 x2 = (4a2 c2 m2)/(a2 m2 + b2)2 -(4a2(c2 - b2))/((a2m2
+ b2))=(4a2 b2 (a2 m2 + b2 - c2))/(a2 m2 + b2)2.
Hence the length of the chord is (((1+m2)4a2 b2 (a2 m2 + b2 - c2))/(a2 m2 + b2)2).
i.e. 2ab/(a2 m2 + b2)2) ((1+m2))(a2) m2)+b2)-c2))).
=> e cos(+)/2 = cos(-)/2
=> e [cos /2 cos /2 + sin /2 sin /2] = cos /2 cos /2 + sin /2 sin /2
=> e [1 - tan /2 tan /2] = 1 + tan /2 tan /2 => tan /2 . tan/2 = (e-1)/(e+1).
If the chord passes through (-ae, 0) then tan/2 . tan/2 = (e+1)/(e-1).

Tangent and Normal


Illustration:
The tangent and normal at a point P on an ellipse meet the minor axis at A and B. Prove
that AB subtends a right angle at each of foci.
Solution:
The equations of the tangents and normal at a point P(x1, y1) on the ellipse x2/a2+ y2/b2 =
1 are
xx1/a2 + yy1/b2 = 1

...... (1)

and (x-x1)/((x1/a2))+(y-y2)/((y1/b2)) = 1

...... (2)

Also the minor axis is y-axis i.e. x = 0


Solving (1) and x = 0, we have A (0, b2/y1)
Solving (2) and x = 0, we have B (0, y1 - a2y1/b2)
Let S(ae, 0) be one of the foci of the ellipse. Then the slope of SA = ((b2/y1) - 0)/(0-ae)=
b2/aey1 = m1
(say)
And the slope of SB = ([y1 (a2/b2 ) y1 ]-0)/(0-ae)
= y1/ae ((a2-b2 ))/b2= (y1 a2 e2)/(aeb2) [. b2 = a2(1 - e2)]
= aey1/b2 = m2

(say)

Evidently m1m2 = -1 => SA | SB


i.e. AB subtends a right angle at S(ae, 0). Similarly we can show that AB subtends a right
angle at the other focus S'(-ae, 0)

Illustration:
Prove that the locus of the middle points of the portion of tangent to
the ellipsex2/a2 +y2/b2 = (a+b) between the axis is the curve a/x2 + b/y2 = 4/((a+b)).
Solution:
Any tangent to the ellipse x2/a2 +y2/b2 = 1 , is
y = mx + (a2 m2 + b2)

Therefore the tangent to given ellipse is


y = x + (a(a+b) m2+b(a+b))

...... (1)

(1) Meets x-axis at {-(a(a+b) m2+ b(a+b) )/m,0}


(2) Meets y-axis at {0,(a(a+b) m2+b(a+b))}
Let (x1, y1) be the mid points of the portion of the tangent intercepted between the axes,
then
x1 = 1/2 [{-(a(a+b) m2 + b(a+b))/m} + 0]
y1 = 1/2 [0+{(a(a+b) m2+b(a+b))} + 0]
=> 4x12 = (a(a+b) m2+ b(a+b))/m2
and 4y12 = a(a + )m2 + b(a + b)
Eliminating m, we get
a(a+b)/(4x12) - b(a+b)/(4y12) = am2/(am2+b) + b/(am2+b) = 1
.. The locus of (x1, y1) is a/x2 + b/y2 = 4/(a+b) = 1

Illustration:
If PS1Q and PS2R be two focal chord of the ellipse whose two foci are S1 and S2and the
eccentric angle of P is '' then show that the equation of chord QR is x/a cos + y/b.(1+e 2)/
(1-e2) sin + 1 = 0.
Solution:
Let Q be (a cos , b sin ) and R be (a cos , b sin ) then the equation of the chord QR is
x/ cos(+)/2 + y/b sin(+)/2 = cos(-)/2
which on simplifying becomes
x/a (1-tan /2 tan /2)+y/b (tan /2+tan /2) = 1 + tan /2 tan /2 ..... (1)
Also, PQ and PR are focal chord thus
tan /2 tan /2 = (e-1)/(e+1) and tan /2 tan /2 = (e+1)/(e-1)
(From previous illustration)
On substituting the values of tan /2 and /2 in (1), we get

Tangent and Normal


Illustration:
Show the locus of middle points chord of the ellipse x2/a2 +y2/b2 = 1 which subtend right
angle at the centre is x2/a4 + y2/b4 =(1/a2 +1/b2 ) (x2/a2 + y2/b2)2.
Solution:
Let (x1, y1) be the middle point of chord PQ, then its equation is
T = S1 or 1/a2 + yy1/b2 =(x1/b2)/a2 +(y1/b2)/b2

...... (1)

Since the origin 'O' is the centre so the equation of pair of lines OP and OQ can be obtained
by homogenizing the equation of the ellipse x2/a2 +y2/b2 = 1, with the help of (1), thus

x2/a2 +y2/b2 = (1)2 =


or ((x12)/a2 +(y12)/b2)2 (x2/a2 +y2/b2 ) = (x2 x12)/a4 +(y2 y12)/b4 + (2xyx1 y1)/(a2b2).
It represents a pair of perpendicular lines if
1/a2 ((x12)/a2 +(y12)/b2 )2 - (x12)/a4 + 1/b2 ((x12)/a2 +(y12)/b2)2-(y12)/b4 = 0.
or, (x12)/a4 +(y12)/b4 = (1/a2 +1/b2)(x2/a4 +y2/b4)2.
So the locus of (x1, y1) is
x2/a4 + y2/b4 = (1/a2 + 1/b2 ) (x2/a2 +y2/b2 )2.

Propositions on Ellipse:
Auxiliary circle of an ellipse
Auxiliary circle of an ellipse which is a circle described on the major axis of anellipse as
its diameter.
Let the ellipse be
x2/a2 + y2/b2 =1

...... (1)

Then the equation of its auxiliary circle is x2 + y2 = a2

...... (2)

Take a point P(x1, y1) on (1).


Through P, draw a line perpendicular to major axis intersecting major axis in N and auxiliary
circle in P'.

Let P' be (x1, y2). Then we have (x12)/a2 + (12)/b2 = 1 and

+ y12 = a2

From these two relations, we get:


(y22)/(y12)=(a2-12)/(b2 {1-(x12/a2)}) = a2/b2
=> y2/1 = a/b
Now, let OP' make an angle f with the major axis of the ellipse (1), P' being the
corresponding point of P on the auxiliary circle of the ellipse. Then is called the
eccentric angle of the point P. From the figure it is evident that if (x 1, y1) are the co-ordinates
of P, then x1 = ON = OP' cos = a cos. Also P(x1, y1) is a point on (1).
So solving(x12)/a2 +(12)/b2 =1 for y1, we get y1 = b sin

Therefore (a cos , b sin ) are the parametric co-ordinates of any point P on theellipse,
where is the eccentric angle of P.
Propositions on Ellipse:
Note:
1.

Tangent to the ellipse at the point:

The equation of the line PQ reduces to that of tangent when x/a cos + y/b sin = 1
...... (a)
2.

Normal to the ellipse at the point 'f':

Equation of normal can be derived by using the formula of equation of straight line passing
through point (a cos, b sin) and perpendicular to tangent (a) i.e. (y - b sin ) = (a sin /b
cos ) (x - a cos ).
=> a x sec - b y cosec = a2 - b2

Diameter of an ellipse
The locus of the middle points of a system of parallel chords of an ellipse is called
the diameter of the ellipse.
Let y = mx + c

...... (1)

be the equation of a system of parallel chords of the ellipse x2/a2 + y2/b2 = 1


...... (2)

In (1) m is constant and c varies from chord to chord. Let K (x1, y1) be the midpoint of a
chord PQ of this system.
Eliminating y between (1) and (2) we get
(a2m2 + b2)x2 + 2a2mcx + a2(c2 - b2) = 0

...... (3)

=> x2 + x3 = (-2a2 mc)/(a2 m2+b2 )


But x1 = (x2+x3)/2 =(-a2 mc)/(a2 m2+b2)
Or c = (-x(a2 m2+b2))/(a2 m2)
Also K (x1, y1) is a point on (1) so
y1 = mx1 + c => y1= -b2x1/a2m
.. The locus of K (x1, y1) is y = -b2x/a2m which is a diameter of the ellipse x2/a2 + y2/b2 = 1.

Note:
1. Conjugate Diameters: Two diameters of an ellipse which bisects chords parallel to each
other are called conjugated diameters. Therefore the diameters y = mx and y = m 1x of
the ellipse x2/a2 +y2/b2 = 1 are conjugate if mm1 = -b2/a2.
2. In an ellipse, the major axes bisects all chords parallel to the minor axes and vice-versa,
therefore major axes and minor axes of an ellipse are conjugate diameters but they do not
satisfy the condition
mm1 = -b2/a2 and are the only perpendicular conjugate diameters.
3.

Equi-conjugate diameter:

If the length of two conjugate diameters of ellipse be equal then they are called equi
conjugate diameters.
The equation of equi conjugated diameters are x2/a2 y2/b2 = 1.

4. The eccentric angles of the ends of a pair of conjugate diameters of an ellipsediffer by


a right angle i.e., if one end of a diameter (PQ) is
P(a sin , b cos ).
5. The sum of the squares of any two conjugate semi-diameters of an ellipse is constant
and equal to the sum of the squares of the
semi-axis of the ellipse i.e. OP2 + OP2 = a2 + b2.
6. The produce of the focal distances of a point on an ellipse is equal to the square of the
semi-diameters, which is conjugate to the diameter through the point.
7. The tangents at the ends of a pair of conjugate diameters of an ellipse form a
parallelogram and the area of the parallelogram is constant and is equal to the product of
the axis i.e. equal to 4ab.

Director circle of an ellipse


The director circle is the locus of the point of intersection of pair of perpendicular tangents
to an ellipse.
Two perpendicular tangents of ellipse x2/a2 + y2/b2 = 1 are
y - mx = (a2 m2+b2)
and my + x = (a2+b2 m2)

...... (1)
...... (2)

To obtain the locus of the point of intersection (1) and (2) we have to eliminate m squaring
and adding (1) and (2), we get
(y - mx)2 + (my + x)2 = (a2m2 + b2) + (a2 + b2m)
=> x2 + y2 = a2 + b2, which is the equation of the director circle.

Enquiry:
y1)?

How to find equation of a chord of an ellipse whose mid point is (x 1,

Let the ellipse be x2/a2 + y2/b2 = 1


Any line through (x1, y1) is y - y1 = m(x - x1)

...... (1)
...... (2)

Eliminating y between (1) and (2) we get


x2 (b2 + a2m2) + 2ma2 (y1 - mx1) x + a2 [(y1 - mx1)2 - b2] = 0
If x2 and x3 are the roots of this equation then
x2 + x3 = -2ma2 (y1 - mx1)/(b2 + a2m2)
But (x1, y1) the mid point of the chord.
.. x1 = (x2+x3)/2=(-ma2 (y1-mx1 ))/(b2+a2 m2)
=> m = (-b2 x1)/(a2 y1)
.. From (2) and (3) the equation of the chord where mid point is (x 1, y1) is
y - y1 = ((-b2 x1)/(a2 y1)) (x - x1)
=>

xx1/a2 + yy1/b2 =(x12)/a2 +(y12)/b2

Or

T = S1 where T = xx1/a2 + yy1/b2 -1

and S1 = (x12)/a2 + (y12)/b2 -1

Pole and polar of an ellipse


The locus of the points of intersection of tangents drawn at the point extremities of the
chords passing through a fixed point is called the polar of that fixed point and the fixed point
is called the pole.

Equation of the polar of P(x1, y1) with respect to the ellipse


x2/a2 + y2/b2 = 1 is
xx1/a2 + yy1/b2 =1

Illustration:
How to find out pole of the line lx + my + n = 0 w.r.t. the ellipse x2/a2+y2/b2 = 1.
Solution:
Let (x1, y1) be the pole of line lx + my + n = 0.
w.r.t. the ellipse x2/a2 + y2/b2 = 1

...... (1)
...... (2)

Now the polar of (x1, y1) w.r.t. (2) is


xx1/a2 + yy1/b2 = 1

...... (3)

Since (1) and (3) represents the same polar, so comparing them we have
(x1/a2)/l+(y1/b2)/b2 = (-1)/n
or

x1 = (-a2 l)/n

y1 = (-b2 m)/n

.. The required pole is ((-a2 l)/n,(-b2 m)/n)

Solved Examples of Ellipse:


Example 1:
Find the points on the ellipse x2 + 3y2 = 6 where the tangent are equally inclined to the
axes. Prove also that the length of the perpendicular from the centre on either of
these tangents is 2.
Solution:
The given ellipse is x2 + 3y2 = 6
Or x2/6+y2/2 = 1

...... (1)

If the coordinates of the required point on the ellipse (1) be (6 cos ,2 sin ) then
the tangent at the point is x/6 cos + y/2 sin = 1
...... (2)
Slope of (2) = (-cos )/62/(sin )=(-2)/6 cot

As the tangent are equally inclined to the axes so we have


-1/3 cot = + tan 45o = + 1
.. tan = + 1/3
The coordinates of the required points are
(63/2, 21/2) and (63/2, +-21/2)
= ((32)/2,1/2) and ((32)/2,+-1/2)
Again the length of perpendicular from (0, 0) and (2),
= (6.2)/(2 cos2 + 6 sin2)=(23)/((2,3/4)+(6.1/4) )=(23)/3 = 2

Example 2:
If P be a point on the ellipse x2/a2 + y2/b2 = 2/c whose ordinate is 2/c, prove that the angle
between the tangent at P and SP is tan-1 (b2/ac), where S is the focus.
Solution:
The given ellipse x2/a2 + y2/b2 = 2/c

...... (1)

If (x',(2/c)) be the coordinates of the given point P on the ellipse (1).


Then the tangent at P will be:(xx')/a2 +(yy')/b2 = 1
(xx')/(a2(2/c) )+(yy' ((2/c)))/(b2 (2/c))= 1
The slope of tangent at P is (-b2 (2/c) x')/(a (2/c)(2(2/c)))= m1 (say)
If S be the focus, then slope of PS = = y'/(x'+ae)=(2/c)/(x'+a(2/c)e)= m 2 (say)
If angle between the focal distance SP and tangent at P is , then
tan = (m2-m1)/(1+m2 m1).
(a2 (2/c)+b2 x'2+ aeb2 x' (2/c))/((a2 x'+a3 e(2/c)-b2 x')(2/c))
point (x',(2/c)) lies on ellipse (1), we have
b2 x'2 + a2(2/c)=(2a2 b2)/c
and . a2 - b2 = a2 e2 so we have
(-(2a2 b2)/c + aeb2 x' (2/c))/((a2 e2 x'+a2 e(2/c)) (2/c)) = ((2/c) ab2 ((2/c)

a+ex'))/(a2 e((2/c a+ex')) (2/c)) = b2/ae.


.. = tan-1 b2/ae. Hence proved.
Example 3:
If P, Q, R are three points on the ellipsex2/a2 +y2/b2 = 1 whose eccentric angles are , and
then find the area of PQR.
Solution:
The coordinates of the gives points P, Q, R on the ellipse x2/a2 +y2/b2 = 1, will be (a cos , b
sin ), (a cos , b sin ) and (a cos , b sin ). Area of triangle PQR formed by these points
= 1/2 [x1y2 - x2y1 + x2y3 - x3y2 + x3y1 - x1y3]
= 1/2 [ab cos sin - ab sin cos + ab cos sin - ab sin cos + ab in cos - ab
cos sin ]
= 1/2 ab [2 sin(-)/2 cos (-)/2 + 2 sin (-)/2 cos (-)/2 + 2 sin (-)/2 cos (-)/2]
= ab sin(-)/2 (cos(++2)/2 cos(-)/2)
= 2ab sin (-)/2 cos (-)/2 cos (-)/2.

Example 4:
Find the locus of the extremities of the latus recta of all ellipses having a given major axis
6a.
Solution:
Let LSI be the latus rectum, C be the centre of the ellipse and the coordinates of L be (x, y)
then x = CS = 3 ae
...... (1)
And y = SL = b2/3a =(9a2(1-e2))/3a = 3a (1 - e2)

...... (2)

Eliminating the variable 'e' from (1) and (2) we get eh locus of L.
Hence putting the value of e from (1) and (2), we get
y = 3a(1-x2/9a2)
=> x2 = 9a2 - 3ay
=> x2 = 3a(3a - y), which is clearly a parabola. Similarly we can show that the locus of L' is
x2 = 3ay(y + 3a) which is again a parabola.

Example 5:
Find the equation of the normals at the end of the latus rectum of the ellipsex2/a2 +y2/b2 =
c2 and find the condition when each normal through one end of the minor axis.
Solution:
The ellipse x2/a2 + y2/b2 = c2
=> x2/(a2 c2+y2/(b2 c2) = 1

...... (1)

Then the end point of the latus rectum is (ace,(b2 c)/a)


The normal at this point will be
(x-ace)/(ace/(a2 c2)) = (y-b2 c/a)/((b2 c/a)/(b2 c2))
=> ((x-ace))/e ac = (y-(b2 c)/a) ac
=> (x-ace)/e = y-(b2 c)/a
If this normal passes through (0, - bc), then, we have
(-ace)/e = -bc-b2/a c
=> a = b + a(1 - e2)
=> b - ae2 = 0
=> b/1-e2 => b2/a2 = e4
=> e4 + e2 = 1. This is required condition.

Example 6:
The circle x2 + y2 = 4 is concentric with the ellipse x2/7 + y2/3 = 1; prove that the
common tangent is inclined to the major axis at an angle 30o and find its length.
Solution:
The ellipse x2/7+y2/3=1

...... (1)

The equation to the circle is


x2 + y2 = 4

...... (2)

As the line y = mx + (a2 m2+b2)


i.e. y = mx + (7m2+3)

...... (3)

is always the tangent on the ellipse. If this is also a tangent on the circle (2) then length
of perpendicular from the centre (0, 0) on the line (1) must be equal to radius of circle i.e. 2.
Hence, (7m2+3)/(1+m2) = 2
=> 7m2 + 3 = 22 (1 + m2)
=> 7m2 - 4m2 = 4 - 3
=> m2 = 1/3
=> m = + 1/3
Hence the common tangent to the two curves is inclined at an angle of tan -1 (+1/3) i.e.
30o to the axis.
Note:
We can also prove the above result by using the fact that the line = mx + (7m 2+3) will
be tangent to x2 + y2 = 4 if discriminent of x2 + (m + (7m2+3))2 = 4 is zero.
Let P and Q be the points of contact of the common tangent with ellipse and circle
respectively and O be the common centre of the two, then PQ = (OP 2-OQ2)[ CPQ = 90o]
The coordinates of P are [(-a2m)/(a2 m2 + b2), b2/(a2 m2 + b2)]
i.e. [(-7/3)/(16/3),3/(16/3)]
i.e.[(-7)/4,(33)/4]
and coordinate of O are (0, 0)
So, OP = (((-7)/4)2+ ((33)/4)2) = (19/4)
As OQ = r = 2
.. PQ = (OP2-OQ2 ) = (19/4-4) = 3/2

Example 7:
If q be the angle between CP and normal at point P, on the ellipse a2 x2 + b2 y2 = 1, then
find out tan and prove that its greatest value is (b2-a2)/2ab. C is centre of ellipse and P is
any point on ellipse.

Solution:
The equation of the ellipse be
x2/(1/a2)+y2/(1/b2) = 1

..... (1)

If be the angle between the normal at P = (1/a cos ,1/b sin and PC where C is the
centre of the ellipse given by (1) equation to the normal PG is
x/a sec - y/b cosec = 1/a2 -1/b2
=> bx sec - ay cosec = (b2 - a2)/ab
Its slope = (b sec )/(a cosec ) = b/a tan = m 1 (say)
The slope of PC = (1/b sin )/(1/a cos ) = a/b tan = m2 (say)
tan = (m1-m2)/(1+m1 m2) = (a/b tan -a/b tan )/(1+(b/a) tan (a/b) tan ) = ((b 2-a2 )tan
)/ab(1-tan2 )
(b2-a2)/2ab.(2tan)/(1-tan2 )
tan = (b2 - a2)/2ab sin 2
The value of tan will be maximum when sin 2 is maximum, sin 2 is maximum i.e. sin 2
is 1. Therefore the greatest value of tan is (b2-a2)/2ab.

Example 9:
Find the locus of the point of intersection of the two straight lines (x tan )/a - y/b + tan x =
0 and x/a+(y tan )/b where a is fixed angle. Also find the eccentric angle of the point of
intersection.
Solution:
Equation of the lines are given as
(x tan )/a-y/b + tan = 0

...... (1)

x/a+(y tan )/b - 1 = 0

...... (2)

To find the locus of the point of intersection, we have to eliminate the variable 'tan a' from
(1) and (2), so by (2),
y/b=1/(tan ) (1-x/a) and by (1)
y/b = tan (1+x/a)

Multiplying we get
y/b)2-(1-x/a)(1+x/a)
=> x2/a2 +y2/b2 = 1
This is the equation of an ellipse
Again solving (1) and (2), we get
x = a(1-tan2 )/((1+tan2 ))
x = a(1-tan2 )/(sec2 )
Let the abscissa of the point of intersection be a cos f, then
x = a cos = a(1-tan2)/(sec2 )
=> cos = (1-tan2 )/(sec2 )
=> (1-cos )/(1+cos )=(sec2 -(1-tan2))/(sec2 +(1-tan2 ) )
(By components & dividendo)
= (2 tan2 )/2 = tan2
=> (2 sin2/2)/(2 cos2 ) = tan2
=> tan2 /2 = tan2
=> tan /2 = tan
Hence = 2
Example 10:
If TP and TQ are perpendiculars upon the axes from any point T on the ellipsex2/a2+ y2/b2 =
4. Prove that PQ is always normal to fixed concentric ellipse.
Solution:
The ellipse is given by x2/(4a2 )+ y2/(4b2 ) = 1

...... (1)

If the co-ordinates of T on the ellipse be (2a cos f, 2b sin f) an TP and TQ perpendiculars on


x-axis and y-axis respectively, the co-ordinates of P and Q will be (2a cos f, 0) and (0, 2b sin
f) respectively.
Now equation to PQ is
y - 0 = (2b sin -0)/(0-2a cos )(x - 2a cos )

=> x/(2a cos )+y/(2b sin ) = 1


=> x/a sec + y/b cosec = 2

...... (2)

Now equation to the normal at point (A cos , B sin ) with respect to any other
concentric ellipse
x2/A2 +y2/B2 = 1 is
Ax sec - By cosec = A2 - B2

...... (3)

As (2) and (3) are similar, on comparing then we have,


A/(12a)+B/(12b) = A2 - B2

...... (4)

Solving the two equations given by (4), we get


B = (2a2b)/(a2-b2) and A = (-2 ab2)/(a2-b2 )
So the line (2) i.e. x/2a sec + y/2 cosec = 1 is a normal to the fixed ellipsex2/A2 +y2/B2.
Where A = (-ab2)/(a2-b2)and B =(a2 b)/(a2-b2).

Example 11:
If the straight line y = 2x + 2 meet the ellipse 2x2 + 3y2 - 6, prove that equation to the
circle, described on the line joining the points of intersection as diameters, is 7x 2+ 7y2 + 12x
- 4y - 5 = 0.
Solution:
The line is given as
y = 2x + 2

...... (1)

and the ellipse is given as


x2/a2 +y2/b2 = 1
Solving (1) and (2), we gets
x2/3+(2x+2)2/2 = 1
=> x2/3 + 2(x2 + 2x + 1) = 1
=> 7x2 + 12x + 6 = 3
=> 7x2 + 12x + 6 = 3

...... (2)

Let x1 and x2 be the two roots of this equation.


x1 + x2 = -12/7
and x1x2 = 3/7

...... (3)
...... (4)

Let y1 and y2 be the corresponding ordinates for the abscissa x1 and x2; so the co-ordinates
of the points of intersection will be (x1, y1) and (x2, y2). As these lie on line
y = 2(x + 1)
We have y1 = 2(x1 + 1) and y2 = 2(x2 + 1)
Where y1 + y2 = 2(x1 + x2) + 4
y1y2 = 4(x1 + 1) (x2 + 1)

...... (5)
...... (6)

The equation to the circle drawn with the line joining (x1, y1) and
(x2, y2) as diameter is
(x - x1) (x - x2) + (y - y1) (y - y2) = 0
=> x2 + y2 - x(x1 + x2) - y(y1 + y2) + x1x2 + y1y2 = 0
=> x2 + y2 - x(x1 + x2) y[2(x1 + x2) + 4] + x1x2 +......+ 4[x1x2 + (x1 + x2) + 1] = 0
(By (5) and (6))
Putting the values from (3) and (4)
x2 + y2 - x (-12/7) - y[2(-12/7)+4] + 3/7 + 4[3/7+(-12/7)+1] = 0
= 7x2 + 7y2 + 12x - 4y - 5 = 0. Hence proved.

Example 12:
If the product of the perpendiculars from the foci upon the polar of P be constant and equal
to c2. Find the locus of P.
Solution:
Suppose the equation to the ellipse x2/a2 + y2/b2 = 1. The co-ordinate of foci are (ae, 0) and
(-ae, 0).
Let the co-ordinates of P be (h, k). Then polar of P is

xh/a2 +yk/b2 = 1
Or b2xh + a2yk - a2b2 = 0

...... (1)

If P1 and P2 be the lengths of the perpendiculars on the line (1) from (ae, 0) respectively are
P1 = (b2 hae-a2 b2)/(b2 h2 + a4 k2)
And P2 = (-a2 b2-b2 hea)/(b2 h2+a4 x2)
.. P1P2 = (-b2 h2 a2 e2+a4 b4)/(b4 h2+a4) = c2 (By hypothesis)
=> a4b4 - b4h4a2e2 = c2b4h2 - c2a4k2
=> b4h2 (c2 + a2e2) + c2a4k2 = a4b4
Generalizing the locus of the point P(h, x) is
b4 x2 (c2 + a2 e2) + c2 a4 y2 = a4 b4
Example 13:
Chords of ellipse x2/a2 +y2/b2 = 1 always touch another concentric ellipse x2/2+ y2/2 =
1, show that the locus of their poles is ( 2 x2)/a2 + (2 y2)/b2 = 1.
Solution:
Let (x1, y1) be the pole of a chord of the ellipse
x2/a2 +y2/b2 = 1

...... (1)

Then the equation of this cord is the same as the polar of (x 1, y1) with respect to (1)
i.e. xx1/a2 + yy1/b2 = 1
If (2) touches the ellipse x2/2 + y2/2 = 1, then
(b2/y1)2 = 2{-{b2x1/a2y1}}2 + 2
=> b4/y21 = (2b4x21/a4y21) + 2
=> (2 x12)/a4 +(2 y12)/b4 = 1
.. The locus of (x1, y1) is (2 x2)/a2 +(2 y2)/b4 = 1
Hence proved.

Example 14:

...... (2)

If the straight line y = x tan + ((a2 tan2 +b2)/2), being the angle of inclination,
intersects the ellipse x2/a2 +y2/b2 = 1. Then prove that the straight lines joining the centre to
their point of intersection are conjugate diameters.
Solution:
The equation of the ellipse be
x2/a2 +y2/b2 = 1

...... (1)

and equation to the line is given as


y = x tan + ((a2 tan2 +b2)/2), being angle of inclination.
We can write this equation as,
y = mx + ((a2 m2 +b2)/2)
=> ((y-mx)/2)/(a2 m2 + b2 ) = 1

...... (2)

To get the equation to the lines joining the point of intersection to the origin, making (1)
homogeneous with the help of (2), we have
x2/a2 + y2/b2 = [((y-mx)/2)/(a2 m2 + b2 )]2
= 2((y2 + m2 x2 - 2mxy))/(a2 m2 + b2)
=> (b2 x2 + a2 y2) (a2m2 + b2) = 2a2 b2(y2 + m2x2 - 2mxy)
y2 a2(a2m2 - b2) + 4m2 - b2xy - b2x2 (a2m2 - b2) = 0
=> y2 + (4 mb2)/((a2 m2+b2)) xy -b2/a2 x2 = 0

...... (3)

This equation represents two straight lines y = m1x and y = m2x then the combined equation
will be y2 - (m1 + m2)xy + m1m2x2 = 0.
Comparing (3) and (4); we get
m1m2 = -b2/a2
which is the condition of diameter to be conjugate. Hence the lines are the conjugate
diameters.
Example 15:
The eccentric angles of two points P and Q on the ellipse 1, 2. Find the area of the
parallelogram formed by the tangents at the ends of diameters through P and Q.
Solution:

The ellipse is
x2/a2 +y2/b2 = 1
Equation to the tangent at the points P and Q are
x/a cos 1 + y/b sin 1 = 1

...... (1)

x/a cos 2 + y/b sin 2 = 1

...... (2)

and

Solving (1) and (2), we will have the coordinates of the point of intersection. Multiplying (1)
by sin 2 and (2) by sin 1 and subtracting, we get
x/a (sin 2 cos 1 - cos 2 sin 1) = sin 2 - sin 1
=> x/a sin (2 - 1) = 2 sin (2 - 1)/2cos (2 + 1)/2
.. x = a (cos((1 - 2)/2))/(cos(1 - 2)/2); y = b (sin(1 + 2)/2)/(cos (1 - 2)/2)
Above are co-ordinates of the point of intersection L of tangents at p and Q, i.e. at 1 and 2.
Putting 1 = p + 1 in above we get the co-ordinates of the point of intersection M of
tangent at Q and P' as

Area of the parallelogram LMNO = 4CLM


= 4 . 1/2 (x1 y1 - x2 y2)
= 2 ab/(sin (1 - 2)/2 cos (1 - 2)/2) [ - cos2 (1 + 2)/2-sin (1 + 2)/2]
= (-4ab)/(sin(1 - 2))
= - 4ab cosec (1 - 2) Area can't be (-) ve
So the area = 4ab cosec (1 - 2)

Hyperbola

We have studied earlier about parabolas and ellipses as two conic sections. In this chapter we will
study another conic section called hyperbola which can be obtained by cutting a right circular come
at both the nappe by a plane. Thus it has two branches, one on each nappe.
Hyperbola is the locus of a point which moves in a plane such that it distance from a fixed point is
e(>1) times its distance from a fixed straight line. It is symmetrical about two axes and one branch
is the reflection of other about one of the axes. Since, we have studied ellipse in the previous
chapter, it will be easier to understand this chapter. We will give you a proportion by which you can
easily get the formulae for hyperbola if you know the formulae for ellipse.
Most of the properties of hyperbola are similar to those of the ellipse. We will introduce the
concept of asymptotes. You will also learn about rectangular hyperbolas and conjugate hyperbola.
The rectangular hyperbola can be very simply represented in a parametric form. It is advisable
that this fact should always be kept in the fore while solving problems on and related to rectangular
hyperbola.
Wishing you All the Best for the preparation Hyperbola with askIITians.com.
Topics Covered:
Basic Concepts of Hyperbola
Relation between Focal Distances
Parametric Coordinates
Important Properties of Hyperbola
Examples Based on Hyperbola
Ellipse Vs Hyperbola
Propositions of a Hyperbola
Examples on Propositions of a Hyperbola
Rectangular hyperbola
Intersection of a Circle and a Rectangular Hyperbola
Conjugate hyperbola
Examples on finding locus of point
Solved Examples on Hyperbola Part-I
Solved Examples of Hyperbola (Part-II)

Solved Examples of Hyperbola Part-III

Basic Concepts
As we have studied earlier that by slicing a cone with a plane in different orientations, we
obtain conic sections. The hyperbola is obtained by cutting a right circular cone at both the
nappes by a plane. The hyperbola is to be thought of a single curve consisting of two
branches, one on each nappe.
Definitions
A hyperbola is the locus of a point which moves in a plane such that its distance from a fixed
point (called the focus) is e ( >1) times its distance from a fixed straight line (called the
directrix).
The hyperbola is a conic section for which the eccentricity (e) is greater than unity.
Standard Equation
Let S be the focus and ZM the directrix of a hyperbola.

Since e > 1, we can divide SZ internally and externally in the ratio e : 1; let the points of
division be A and A as in the figure. Let AA = 2a and be bisected at C. Then, SA = e.AZ, SA
= e.ZA
SA + SA = e(AZ + ZA) = 2ae
i.e., 2SC = 2ae or SC = ae.
Similarly by subtraction, SA SA
= e(ZA ZA) = 2e.ZC 2a = 2eSC SC = a/e.
Now, take C as the origin, CS as the x-axis, and the perpendicular line CY as the y-axis. Then,
S is the point (ae, 0) and ZM the line x = a/e. Let P(x, y) be any point on the hyperbola. Then
the
condition

PS2 = e2.(distance of P from ZM) 2 gives (x ae)2 + y2 = e2 (x a/e)2 or x2(1 e)2+ y2 = a2(1
e 2)
i.e. x2/a2 y2/a2(e21) = 1.

(i)

Since e > 1, e2 1 is positive. Let a 2 (e2 1) = b2. Then the equation (i) becomes x2/a2
y2/a2 = 1.
The eccentricity e of the hyperbola x2/a2 y2/a2 = 1 is given by the relation e2 = (1 + b2/a2).
Since the curve is symmetrical about the y-axis, it is clear that there exists another focus S
at (ae, 0) and a corresponding directrix ZM with the equation x = a/e, such that the same
hyperbola is described if a point moves so that its distance from S is e times its distance
from ZM.
The points A and A where the straight line joining the two foci cuts the hyperbola are
called the vertices of the hyperbola.
The straight line joining the vertices is called the transverse axis of the hyperbola, its
length AA is 2a.
The middle point C of AA possesses the property that it bisects every chord of the
hyperbola passing through it. It can be proved by taking P(x 1, y1) as any point on the
hyperbola. If (x1, y1) lies on the hyperbola then so does P(x 1, y1) because the hyperbola is
symmetrical about the x and the y axes. Therefore PP is a chord whose middle point (0, 0),
i.e. the origin C. On account of this property the middle point of the straight line joining the
vertices of the hyperbola is called the centre of the hyperbola.
The straight line through the centre of a hyperbola which is perpendicular to the
transverse axis does not meet the hyperbola in real points. If B and B be the points on this
line such that BC=CB=b, the line BB is called the conjugate axis.
A latus rectum is the chord through a focus at right angle to the transverse axis.
The length of the semi-latus rectum can be obtained by putting x = ae in the equation of
the hyperbola. Thus y = b a2e2/a21 = be21 = b.b/a = b2/a.
Foci and Directrices:
Since the curve is symmetrical about x-axis, therefore there exists another focus at point
(ae, 0) of the hyperbola. (Similar to ellipse).
Corresponding to these foci, there are two directrices whose equations are x = a/e and x =
a/e.

Relation between Focal Distances


The difference of the focal distances of a point on the hyperbola is constant. PM and PM are
perpendiculars to the directrices MZ and MZ and PS PS = e(PM PM) = eMM = e(2a/e)
= 2a = constant.

Another Definition of Hyperbola


A hyperbola can be defined in another way; Locus of a moving point such that the difference
of its distances from two fixed points is constant, would be a hyperbola.
Transverse and Conjugate axes:
The points A(a, 0) & A(a, 0) are called the vertices of the hyperbola and the line AA joining
the vertices is called the transverse axis and the line perpendicular to it, through the centre
(0, 0) of the hyperbola is called conjugate axis.
Centre:
Any chord of the hyperbola through C will be bisected at C (by symmetry), therefore C is
called the centre of hyperbola.
Thus hyperbola is a central curve.
Latus rectum:
The chord of a hyperbola through one of the foci and at right angle to the transverse axis is
called the latus rectum:
If 2l be the length of the latus rectum, then the co-ordinates of one of its extremities is (ae,
l).
The point (ae, l) lies on the hyperbola x2/a2 y2/b2, so we have
e2 (l2/b2) = 1
l2 = b2(e21) = b2a2(e21)/a2 = b4/a2 (? b2 = a2(e2 1)
l = b2/a2
The length of the latus rectum = 2 b2/a2
Relative Position of a Point with respect to the Hyperbola

The quantity x12/a2 y12/b2 = 1 is positive, zero or negative, according as the point (x1, y1)
lies within, upon or without the curve.

Parametric Coordinates
We can express the coordinate of a point of the hyperbola x 2/a2 y2/b2 = 1 in terms of a
single parameter, say .

In the adjacent figure OM = a sec and PM = b tan . Thus any point on the curve, in
parametric form is x = a secq, y = b tan.

In other words, (a sec , b tan ) is a point on the hyperbola for all values of . The point (a
sec, b tan) is briefly called the point .

Important Properties of Hyperbola


Since the fundamental equation of the hyperbola only differs from that of the ellipse in
having b2 instead of b2, it will be found that many propositions for the hyperbola are derived
from those for the ellipse by changing sign of b 2. Some results for the hyperbola x 2/a2y2/b2 =
1 are

The tangent at any point (x1, y1) on the curve is xx1/a2 yy1/b2 = 1.
The tangent at point is x sec/a y tan/b = 1.
The straight line y = mx + c is a tangent to the curve, if c 2 = a2 m2 b2. In other words, y =
mx + a2m2b2 touches the curve for all those values of m when m > b/a or m < b/a.
Equation of the normal at any point (x1, y1) to the curve is xx1/x1/a2 = yy1/y1(b)2.

The

equation

of

the

chord

through

the

points 1 and 2 is
The equation of the normal at is ax cos + by cot = a2 + b2.
Through a given point, four normals (real or imaginary) can be drawn to a hyperbola.
The tangent drawn at any point bisects the angle between the lines, joining the point to
the foci, whereas the normal bisects the supplementary angle between the lines.
Equation of the director circle is x 2 + y2 = a2 b2. That means if a2 > b2, there would exist
several points such that tangents drawn from them would be mutually perpendicular. If a 2 <
b2, no such point exist. For a2 = b2, centre is the only point from which two perpendicular
tangents (asymptotes) to the hyperbola can be drawn.
From any point (x1, y1) in general two tangents can be drawn to hyperbola. The equation of
the pair of tangents is
(x1/a2 y2/b2 1) (x12/a2 y12/b2 1) (xx1/a2 yy1/b2 1)2 or SS1 = T2
The equation of the chord of contact is xx1/a2 yy1/b2 1 = 0 or T = 0.
The equation of the chord bisected at the point (x1, y1) is
xx1/a2 yy1/b2 1 = x12/a2 y12/b2 1 or T = S1.
Equation of the chord the points (a sec , b tan ) and (a sec ?, b tan ?) is
x/a cos ?/2 y/b sin +?/2 = cos +?/2.

Examples Based on Hyperbola


Illustration:
Find the equation of the hyperbola whose directrix is 2x + y = 1, focus is (1,
1) and eccentricity is 3.

Solution:
Let S(1, 1) be focus and P(x, y) be any point on the hyperbola. From P draw
PM perpendicular to the directrix then PM =2x+y1/22+12 = 2x+y1/5
Also from the definition of the hyperbola, we have
SP/PM = e SP = ePM
(x1)2+(y1)2 = 3 (2x+y1/5)
(x 1)2 + (y 1)2 = 3 (2x+y1)2/5
5[(x2+12x)+(y2 + 1 2y)]=3(4x2 + y2 + 1 + 4xy 4x 2y)
7x2 2y2 + 12xy 2x 4y 7 = 0

Illustration:
Find the directrix, foci and eccentricity of the hyperbola
ax2 y2 = 1
Solution:
The given hyperbola is
ax2 y2 = 1
or

x2/(1/a) y2/1 = 1

(1)

which is of the form(x2/a2) (y2/b2) = 1


Here a2 = 1/a, b2 = 1
If e be the eccentricity of the hyperbola, then
b2 = a2(e2 1)
1 = (e2 1)
a = (e2 1)
or e2 = a + 1 or e =a+1
Also the foci are given by (+ ae, 0)
The required foci are
(+ 1/a (a+1), 0)
or (+ a+1/a, 0)
And the directricies are given by x = + (a/e)
x = + [1/a/a+1]
x = + 1/a(a+1)

Illustration:

(? a = 1/a, e = 1/a+1)

Find the locus of a point, the difference of whose distances from two fixed
points is constant.
Solution:
Let two fixed points be S (ae, 0) and S (ae, 0). Let P(x, y) be a moving point
such that
SP SP = Constant = 2a (say).
Then [(xae)2 + (y0)2] [x+ae]2+(y0)2 = 2a
[(xae)2+y2] = + 2a + [(x+ae)2+y2
(x ae)2 + y2 = 4a2 + (x + a2)2 + y2 + 4a [(xae)2+y2]
(x ae)2 (x + ae)2 4a2 = + 4a [(xae)2+y2]
4aex 4a2 = + 4a [(xae)2+y2]
(ex + a)2 = (x + ae)2 + y2
(e2 1)x2 y2 = a2(e2 1)
x2/a2y2/b2 = 1 taking b2 = a2(e2 1)
This is a hyperbola.

Illustration:
If A, B, C are three points on the hyperbola xy = c2 and AC is perpendicular to
BC, prove that AB is parallel to the normal to the curve at C.

Solution:
Let the three points A, B, C respectively be (ct1, c/t1), (ct2, c/t2) and (ct3, c/t3).
Since AC is perpendicular to BC,
(c/t3c/t1/ct3ct1) = 1 t1t2 = 1

(1)

Normal to the curve at C (ct3, c/t3) is


y = xt32 + 2/t3 (1t34) and its slope is t32 = 1/t1t2

(2)

Slope of AB = (c/t2c/t1/ct2ct1) = 1/t1t2 = t32


AB is parallel to the normal at C.

Illustration:
Find the equation of the hyperbola the distance between whose foci is 16,
whose eccentricity is 2 and whose axis is along the x-axis centre being the
origin.

Solution:
We have b2 = a2(e2 1) = a2 b = a.
Also 2ae = 16 ae = 8 a = 42.
Hence the equation of the required hyperbola is
x2/32 y2/32 = 1 x2 y2 = 32.

Illustration:
The perpendiculars drawn from the centre of a hyperbola x2/a2 y2/b2 = 1
upon the tangent and normal at any point of the hyperbola meet them in Q
and R. Find the locus of Q and R.

Solution:
Tangent at any point P(a sec , b tan ) is sec y/b tan = 1. (1)
Equation of the line through centre (origin) perpendicular to (1) is y = a
sin /b x
sin = by/ax
Eliminating from (1), we get x/a cos y/b cos (by/ax) = 1.
x2 + y2 = ax cos (x2 + y2)2 = a2x2(1 b2y2/a2x2)
Or (x2 + y2)2= a2x2 b2y2, which is the locus of Q.
Normal at the point P (a sec , b tan ) is ax cos + by
cot = a2 + b2

(2)

Equation of the line perpendicular to (2) drawn from the centre is


y = bx/a sin

(3)

Form (2) and (3),


sin = bx/ay and ax 1b2y2/a2x2 + by 1b2x2/a2y2 . ay/bx = a2 + b2
(x2 + y2)2 (a2y2 b2x2) = (a2 + b2)2 x2y2, which is the locus of R.

Ellipse Vs Hyperbola
Enquiry: How do we get formulae for hyperbola if we know the
formulae for ellipse?
Most of the results obtained in the case of the ellipse x 2/a2 y2/b2 = 1 hold
good for the hyperbola x2/a2 y2/b2 = 1, when only the sign of b 2 is changed.
The proofs of these results can be derived exactly in the same manner as
they were derived for ellipse. So let us see some final results.
1. Tangent at (x1, y1) to the hyperbola x2/a2 y2/b2 = 1 is xx1/a2 yy1/b2= 1 i.e.
T = 0.
2. Equation of tangent in terms of m is y = mx + (a2m2b2).
3. Equation of the normal at (x 1, y1) to the hyperbola is xx 1/(x1/a2)yy1/
(y1/b2).
4. Equation of pair of tangents drawn from point (x 1, y1) to the
hyperbolax2/a2 y2/b2 = 1 is given by SS1 = T2
Where S = x2/a2 y2/b2 = 1
S1 = x12/a2 y12/b2 = 1
T = xx1/a2 yy1/b2 = 1
5. The Chord of Contact of tangents from (x 1, y1) to the hyperbola x2/a2
y2/b2 = 1 is given by T = 0 i.e. xx1/a2 yy1/b2 = 1.
6. The Polar of Pole (x1, y1) to the hyperbola x2/a2 y2/b2 = 1 is given by T = 0
i.e. xx1/a2 yy1/b2 = 1.

7. The equation of Chord of hyperbola x2/a2 y2/b2 = 1 whose middle point is


(x1, y1) is given by T = S1 i.e. x12/a2 y12/b2 = xx1/a2 yy1/b2 = 1.
Pause:

Try to get above results yourself using traditional methods similar to the
ellipse.
Enquiry: Can we represent a hyperbola mathematically in form of one
parameter and is there any geometrical significance of that parameter like
eccentric angle in case of ellipse?

Yes. Before that let us understand the concept of the auxiliary circle of a
hyperbola. The circle described on the transverse axis of hyperbola as its
diameter is called its auxiliary circle.

We know that the line AA joining the vertices A(a, 0) and A(a, 0) of the
hyperbola x2/a2 y2/b2 = 1 is called the transverse axis.
The equation of the auxiliary circle, described on AA as diameter, is
(x a) [x (a)] + (y 0)] + (y 0)(y 0) = 0
or

x2 + y2 = a2

Now let us draw the foot N of any ordinate NP of the hyperbola draw a
tangent NU to this circle, and join CU. The
CU = CN cos NCU
i.e.

x = CN = a sec NCU

The angle NCU is therefore the angle ?.


Also NU = CU tan ? = a tan ?

So that NP : NU = b : a
So the ordinate of the hyperbola is therefore in a constant ratio to the length
of the tangent drawn from its foot to the auxiliary circle.
When it is desirable to express the co-ordiantes of any point of the curve in
terms of one parameter than we use
x = a sec ?, and y = b tan ?
Note:

This angle ? is not so important an angle for the hyperbolas the eccentric
angle is for the ellipse.

Propositions of a Hyperbola:
Diameter of a hyperbola
The locus of the middle point of a system of parallel chords of a hyperbola is
called its diameter.
The equation of the diameter is y = b2x/(a2m), where m is the slope of the
system of parallel chords.
Note:

Conjugate Diameters:
Two diameters of a hyperbola which bisect chords parallel to each other are
called conjugate diameters.
The diameters y = mx and y = m1x of the hyperbola x2/a2 y2/b2 = 1 are
conjugate if mm1 = b2/a2.
Director circle of a hyperbola
The director circle is the locus of the point of intersection of a pair of
perpendicular tangents to a hyperbola.
Equation of the director circle of the hyperbola x2/a2 y2/b2 = 1 is x2 + y2 =
a2 b2 i.e. a circle whose centre is origin and radius is (a2b2).

Note:

If b2 < a2, this circle is real.


If b2 = a2, the radius of the circle is zero, and it reduces to a point circle at
the origin. In this case the centre is only point from where tangents at right
angle can be drawn to the hyperbola.
If b2 > a2, the radius of the circle is imaginary, so that there is no such circle,
and so no tangents at right angles can be drawn to the circles.
Asymptote
A line, which is tangent to the hyperbola at infinity, but which is not itself at
infinity, is called the asymptote of the curve.
To find the equation of the asymptotes of the hyperbola x2/a2 y2/b2 = 1.
Let y = mx + c be an asymptotic to the given hyperbola. Then eliminating y,
the abscissas of the points of intersection of y = mx + c
or x2(b2 a2m2) 2a2mcx a2(b2 + c2) = 0

(1)

If the line y = mx + c is an asymptote of the hyperbola then it touches the


hyperbola at infinity i.e. both the roots of the equation (1) are infinite and for
this we must have b2 a2m2 = 0 and 2a2mc = 0. Hence we get m = + (b/a)
and c = 0.
The asymptotes are y = + (b/a) x
Or x2/a2 + y2/b2 =

Their combined equation is (x/a+y/b)(x/ay/b) = 0 or xx 2/a2 y2/b2 =


0which shows that the equation of the asymptote differs from that of the
hyperbola in the constant term only. Also the angle between the asymptotes
is 2 tan1 (b/a).
The lines x2/a2+y2/b2=0 are also asymptotes to the conjugate hyperbolax 2/a2
y2/b2=1.

Remarks:

The equation of the hyperbola and that of its pair of asymptotes differ by a
constant. For example, if S = 0 is the equation of the hyperbola, then the
combined equation of the asymptotes is given by S + K = 0. The constant K
is obtained from the condition that the equation S + K = 0 represents a pair
of lines. Finally the equation of the corresponding conjugate hyperbola is S +
2K = 0.

Any line drawn parallel to the asymptote of the hyperbola would meet the
curve only at one point.

Illustration:
Find the hyperbola whose asymptotes are 2x y = 3 and 3x + y 7 = 0 and
which passes through the point (1, 1).

Solution:

The equation of the hyperbola differs from the equation of the asymptotes by
a constant.
The equation of the hyperbola with asymptotes 3x + y 7 = 0 and 2x y
= 3 is (3x + y 7) (2x y 3) + k = 0. It passes through
(1, 1) k = 6.
Hence the equation of the hyperbola is (2x y 3)(3x + y 7) = 6.

Illustration:

Find the angle between the asymptotes of the hyperbola x 2/a2y2/b2 = 1, then
length of whose latus rectum is 4/3 and which passes through the point (4,
2).

Solution:

We have 2b2/a = length of the latus rectum = 4/3 3b2 = 2a


Also, the hyperbola passes through the point (4, 2).
Hence 16/a2 4/b2 = 1 16/a2 6/a = 1
Or a2 + 6a 16 = 0 (a 2)(a + 8) = 0 a = 2 b2 = 4/3.
The asymptotes of the given hyperbola are y = + b/a x or y + 1/3 x.
If 1 and 2 are the angles which the asymptotes make with the positive xaxis, then
tan 1 = 1 = /6 and tan 2 = 1/3 2 = /6.
Hence the angle between the asymptotes = /3.

Illustration:

Prove that the chords of the hyperbola x2/a2y2/b2 = 1, which touch its
conjugate hyperbola are bisected at the point of contact.

Solution:

Let P(x1, y1) be the mid-point of the chord of the given hyperbola, so that the
equation of the chords is xx1/a2yy1/b2 = x12/a2y12/b2.
(1)

If touches the conjugate hyperbola x2/a2y2/b2 = 1, then


x2/a21/b2 [xx1/a2x12/a2+y12/b2]2.b4/y12 +
Simplifying, we find that

will

have

equal

roots.

x2/a2 [y12/b2x12/a2]+2xx1/a2[x12/a2y12/b2][y12/b2x12/a2]2+y12/b2= 0 has equal


roots so that
4x12/a2 [x12/b2y12/a2]+4[y12/b2x12/a2][[y12/b2x12/a2]2y12/b2]= 0
or, x12/a2 [x12/a2y12/b2][x12/a2y12/b2]2 +
x12/a2+y12/b2)+y12/b2= 0

y12/b2 =

0 or (x12/a2y12/b2)(x12/a2

or x12/a2 y12/b2 + 1= 0 (x1, y1) lies on the conjugate hyperbola.


Hence the chord (1) touches the conjugate hyperbola at its midpoint (x1, y1).

Alternative solution:

Any tangent to the conjugate hyperbola x2/a2y2/b2 = 1 is


x = my + b2m2a2.

(2)

If this is same as the chord (1), then m = a2y1/b2x1 and hence


a4/x12 [x12/a2 y12/b2]2 = b2m2 a2 = b2a4y12/b4x12 a2
Or [x12/a2 y12/b2]2 = y12/b2 x12/a2 or x12/a2 y12/b2 = 1
(x1, y1) lies on the conjugate hyperbola.
the chord (1) touches conjugate hyperbola and is bisected at the point of
contact.

Rectangular hyperbola
If the asymptotes of a hyperbola are at right angles to each other, it is called
a rectangular hyperbola.

A hyperbola whose asymptotes are at right angles to each other is called a


rectangular hyperbola. The angle between asymptotes of the hyperbola
x2/a2 y2/b2 = 1, is 2 tan1 (b/a).
This is a right angle if tan1 b/a = /4, i.e., if b/a = 1 b = a.
The equation of the rectangular hyperbola referred to its transverse and
conjugate axes as axes of coordinates is therefore:
x2 y2 = a2.

(1)

Equation referred to asymptotes as coordinate axes: To obtain this equation


we rotate the axes of reference through 45o. Thus we have to write
x/2 + y/2 for x and x/2 + y2 for y.
The equation (i) becomes
(1/2) (x + y)2 (1/2)(x y)2 = a2 i.e. xy = a2 or xy = c2 where c2 = a2/2.
Equation:

We know that the asymptote of hyperbola


x2/a2 y2/b2 = 1
are given by y = + (b/a) x

(1)
(2)

If be the angle between the asymptotes, then


= tan1 (m1m2/1+m1m2) = tan1 ((b/a)(b/a)/1+(b/ab/a))
tan1 (2(b/a)/1(b2/a2)) = 2 tan1 (b/a)
But if the hyperbola is rectangular, then =/2
i.e., /2 = 2 tan1 (b/a)
or tan (/2) = b/a b = a
From (1) the equation of the rectangular hyperbola is x2 y2 = a2
It should be noted that:

(i)
In a hyperbola b2 = a2 (e2 1). In the case of rectangular hyperbola (i.e.,
when b = a) result become a2 = a2(e2 1) or e2 = 2 or e =2
i.e. the eccentricity of a rectangular hyperbola =2
(ii) In case of rectangular hyperbola a = b i.e., the length of transverse axis
= length of conjugate axis.
Hence it is also called an equilateral hyperbola.

Intersection
of
a
Circle
Rectangular Hyperbola

and

A rectangular hyperbola and a circle meet in four points. The mean of these
four points is the middle point of the centres of the hyperbola and that of the
circle.
Let the rectangular hyperbola be xy = c 2 and the equation of the circle be
x2 + y2 + 2gcp + 2fy + k = 0. Any point on the hyperbola is (cp, c/p). If it lies
on the circle, then c2p2 + c2/p2 + 2gcp + 2fc/p + k = 0.
c2p4 + 2gcp3 + kp2 + 2fcp + c2 = 0.
This is fourth degree equation in p, which has four roots. Hence the circle and
the hyperbola intersect in four points. If p 1, p2, p3, p4 are the roots of this
equation, then
p1 + p2 + p3 + p4 = 2gc/c2 = 2g/c cp1 + cp2 + cp3 + cp4 = 2g
x1+x2+x3+x4/4 = g/2
Also 1/p1 + 1/p2 + 1/p3 + 1/p4 = p1p2p3/p1p2p3p4 = 2fc/c2/c2/c2 = 2f/c
c/p1 + c/p2 + c/p3 + c/p4 = 2f y1+y2+y3+y4/4 = f/2.
Hence the mean of the four points is (g/2, f/2) which is the mid-point of the
centre of the hyperbola and that of the circle.
Illustration:

A circle and a rectangular hyperbola meet in four points A, B, C and D. If the


line AB passes through the centre of the circle, prove that the centre of the
hyperbola lies at the mid-point of CD.

Solution:
The line AB passes through the centre of the circle. Hence AB is the diameter
of the circle and the mid-point of AB is the centre of the circle. Let the coordinates of A, B, C, D be respectively (x 1, y1) (x2, y2), (x3, y3) and (x4, y4). Let
the centres of the hyperbola and the circle be (h, k) and (g, f).
Then x1+x2x3+x4/4 = h+g/2. But g = x1+x2/2
2g+x3/x4/4 h+g/2 x3+x4/2 = h
Similarly y3+y4/2 = k.
Hence (h, k) is the mid-point of CD.
Enquiry: As the asymptotes of a rectangular hyperbola are mutually
perpendicular, can we find the equation of a hyperbola whose asymptotes are
the co-ordinate axes?

Let transverse and conjugate axes as axes of co-ordinates (XOX and YOY in
the above figure), the equation of rectangular hyperbola is
X 2 Y2 = a2

(1)

Also we know that the asymptotes of a rectangular hyperbola are at right


angles to each other.

Let Ox and Oy be the asymptotes, each making as angle of /4 with the coordinate axes. Rotate the axes through as angle of /4 with the co-ordinate
axes. To find the equation of the rectangular hyperbola referred to
asymptotes as axes. We have to substitute for X and Y in (1) the xcos(/4)
ysin(/4) and ycos(/4) + xsin(/4) respectively.

i.e. x+y/2 and yx/2 respectively.


So from (1) equation of the rectangular hyperbola referred to asymptotes as
axes is
(x+y/2)2 = (yx/2)2 = a2 xy = a2/2
xy = c2 where 2c2 = a2

The shape of the above rectangular hyperbola referred to asymptotes as coordinate axes is as shown is the adjacent figure.
Note:
1. Parametric co-ordinates of any point on the rectangular hyperbola xy =
c2 is (ct, c/t) where t is the parameter.
2. Equations of tangent and normal at any point (ct, c/t) on the rectangular
hyperbola xy = c2 are x + yt2 = 2ct and xt3 yt ct4 + c = 0 respectively.

Conjugate hyperbola
A hyperbola whose transverse and conjugate axes respectively are the
conjugate and transverse axes of a given hyperbola is called the conjugate
hyperbola of the given hyperbola. The asymptotes of these two hyperbolas
are also the same.
Equation of a conjugate hyperbola is x2/a2 y2/b2 = 1
of the given hyperbola x2/a2 y2/b2 = 1

(1)

Its transverse and conjugate axes are along y and x axes respectively.

Note:

1.Any point on conjugate hyperbola (2) is (a tan q, b sec q)


2.The equation of the conjugate hyperbola to xy = c2 is xy = c2.
3.By comparing the equations of hyperbola x2/a2 y2/b2 = 1.
asymptotes x2/a2 y2/b2 = 0
and conjugate hyperbola x2/a2 y2/b2 = 1 we find that:
(a) The equation of the hyperbola and asymptotes differ by the same
constant by which the equations of the asymptotes and the conjugate
hyperbola differ.
(b) Hyperbola + Conjugate hyperbola = 2 (Asymptotes). (check yourself).
4.The tangents drawn at the points, where a pair of conjugate diameters
meets a hyperbola and its conjugates form a parallelogram, whose vertices
lie on the asymptotes and whose area is constant. (prove yourself).
5.If a pair of conjugate diameters of hyperbola meet the hyperbola and its
conjugate in P, P and D, D respectively, then the asymptotes bisect PD and
PD. (Prove yourself).

Illustration:

If e1 and e2 are the eccentricities of the hyperbola x2/a2 y2/b2 = 1 and its
conjugate hyperbola, prove that e12 + e22 = 1.

Solution:

The eccentricity e1 of the given hyperbola is obtained from


b2 = a2(e12 1).

(1)

The eccentricity of e2 of the conjugate hyperbola is given by


a2 = b2(e22 1).

(2)

Multiply (1) and (2), we get,


1 = (e12 1)(e22 1) 0 = e12 e22 e12 e22

e12 + e22 = 1.

Example:
C is a centre of the hyperbola x2/a2y2/b2 = 1 and the tangent at any point P meets
asymptotes in the point Q and R. Find the equation to locus of the centre of the circle
circumscribing the triangle CQR.

Solution:

This equation to the hyperbola is given as

x2/a2y2/b2 = 1

(1)

Let P any point on it as (a sec ?, b tan ?), then the equation of tangent at P is

x/a y/b sin ? = cos ?

(2)

The equation to the asymptotes to (1) are

x/a = y/b

and x/a = y/b

(3)

(4)

Solving (2) and (3), we get the coordinates of Q as

(acos?/1sin?, bcos?/1sin?)

Solving (2) and (4), we get the coordinates of R as

(acos?/1+sin?, bcos?/1+sin?)

Let O be the centre of the circle passing through C, Q and R having its coordinates as (h,
k). Then clearly OC = OQ

h2 + k2 = (hacos?/1sin?)2 + (kbcos?/1sin?)2

h2 + k2 = (hacos?/1sin?)2 + (kbcos?/1sin?)2

h2 + k2 = h2 + k2 + (a2 + b2) cos2?/(1sin?)2 (2ah + 2bk) cos?/1sin?

2(ah + bk) = (a2 + b2) cos?/1sin?

(5)

Similarly OC = OR

Hence h2 + k2 = (hacos?/1+sin?)2 + (k+bcos?/1+sin?)2

Which on simplification as in the last case, given

2(ah bk) = (a2 + b2) cos?/1sin?

(6)

to get the locus of the point O we have to eliminate f from (5) and (6), so multiplying the
two we get

4(a2h2 b2k2) = (a2 + b2) cos2?/1sin2? = (a2 + b2)2

for (h, k), we get the required locus as

4(a2x2 b2y2) = (a2 + b2)2

Example:

A straight line is drawn parallel to the conjugate axis of a hyperbola meets it and the
conjugate hyperbola in the points P and Q. Find the locus of point of intersection of
tangents at P and Q.

Solution:

Let the equation to the hyperbola be

x2/a2y2/b2 = 1

and its conjugate hyperbola be

(1)

y2/b2x2/a2 = 1

(2)

Let p be any point (a sec ?, b tan ?) on P. The equation of the line parallel to the
conjugate axis (1) i.e. y-axis passes through P will be
x = a sec ?
(3)

The line (3) will cut the conjugate hyperbola (2) at Q where x = a sec ? and hence y = b
(1+sec2?), therefore the coordinates of Q will be {asec?, b(1+sec 2?)}

Now the equation to the tangent to (1) at P is

x/ay/b sin ? = cos ?

or

x/a cos ? = y/b sin ?

and the equation to the tangent to (2) at Q is

y/b (1+sec2?) sec ? = 1

(4)

x/a + cos ? = y/b (1+cos2?)

(5)

on squaring and adding (4) and (5), we have

2x2/a2 + 2 cos2 ? = y2/b2 [(1 + cos2 ?) + sin2 ?] = 2 y2/b2

cos2 ? = y2/b2 x2/a2

Putting the value of cos ? in (5) we get

x/a + (y2/b2x2/a2) = y/b (1+y2/b2x2/a2)

Squaring we have,

x2/a2 + y2/b2 x2/a2 + 2 x/a (y2/b2 x2/a2) = y2/b2 (1+y2/b2x2/a2)

2x/a (y2/b2x2/a2) = y2/b2(y2/b2x2/a2)

y2/b2 (y2/b2x2/a2) = 2x/a

y4/b4 (y2/b2x2/a2) = 4x2/a2

There is the required locus.

Example:

From a point A, perpendiculars AB and AC are drawn to two straight lines OB and OC. If
the area OBAC is constant, find the locus of A.

Solution:

Let the bisectors of the angles BOC be taken as axis. So the equations of OB and OC are
respectively.

x cos + y sin = 0

and x cos y sin = 0

Take any point A as (h, k); then

where = 1/2 BOC

AB = Perpendicular from A on OB

= hcos+ksin/cos2+sin2 = h cos + k sin

(1)

and similarly

AC = Perpendicular from A on C

= h cos k sin

(2)

The equation to AB will be

(h x) sin + (y k) cos = 0

y cos x sin + h sin k cos = 0

Similarly the equation AC will be

(3)

(h x) sin (y k) cos = 0

Now OB = Perpendicular distance of (3) from (0, 0).

= 00+hcos+ksin/cos2+sin2

= h sin k cos

Similarly OC = perpendicular distance of (0, 0) from (4)

= h sin + k cos .

Now the area of quad. OBAC = OAB + OAC

= 1/2 OB AB + 1/2 OC AC

= 1/2 [h sin kcos][h cos + k sin] + 1/2 [h cos k sin] [h sin + k cos]

= (h2 k2) sin.cos = constant = S (say)

h2 k2 = {s/sina, cosa} which is again constant = a2 (say)

Therefore the locus of the point (h, k) will be x2 y2 = a2, which is hyperbola.

Solved Examples on Hyperbola


Example:

Show that the line 4x 3y = 9 touches the hyperbola 4x2 9y2 = 27.
Solution:

We know that if the line y = mx + c touches the hyperbola x2/a2y2/b2 = 1,


then c2 = a2m2 b2
Here the hyperbola is
x2/(27/4)y2/(27/9) = 1

i.e. here a2 = 27/4

b2 = 27/9 = 3

And comparing 4x 3y = 9 with y = mx + c, we get


m = 4/3,

c = 3

a2m2 b2 = (27/4)(4/3)3 3 = 12 3 = 9 = (3)2


or a2m2 b2 = c2

Hence the given line touches the given hyperbola.


Example:
Prove that the mid points of chords of the hyperbola x2/a2y2/b2 = 1 parallel
to the diameter y = mx lie on the diameter a2my = b2x
Solution:
The hyperbola is x2/a2y2/b2 = 1

(1)

The equation of any chord parallel to the diameter


y = mx is y = mx + c

(2)

Eliminating y between (1) and (2), we get


x2/a2(mx+c)2/b2 = 1
x2(b2 a2m2) 2a2m cx a2(b2 + c2) = 0
b2x3 = a2my3
The locus of (x3, y3) is
b2x = a2my

Hence proved.

Example:

Prove that the angle subtended b any chord of a rectangular hyperbola at the
centre is the supplement of the angle between the tangents at the end of the
chord.
Solution:

Let the equation of the hyperbola be x2 y2 = a2 and P and Q be any two


points on it such that their coordinates are respectively (a sec ?1, a tan ?1)
and (a sec ?2, a tan ?2) and C be the centre of the hyperbola.
Equation of the line PC is y 0 = atan?10/asec?10 (x 0)
y = x sin ?1

(1)

Similarly equation to QC will be y = x sin ?2

(2)

If a be the angle between PC and QC, then


tan = sin?1sin?2/1+sin?1sin?2

(3)

Again the equation to the tangent at P is


x a sec ?1 y a tan ?1 = a2
y = x/sin?1 acos?1/sin?1

(4)

Similarly the equation to the tangent at Q2 is


y = x/sin?2 a cos?2/sin?2
If b be the angle between the tangents at f1 and f2, then
tan = 1/sin?2 1/sin?2/1+1/sin?1 1/sin?2 = sin?2sin?1/1+sin?1sin?2
= (sin?1sin?2)/1+(sin?1sin?2)
tan = tan
tan = tan ( )

(By (3))

Hence proved.

Example:
The normal to the hyperbola 16x 2 9y2 = 144 meets the axes in M and N. MP
and NP and are drawn at right angles to the axes. Find the locus of P.
Solution:
The equation to the hyperbola is
x2/a2 y2/b2 = 1

(1)

Let L be any point on it having the coordinates (3 sec ?, 4 tan ?) then the
equation to the normal at this point will be given by
3x sin ? + 4y = (9 + 16) tan ?

(2)

Let this normal cut the axis of x at M whose coordinates are (x, 0) and the
axis of y at N whose coordinates are (0, y) solving (2) with y = 0, we get
x = 9+16/3cos? = 25/3cos?

(3)

Similarly solving (2) with x = 0, we get


y = (9+16)tan?/4 = 25tan?/4

(4)

If PM and PN be the lines parallel to the axes, the coordinates of


P = (x, y) will be clearly given by (3) and (4). The required locus of P will be
obtained by eliminating ? from (3) and (4).
Using the fact sec2 ? tan2 ? = 1, we get
9x2 16y2 = 625
Example:
Prove that a circle can be drawn through the foci of a hyperbola and the
points at which any tangent meets the tangents at the vertices of the
hyperbola.
Solution:
Recall:

If four points (x1, y1), (x2, y2), (x3, y3) and (x4, y4) are concyclic then

These points are (ae, 0), (ae, 0), (a, b tan ?/2) and (a, b cot f/2)

apply : R1 R1 R2 and expand along R1, we get

= (2ae)
= (2aeb) [(a2 e2 a2 b2) cot ?/2 + 9a2e2 b2 a2) tan ?/2] = 0
points S, S, R and Q are conclycic.
Example:
On a level plain the crack of the rifle and the thud of the ball striking the
target are heared at the same instant, prove that the locus of the hearer is a
hyperbola.
Solution:
Suppose A to be the target and B to be the firing point and let the hearer be
at a point P. If V1 and V2 be the velocity of the sound and bullet respectively,

then the time taken by the bullet in reaching from B to the target A =
BA/v2 and the time taken by the sound in reaching from A to P = AP/v1.
Again the time taken by the sound in reaching from B to P = BP/v1.
As by hypothesis the sound reaches the hearer at P, simultaneously, the time
taken by the sound to reach upto the position P from then target together
with the time taken by the bullet to reach the target must be same as the
time taken by the sound in reaching from the rifle upto P.
Hence BA/v2 + AP/v1 = BP/v1
BA/v2 = BP/v1 AP/v1 = 1/v1 (BP AP)
BP AP = v1/v2 AB
As v1, v2 and AB are constants, hence
BP AP = Constant
Therefore the locus of the point P is the hyperbola having foci at A and B and
transverse axis equal to v1/v2 AB
Example:
Prove that the locus of the pole of a chord of the hyperbola which subtends a
right angles at the vertex, is, x = a2b2/a2+b2.

Solution:
The coordinates of the vertex are (a, 0). Transferring the origin to this point,
the equation of the hyperbola x2/a2y2/b2 becomes;
(x+a)2/a2 y2/b2 = 1
x2/a2y2/b2 = 2x/a
b2x2 a2y2 = 2ab2x

(1)

The equation to the polar of (h, k) w.r.t hyperbola is given by


b2hx a2hky = a2b2

(2)

After transformation the equation (2) becomes

b2(x + a)h a2yk = a2b2


or

b2hx a2yk = a2b2 ab2h

(3)

The equation of the lines joining the points of intersection of the hyperbola
and the chord to the origin is obtained by making (1) homogeneous with the
help of (3). Hence on simplification, this equation becomes
(a2b2 ab2h) (b2x2 a2y2) = 2ab2x(b2xh a2yk)
If they are at right angles, the sum of the coefficients of x2 and y2 must be
zero; hence
b2 a2 + 2b2h/ah = 0
generalizing for (h, k), we get the required locus as
x = a a2b2/a2+b2

Example:
Find the locus of intersection of tangent to a hyperbola, which meet at a
constant angle .

Solution:
Let the equation to the hyperbola be
x2/a2y2/b2 = 1

(1)

Equation to any tangent to (1) is


y = mx + a2m2b2
If the tangent passes through a point (h, k) when we must have
k = mh + (a2m2b2)

or

m2(h2 a2) 2mhk + (k2 + b2) = 0

(2)

Let m1 and m2 be the two roots of this equation.


m1 = tan1 and m2 = tan2, we have
tan1 + tan2 = 2hk/h2a2
tan1 tan2 = k2+b2/h2a2
and as (tan1 tan2)2 = 4h2k24(k2+b2)(h2a2)/(h2a2)2.
= 4(a2k2b2h2+a2b2)/(h2a2)2
If the two tangents met at an angle , clearly = (1 2).
Hence cot = cot(1 2)
= 1/tan(12) = 1+tan1tan2/tan1tan2
cot2 = (1+tan1tan2)2/(tan1tan2)2 = (h2+k2+b2a2)2/4(a2k2b2h2+a2b2)2
Simplifying, the required locus is
(x2 + y2 + b2 a2)2 = 4cot2b(a2y2 b2x2 + a2b2)2

Example:
Find the equation to the hyperbola whose asymptotes are the straight lines x
+ 3y 1= 0 and 2x y + 7 = 0, and which passes through the point (1, 2).

Solution:
Equation to the asymptotes are given as
x + 3y 1 = 0

and

2x y + 7 = 0

(1)
(2)

(1) and (2) may be given by


(x + 3y 1)(2x y + 7) = 0

(3)

As the equation to the hyperbola will differ from (3) only by a constant, it
may be given by
(x + 3y 1)(2x y + 7) =

(4)

(where is a constant)
(1, 2) lies on the curve given by (4), we have
(1 + 6 1)(2 2 + 7) =
= 42
Hence the equation to the hyperbola will be
(x + 3y 1)(2x y + 7) = 42
2x2 xy + 6xy + 7x 3y2 + 21y 2x + y 7 = 42

2x2 3y2 + 5xy + 5x + 22y 49 = 0

3D Geometry
The moving power of mathematical invention is not reasoning but
imagination. A.DEMORGAN
We constantly describe both the shapes and positions of three dimensional objects. For
example, one simple way to describe a 3D object is to approximate its shape as a
mesh of triangles. Each triangle is de?ned by three vertices, and the positions of each of
these vertices will have to be described by three coordinates [x,y,z]t.
In this chapter, we shall study the direction cosines and direction ratios of a line
joining two points and also discuss about the equations of lines and planes in space
under different conditions, angle between two lines, a line and a plane, shortest
distance between two skew lines and distance of a point from a plane.
Lower level geometry of planes and graphical two dimensional coordinates moves into a
three dimensional space in three dimensional geometry.
Topics Covered:

Rectangular Co ordinate system in space


Direction cosines of a line
Direction Ratios
Parallel Lines
Projection of a Line
Theory of 3D Plane
Theory of 3D Straight Line
Shortest Distance between Two Non Intersecting Lines
Theory of Sphere
Solved Problems of 3D Geometry Part I
Solved Problems of 3D Geometry Part II
Solved Problems of 3D Geometry Part III
Solved Problems of 3D Geometry Part IV
Solved Problems of 3D Geometry Part V
Solved Problems of 3D Geometry Part VI

RECTANGULAR COORDINATE SYSTEM IN


SPACE
Let O be any point in space
and
be
three lines perpendicular to each
other. These lines are known as
coordinate axes and O is called
origin. The planes XY, YZ, ZX are
known as the coordinate planes.

Coordinates of a Point in
Space:

Consider a point P in space. The


position of the point P is given by
triad (x, y, z) where x, y, z are
perpendicular distance from YZplane, ZX-plane and XY-plane
respectively.

If We assume i, j, k unit vectors


along OX, OY, OZ respectively,
then position vector of point P is xi
+ yj + zk or simply (x, y, z).

x-axis = {( x, y, z) | y = z = 0}

y-axis = {(x, y, z) | x = z = 0}

z-axis = {(x, y, z) | x = y = 0}

xy plane = {(x, y, z) | z = 0}

yz plane = {(x, y, z) | x = 0}

zx plane = {(x, y, z) | y = 0}

OP = x2 + y2 + z2

Shifting the Origin:

Shifting the origin to another


point without changing the
directions of the axes is called
the translation of
axes.

Let the origin O be shifted to


another point
O' (x', y', z') without changing
the direction of axes. Let the
new coordinate frame be
O'X'Y'Z'. Let P (x, y, z) be a
point with respect to the
coordinate frame OXYZ.

Then, coordinate of point P


w.r.t. new coordinate frame
O'X'Y'Z' is (x1, y1, z1), where

x1 = x x', y1 = y y', z1 = z
z'

Example -1:
If the origin is shifted (1, 2, 3) without changing the directions of
the axes then find the new coordinates of the point (0, 4, 5) with
respect to new frame.

Solution:

x' = x x1,

where (x1, y1, z1) is the shifted origin

y' = y y1

z' = z z1

x' = 0 1 = 1

y' = 4 2 = 2

z' = 5 + 3 = 8

The coordinates of the point w.r.t. to new coordinate frame is (-1, 2, 8).

Note:

Distance between the points P(x1, y1, z1) and Q (x2, y2, z2) is

(x1 x2)2 + (y1 y2)2 + (z1 z2)2

The point dividing the line joining P(x1, y1, z1) and Q(x2, y2, z2) in m : n ratio is

(mx2 nx1 / m + n, my2 ny1 / m + n, mz2 nz1 / m + n) where m + n 0 .,


The coordinates of centroid of a triangle having vertices A (x1, y1, z1), B (x2, y2, z2) and
C (x3, y3, z3) is G (x1 + x2 + x3 / 3, y1 + y2 + y3 / 3, z1 + z2 + z3 / 3).

Example -2:
Find the coordinates of the point which divides the line joining
points (2, 3, 4) and (3, 4, 7) in ratio 3 : 5.

Solution:

Let the coordinates of the required point be (x, y, z), then

x = 2(3) + 3(5) / 3 + 5 = 21/8

y = 3(3) 4(5) / 3 + 5 = 11/8

z = 4(3) + 7(5) / 3 + 5 = 47/8

Hence the required point is (21/8, 11/8, 47/8).

Example -3:
are collinear.

Prove that the three points A (3, 2, 4), B (1, 1, 1) and C (1, 4, 2)

Solution:
The general coordinates of a point R which divides the line joining A (3, 2, 4)
and B (1, 1, 1) in the ratio : 1 are ( + 3 / + 1, 2 / + 1, + 4 / + 1)
(1)

If C (1, 4, 2) lies on the line AB, then for some value of m the coordinates
of the point R will be the same as those of C.

Let x-coordinate of point R = x-coordinate of point C.

Then, + 3 / + 1 = 1 => = 2

Putting = 2 in (1) the coordinates of R are (1, 4, 2) which are also the
coordinates of C.

Hence the points A, B, C are collinear.

DIRECTION COSINES OF A
LINE
If , , be the angles which a given directed line makes
with the positive directions of the co-ordinate axes, then
cos, cos, cos are called the direction cosines of the
given line and are generally denoted by l, m, n respectively.
Thus, l = cos, m = cos and n = cos
By the definition it follows that the direction cosine of the
axis of x are respectively cos0, cos90, cos 90i.e. (1, 0,
0).
Similarly direction cosines of the axes of y and z are
respectively (0, 1, 0) and (0, 0, 1).

Relation between the Direction Cosines:

Let OP be any line through the origin O which has


direction cosines l, m, n.
Let P (x, y, z) and OP = r
Then OP2 = x2 + y2 + z2 = r2
(1)

From P draw PA, PB, PC perpendicular on the


coordinate axes, so that
OA = x, OB = y, OC = z.
Also, POA = , POB = and POC = .
From triangle AOP, l = cos = x/r => x = lr

Dig: Relation between direction


cosines

Similarly y = mr and z = nr
Hence from (1)

r2(l2 + m2 + n2) = x2 + y2 + z2 = r2 => l2 + m2 + n2 = 1


Note:
If the coordinates of any point P be (x, y, z) and l, m, n be the direction cosines of the line
OP, O being the origin, then (lr, mr, nr) will give us the co-ordinates of a point on the line OP
which is at a distance r from (0, 0, 0).

Direction Ratios:
If a, b, c are three numbers proportional to the direction cosine l, m, n of a straight
line, then a, b, c are called its direction ratios. They are also called direction
numbers or direction components.
Hence by definition, we have
1/a = m/b = n/c = k (say)
=> l = ak, m = bk, n = ck => k2(a2 + b2 + c2) = l2 + m2 + n2 = 1
=> k = 1 / a2 + b2 + c2 = 1/a2
l = a/a2. Similarly m = b/a2 and n = n/a2
where the same sign either positive or negative is to be chosen throughout.

Example: If 2, 3, 6 be the direction ratios, then the actual direction cosines


are 2/7, 3/7, 6/7.

Note:

Direction cosines of a line are unique but direction ratios of a line in no way
unique but can be infinite.

Parallel Lines:
Since parallel lines have the same direction, it follows that the direction cosines of two or more parallel
straight lines are the same. So in case of lines, which do not pass through the origin, we can draw a
parallel line passing through the origin and direction cosines of that line can be found.
CXmhaU -4:
Find the direction cosines of two lines which are connected by the relations l5m
+ 3n = 0 and 7l2 + 5m2 3n2 = 0.
hb :

The given relations are

l5m + 3n = 0 => l = 5m 3n

and 7l2 + 5m2 3n2 = 0

(1)

(2)

Putting the value of l from (1) in (2), we get

7(5m 3n)2 + 5m2 3n2 = 0

or, 180m2 210mn + 60n2 = 0 or, (2m n)(3m 2n) = 0

m/n = 1/2 or 2/3

when m/n = 1/2 i.e. n = 2m

l = 5m 3n = m or 1/m = 1

thus and = 1 giving

or, 1/1 = m/1 = n/2 = (l2 + m2 + n2) / {(1)2 + l2 + 22} = 1/v6

So, direction cosines of one line are 1/v6, 1/v6, 2/v6

Again when m/n = 2/3 or n = 3m/2

l = 5m 3.3m/2 = m/2 or 1/m = 1/2

Thus, m/n = 2/3 and i/m = 1/2 giving i/1 = m/2 = n/3 = 1/v1 2 + 22 + 32 = 1/v14

The direction cosines of the other line are 1/v14, 2/v14, 3/v14.

Direction Cosine of a Line joining two given Points:

The direction ratios of line PQ joining P (x1, y1, z1) and Q(x2, y2,
z2) are x2 x1 = a(say), y2 y1 = b (say) and z2 z1 = c (say).

Then direction cosines are


l = (x2 x1) / (x2 x1)2, m = (y2 y1) / (x2 x1)2, n = (z2
z1) / (x2 x1)2

Alt txt: direction cosine of


a line

CXmhaU -5:
Find the direction ratios and direction cosines of the line joining the points
A(6, 7, 1) and B(2, 3, 1).

hb :

Direction ratios of AB are (4, 4, 2) = (2, 2, 1)

a 2 + b 2 + c2 = 9

Direction cosines are ( 2/3, 2/3, 1/3).

Angle between two Lines:

Let be the angle between two straight lines AB and AC whose direction
cosines are given whose direction cosines are l1, m1, n1 and l2, m2,
n2 respectively, is given by cosq = l1l2 + m1m2 + n1n2

If direction ratios of two lines are a1, b1, c1 and a2, b2, c2 are given, then angle
between two lines is given by

cos =

alt txt: angle between two lines

Particular Results:

We have, sin2 = 1 cos2


= (i12 + m12 + n12)(i22 + m22 + n22) (l1l2 + m1m2 + n1n2)2

= (l1m2 l2m1)2 + (m1n2 m2n1)2 + (n1l2 n2l1)2

=> sin = (l1m2 l2m1)2 .

Condition of perpendicularity:

If the given lines are perpendicular, then = 900 i.e. cos = 0

=> l1l2 + m1m2 + n1n2 = 0 or a1a2 + b1b2 + c1c2 = 0 .

Condition of parallelism:

If the given lines are parallel, then q = 00 i.e. sin = 0

(l1m2 l2m1)2 + (m1n2 m2n1)2 + (n1l2 n2l1)2

which is true, only when

l1m2 l2m1 = 0, m1n2 m2n1 = 0 and n1l2 n2l1 = 0 => I1/l2 = m1/m2 = n1/n2

Similarly, a1/a2 = b1/b2 = c1/c2.

CXmhaU -6:

hb :

Show that two lines having direction ratios 1, 3, 2 and 2, 2, 2 are perpendicular.

a1a2 + b1b2 + c1c2 = (1)(2) + (3)(2) + (2)(2) = 2 + 6 4 = 0

lines are perpendicular

Projection of a
Line:
Projection of the line joining two point P (x1,
y1, z1) and Q (x2, y2, z2) on another line
whose direction cosines are l, m, n is
AB = l(x2 x1) + m(y2 y1) + n(z2 z1)

Perpendicular Distance of a Point from a Line:

Let AB is straight line passing through point A (a, b, c) and


having direction cosines l, m, n.
AN = projection of line AP on straight line AB
= l(x a) + m(y b) + n(z c)
and AP = (xa)2 + (yb)2 + (zc)2
perpendicular distance of point P
PN = AP2 AN2

CXmhaU -7:
Find out perpendicular distance of point P (0, 1, 3) from straight line passing
through A (1, 3, 2) and having direction ratios 1, 2, 2.

hb :

Direction cosines of the line is 1/12 + 22 + 22, 2/12 + 22 + 22, 2/12 + 22 + 22

i.e. .1/3, 2/3, 2/3

PN = l(x a) + m(y b) + n(z c)

1/3 = (0 1) + 2/3 (1 + 3) + 2/3(3 2) = 5/3

AP = (01)2 + (1 + 3)2 + (32)2 = 6

Perpendicular distance PN = AP2 PN2 = 625/9 = 29/3.

Area of a Triangle

So, area of ABC is given by the relation 2 = x2 + y2 + z2

THE PLANE
Definition: Consider the locus of a point P(x, y, z). If x, y, z are allowed to vary without any restriction
for their different combinations, we have a set of points like P. The surface on which these points lie, is
called the locus of P. It may be a plane or any curved surface. If Q be any other point on its locus and all
points of the straight line PQ lie on it, it is a plane. In other words if the straight line PQ, however small
and in whatever direction it may be, lies completely on the locus, it is a plane, otherwise any curved
surface.
Equation of Plane in Different Forms:

General equation of a plane is ax + by + cz + d = 0

Equation of the plane in Normal form is lx + my + nz = p where p is the length of the normal from
the origin to the plane and (l, m, n) be the direction cosines of the normal.

The equation to the plane passing through P(x1, y1, z1) and having direction ratios
(a, b, c) for its normal is a(x x1) + b(y y1) + c (z z1) = 0

The equation of the plane passing through three non-collinear points (x1, y1, z1),

(x2, y2, z2) and (x3, y3 , z3) is

=0

The equation of the plane whose intercepts are a, b, c on the x, y, z axes respectively is x/a + y/b
+ z/c = 1 (a b c 0)

Equation of YZ plane is x = 0,

equation of plane parallel to YZ plane is x = d.

Equation of ZX plane is y = 0,

equation of plane parallel to ZX plane is y = d.

Equation of XY plane is z = 0,

equation of plane parallel to XY plane is z = d.

Four points namely A (x1, y1, z1), B (x2, y2, z2), C (x3, y3, z3) and D (x4, y4, z4) will be coplanar if one
point lies on the plane passing through other three points.

CXmhaU -8: Find the equation to the plane passing through the point (2, -1, 3) which is the foot of the
perpendicular drawn from the origin to the plane.

hb :

The direction ratios of the normal to the plane are 2, -1, 3.

The equation of required plane is 2(x 2) 1 (y + 1) + 3 (z 3) = 0

=> 2x y + 3z 14 = 0

Angle between the Planes

Angle between the planes is defined as angle between normals of the planes drawn from any point to the
planes.

Angle between the planes a1x + b1y + c1z + d1 = 0 and a2x + b2y + c2z + d2 = 0

is

Note:

If a1a2 +b1b2 +c1c2 = 0, then the planes are perpendicular to each other.

If a1/a2 = b1/b2 = c1/c2 then the planes are parallel to each other.

CXmhaU -9: Find angle between the planes 2x y + z = 11 and x + y + 2z = 3.

hb :

CXmhaU -10:
axis.

hb :

Find the equation of the plane passing through (2, 3, 4), (1, 1, 3) and parallel to x-

The equation of the plane passing through (2, 3, 4) is

a(x 2) + b(y 3) + c(z + 4) = 0

(1)

since (1, 1, 3) lie on it, we have

a + 4b 7c = 0

(2)

since required plane is parallel to x-axis i.e. perpendicular to YZ plane i.e.

1.a + 0.b + 0.c = 0 a = 0 4b 7c = 0 => b/7 = c/4

Equation of required plane is 7y + 4z = 5.

Perpendicular Distance:
The length of the perpendicular from the point P(x1, y1, z1) to the plane ax + by + cz + d = 0 is |ax1 + by1 +
cz1 + d / a2 + b2 + c2|.

Family of Planes:
Equation of plane passing through the line of intersection of two planes u = 0 and v = 0 is
u + v = 0.

Intersection of a Line and Plane:


If equation of a plane is ax + by + cz + d = 0, then direction cosines of normal to this plane are a, b, c. So
angle between normal to the plane and a straight line having direction cosines l, m ,n is given by
cos = al + bm + cn / a2 + b2 + c2.
Then angle between the plane and the straight line is /2 .

Plane and straight line will be parallel if al + bm + cn = 0

Plane and straight line will be perpendicular if a/l = b/m = c/n.

CXmhaU -11:
Find the equation of plane passing through the intersection of planes 2x 4y + 3z +
5= 0, x + y + z = 6 and parallel to straight line having direction cosines (1, 1, 1).
hb :

Equation of required plane be

(2x 4y + 3z + 5) + (x + y z 6) = 0

i.e. (2 + )x + (4 + )y + z(3 ) + (5 6)

=0

This plane is parallel to a straight line. So, al + bm + cn = 0

1(2 + ) + (1)(4 + ) + (1)(3 ) = 0 i.e. = 3

Equation of required plane is x 7y + 6z + 23 = 0

i.e. x + 7y 6z 23 = 0.

Bisector Planes of Angle between two Planes:


The equation of the planes bisecting the angles between two given planes a 1x +b1y +c1z +d1 = 0 and a2x +
b2y + c2z +d2 = 0 is

THE STRAIGHT LINE


Straight line in three dimensional geometry is defined as intersection of two planes. So general equation
of straight line is stated as the equations of both plane together i.e. general equation of straight line is a 1x
+ b1y + c1z + d1 = 0, a2x + b2y + c2z + d2 = 0
(1)

So, equation (1) represents straight line which is obtained by intersection of two planes.

Equation of Straight Line in Different Forms:


Symmetrical Form:

Equation of straight line passing through point P (x1, y1, z1) and whose direction cosines are l, m, n
is xx1 = y y1/m = z z1 / n

Equation of straight line passing through two points P (x1, y1, z1) and Q (x2, y2, z2) isxx1 / x2 x1 =
yy1 / y2 y1 = z y1 / z2 z1

Note:

The general coordinates of a point on a line is given by (x 1 + lr, y1 + mr, z1 + nr) where r is
distance between point (x1, y1, z1) and point whose coordinates is to be written.
CXmhaU 10. Find the equations of the straight lines through the point (a, b, c) which are
(a) parallel to z-axis
(b) perpendicular to z-axis
10. (i) Equation of straight lines parallel to z-axis have = 90 0, = 900, = 00

=> l = 0, m = 0, n = 1

Therefore equation of straight line is parallel to z-axis and passing through (a, b, c) is x a / 0 = y
b/0=zc/1

(ii)

equation of straight lines perpendicular to z-axis

let they make , angle with x and y axes respectively.

Then equation of straight lines perpendicular to z axis and passing through (a, b, c) is xa / cos
= y b / sin = z c / 0

=> x a / l = y b / m = z c / 0.

CXmhaU -12:
Find the coordinates of the point where the line joining the points (2, 3, 1) and (3,
4, 5) cuts the plane 2x + y + z = 7.

hb :

The direction ratios of the line are 3 2, 4 (3), 5 1 i.e. 1, 1, 6

Hence equation of the line joining the given points is

x2 / 1 = y + 3 / 1 = z 1 / 6 = r (say)

Coordinates of any point on this line are (r + 2, r 3, 6r + 1)

If this point lies on the given plane 2x + y + z = 7, then

2(r + 2) + (r 3) + (6r + 1) = 7 => r = 1

Coordinates of the point are (1 + 2, (1) 3, 6(1) + 1) i.e. (1, 2, 7).

Note:

If equation of straight line is given in general form, it can be changed into symmetrical form. The
method is described in following Example.

CXmhaU -13:

Find in symmetrical form the equations of the line

3x + 2y z 4 = 0= 4x + y 2z + 3.

hb :

The equation of the line in general form are

3x + 2y z 4 = 0, 4x + y 2z + 3 = 0

(1)

Let l, m, n be the direction cosines of the line. Since the line is common to both the planes,
it is perpendicular to the normals to both the planes.

Hence 3l + 2m n = 0, 4l + m 2n = 0

Solving these we get,

1/4+1 = m4+6 = n/38 i.e. 1/3 = m/2 = n/5 = 1/(3)2 + 22 + (5)2 = 1/38

So, direction cosines of the line are 3/38, 2/38, 5/38

Now to find the coordinates of a point on a line. Let us find out the point where it meets the
plane z = 0. Putting z = 0 in the equation given by (1), we have

3x + 2y 4 = 0, 4x + y + 3 = 0

solving these, we get x = 2, y = 5

So, one point of the line is (2, 5, 0)

equation of the line in symmetrical form is

i.e. .

Shortest Distance between two non


Intersecting Line:
Two lines are called non intersecting lines if they do not lie in the same plane. The straight line which is
perpendicular to each of non-intersecting lines is called the line of shortest distance. And length of
shortest distance line intercepted between two lines is called length of shortest distance.

Method: Let the equation of two non-intersecting lines be

xx1 / l1 = yy1/m1 = zz1/n1 = r1 (say)

(1)

And xx2 / l2 = yy2/m2 = zz2/n2 = r2 (say)

(2)

Any point on line (1) is P (x1 + l1r1, y1 + m1r1, z1 + n1r1) and on line (2) is

Q (x2 + l2r2, y2 + m2r2, z2 + n2r2).

Let PQ be the line of shortest distance. Its direction ratios will be

[(l1r1 + x1 x2 l2r2), (m1r1 + y1 y2 m2r2), (n1r1 + z1 z2 n2r2)]

This line is perpendicular to both given line. By using condition of perpendicularity we obtain 2 equations
in r1 and r2.

So by solving these, values of r1 and r2 can be found. And subsequently point P and Q can be found. The
distance PQ is shortest distance.

The shortest distance can be found by PQ =

Note:

If any straight line is given in general form then it can be transformed into symmetrical form and
we can further proceed.

CXmhaU -14:
Find the shortest distance between the lines , x3/3 = y 8/1 = z3/1, x + 3/3
= y+7/2 = z6/4 . Also find the equation of line of shortest distance.

hb :

Given lines are x3/3 = y8/1 = z3/1 = r1 (say)

= x+3 / 3 = y + 7 / 2 = z 6 / 4 = r2 (say)

(1)

(2)

Any point on line (1) is P (3r1 + 3, 8 r1, r1 + 3) and on line (2) is

Q (3 3r2, 2r2 7, 4r2 + 6).

If PQ is line of shortest distance, then direction ratios of PQ

= (3r1 + 3) (3 3r2), (8 r1) (2r2 7), (r1+ 3) (4r2 + 6)

i.e. 3r1 + 3r2 + 6, r1 3r2 + 15, r1 4r2 3

As PQ is perpendicular to liens (1) and (2)

3(3r1 + 3r2 + 6) 1(r1 2r2 + 15) + 1(r1 4r2 + 3) = 0

=> 11r1 + 7r2 = 0

(3)

and 3(3r1 + 3r2 + 6) + 2(r1 2r2 + 15) + 4(r1 4r2 + 3) = 0

i.e. 7r1 + 11r2 = 0

(4)

On solving equations (3) and (4), we get r1 = r2= 0.

So, point P (3, 8,3) and Q (3, 7, 6)

Length of shortest distance PQ = {(33)2 + (78)2 + (63)2} = 330

Direction ratios of shortest distance line is 2, 5, 1

Equation of shortest distance line x3/2 = y8/5 = z3/1.

THE SPHERE

A sphere is a locus of a point which moves in space such that its distance from a fixed point is constant.
Fixed point is called centre of sphere and constant distance is called radius of sphere.

Equation of Sphere in Different Forms:

If centre of sphere is (a, b, c) and radius is r, then equation of sphere is


(x a)2 + (y b)2 + (z c)2 = r2.

If centre of sphere is origin and radius is r, then x2 + y2 + z2 = r2.

General form: The general equation of a sphere is x2 + y2 + z2 + 2ux + 2vy + 2wz + d = 0


Centre of sphere = (u, v, w), radius = 2 + v2 + w2 d.

Diameter form: Equation of a sphere whose extremities of diameter are A (x 1, y1, z1) and B (x2,
y2, z2) is (x x1) (x x2) + (y y1) (y y2) + (z z1) (z z2) = 0.
CXmhaU 13. Find the equation of the sphere which passes through the points (1, 3, 4), (1, 5, 2) and (1,
3, 0) and whose centre is on the plane x + y + z = 0.

hb :

Let equation of the sphere be


x2 + y2 + z2 + 2ux + 2vy + 2wz + d = 0
its centre is (- u, - v, - w) which is on x + y + z = 0
=> u + v + w = 0

(1)

it passes through (1, - 3, 4) => 2u - 6v + 8w + d = - 26

(2)

(1, - 5, 2) => 2 u - 10 v + 4 w + d = - 30

(3)

and it passes through (1, - 3, 0) => 2 u - 6 v + d = - 10

(4)

solving these four equations we get,


u = - 1, v = 3, w = - 2 and d = 10
Therefore required equation of the sphere is
x2 + y2 + z2 - 2 x + 6 y - 4 z + 10 = 0.

CXmhaU -15:
Find the equation of the sphere whose centre is (2, 3, 4) and which passes through
the point (1, 2, 1).

hb :

Radius of sphere = {(21)2 + (32)2 + (4+1)2} = 51


Equation of the sphere is (x 2)2 + (y + 3)2 + (z 4)2 = (51)2
i.e. x2 + y2 + z2 4x + 6y 8z 22 = 0.

SOLVED PROBLEMS
OBJECTIVE
1.
The angle between two lines whose direction cosines are given by the equation l + m + n =
0, l2 + m2 + n2 = 0 is

hb :

(A) /3

(B) 2/3

(C) /4

(D) None of these

Eliminating n between the two relations, we have

l2 + m2 (l + m)2 = 0 or 2lm = 0 => either l = 0 or m = 0

if l = 0, then m + n = 0 i,e. m = n

=> l/0 = m/1 = n/1, giving the direction ratios of one line.

If m = 0, then l + n = 0 i.e. l = n

=> l/0 = m/1 = n/1, giving direction ratios of the other lines.

The angles between these lines is

2.
The equation of the plane which contains the line of intersection of the planes x + y + z 6
= 0 and 2x + 3y + z + 5 = 0 and perpendicular to the xy plane is:

hb :

(A) x 2y + 11 = 0

(B) x + 2y + 11 = 0

(C) x + 2y 11 = 0

(D) x 2y 11 = 0

Equation of the required plane is (x + y + z 6) + (2x + 3y + z + 5) = 0

i.e. (1 + 2)x + (1 + 3)y + (1 + )z + (6 + 5) = 0

This plane is perpendicular to xy plane whose equation is z = 0

i.e. 0 . x + 0 . y + z = 0

By condition of perpendicularity

0.(1 + 2) + 0. (1 + 3) + (1 + ) .1 = 0 i.e. = 1

Equation of required plane is

(1 2)x + (1 3)y + (1 1)z + (6 5) = 0 or x + 2y + 11 = 0.

3.
The coordinates of the foot of the perpendicular drawn from the origin to the plane 3x + 4y
6z + 1 = 0 are :

(A) (3/61, 4/61, 6/61)

(C) (3/61, 4/61, 6/61)

hb :

(B) (3/61, 4/61, 6/61)

(D) (3/61, 4/61, 6/61)

The equation of the plane is 3x + 4y 6z + 1 = 0

(1)

The direction ratios of the normal to the plane (1) are 3, 4, 6. So equation of the line
through (0, 0, 0) and perpendicular to the plane (1) are

x/3 = y/4 = z/6 = r (say)

(2)

The coordinates of any point P on (2) are (3r, 4r, 6r). If this point lie on the plane (1), then

3(3r) + 4(4r) 6(6r) + 1 = 0 i.e. r = 1/61

Putting the value of r coordinates of the foot of the perpendicular P are (3/61, 4/61,
6/61).

4.
The distance of the point (1, 2, 3) from the plane x y + z = 5 measured parallel to the
line x/2 = y/3 = z/6 is:

(A) 7 unit

(B) 4 unit

(C) 1 unit

(D) 2 unit

hb :
Here we are not to find perpendicular distance of the point from the plane but distance
measured along with the given line. The method is as follow:

The equation of the line through the point (1, 2, 3) and parallel to given line is

x1/2 = y+2/ 3 = z3/6 = r (say)

The coordinate of any point on it is (2r + 1, 3r 2, 6r + 3).

If this point lies in the given plane then

2r + 1 (3r 2) + (6r + 3) = 5 7r = 1 or r = 1/7

point of intersection is (9/7, 11/7, 15/7)

The required distance

= the distance between the points (1, 2, 3) and (9/7, 11/7, 15/7)

= (19/7)2 + (2+11/7)2 + (315/7)2 = 1/7 = 49 = 1 unit.

5.

The image of the point (1, 3, 4) in the plane 2x y + z + 3 = 0 is :

(A) (3, 5, 2)

(B) (3, 2, 5)

(C) (5, 3, 2)

(D) (2, 5, 3)

hb :
As it is clear from the figure that PQ will
be perpendicular to the plane and foot of this
perpendicular is mid point of PQ i.e. N.
So, direction ratios of line PQ is 2, 1,
1
=> Equation of line PQ = x1/2 = y3/
1 = z4/1 r (say)
Any point on line PQ is (2r + 1, r + 3, r
+ 4)
If this point lies on the plane, then
2(2r + 1) (r + 3) + (r + 4) + 3 =
0 => r = 1
coordinate of foot of perpendicular N
= (1, 4, 3)
As N is middle point of PQ

1 = 1+x1/2, 4 = 3+y1/2, 3 = 4+z1/2

=> x1 = 3, y1 = 5, z1 = 2

Image of point P (1, 3, 4) is the point Q (3, 5, 2).

6.
The equation of the sphere which passes through the points (1, 0, 0), (0, 1, 0) and (0, 0, 1)
and has its radius as small as possible is :

(A) 3(x2 + y2 + z2) + 2(x + y + z) 1= 0

(B) 3(x2 + y2 + z2) 2(x + y + z) 1= 0

(C) (x2 + y2 + z2) 2(x + y + z) 1= 0

(D) (x2 + y2 + z2) + 2(x + y + z) 1= 0

hb :

Let equation of sphere be given by

x2 + y2 + z2 + 2ux + 2vy + 2wz + d = 0

(1)

As sphere passes through points (1, 0, 0), (0, 1, 0) and (0, 0, 1). So we have

1 + 2u + d = 0, 1 + 2v + d = 0, 1 + 2w + d = 0

On solving u = v = w = 1/2 (d + 1)

If r is the radius of the sphere, then

r = u2 + v2 + w2 d

r2 = 3/4 (d + 1)2 d = m (say)

for r to be minimum

d/dd = 0 => 3/4.2(d + 1) 1 = 0 or d = 1/3

Also, d2/dd2 = 3/2 positive at d = 1/3

Hence m is minimum at d = 1/3

So, substituting value of d we have u = v = w = 1/3

equation of the sphere

x2 + y2 + z2 (x + y + z) 1/3 = 0 => 3(x2 + y2 + z2) 2(x + y + z) 1= 0.

7.
a:

A point moves so that the ratio of its distances from two fixed points is constant. Its locus is

(A) plane

(B) st. lines

(C) circle

(D) sphere

hb :
Let the coordinates of moving point P be (x, y, z). Let A (a, 0, 0) and B (a, 0, 0) be two
fixed points. According to given condition

AP/BP = constant = k (say) => AP2 = k2BP2

or, (x a)2 + (y 0)2 + (z 0)2 = k2{(x + a)2 + (y 0)2 + (z 0)2}

=> (1 k2)(x2 + y2 + z2) 2ax(1 + k2) + a2(1 k2) = 0

required locus is x2 + y2 + z2 2a(1+k2)/(1k2) + a2 = 0. Which is a sphere.

8.

hb :

The ratio in which yzplane divides the line joining (2, 4, 5) and (3, 5, 7)

(A) -2 : 3

(B) 2 : 3

(C) 3 : 2

(D) -3 : 2

Let the ratio be : 1

x-coordinate is 3+2 / +1 = 0 => = 2/3 .

Hence (A) is the correct answer.

9.
A line makes angles , , , with the four diagonals of a cube then cos2 +
2
2
cos + cos + cos2 =

(A) 1

(B) 4/3

(C) 3/4

hb
(1, 1, 1)

(D) 4/5

The direction ratios of the diagonal

Direction cosine are (1/3, 1/3, 1/3)


Similarly direction cosine of AS are (1/3,
1/3, 1/3)
BP are (1/3, 1/3, 1/3)

CQ are (1/3, 1/3, 1/3)

Let l, m, n be direction cosines of the line

cos = l+m+n/3, cos = lmn/3, cos = l+mn/3, cos = lm+n/3

cos2a + cos2 + cos2 + cos2 = 4(l2 + m2 + n2) / 3 = 4/3 ( since l2 + m2+ n2 = 1)

Hence (B) is the correct answer.

10.

The points (0, -1, -1), (-4, 4, 4), (4, 5, 1) and (3, 9, 4) are

(A) collinear

(C) forming a square

(B) coplanar

(D) none of these

hb :

Equation of the plane passing through the points (0, -1, -1), (-4, 4, 4) and (4, 5, 1)

is

=0

. (1)

The point (3, 9, 4) satisfies the equation (1).

Hence (B) is the correct answer.

11.
A variable plane passes through a fixed point (a, b, c) and meets the coordinate axes in A,
B, C. The locus of the point common to plane through A, B, C parallel to coordinate planes is

hb :

(A) ayz + bzx + cxy = xyz

(B) axy + byz + czx = xyz

(C) axy + byz + czx = abc

(D) bcx + acy + abz = abc

Let the equation to the plane be x/ + y/ + z/ = 1

=> x/ + y/ + z/ = 1 (the plane passes through a, b, c)

Now the points of intersection of the plane with the coordinate axes are A(, 0, 0), B(0, ,
0) & C(0, 0, )

=> Equation to planes parallel to the coordinate planes and passing through A, B & C are x
= , y = and z = .

The locus of the common point is

a/x + b/y + c/z = 1 (by eliminating , , from above equation)

Hence (A) is the correct answer.

12.

Consider the following statements:

Assertion (A): the plane y + z + 1 = 0 is parallel to x-axis.

Reason (R): normal to the plane is parallel to x-axis.

Of these statements:

(A) both A and R are true and R is the correct explanation of A

(B) both A and R are true and R is not a correct explanation of A

(C) A is true but R is false

(D) A is false but R is true

hb :

Given plane y + z + 1 = 0 is parallel to x-axis as 0.1 + 1.0 + 1.0 = 0

but normal to the plane will be perpendicular to x-axis.

Hence (C) is the correct answer.

13.
The equation of the plane containing the line x = /l, y = /m and z =/n is a(x ) +
b(y ) + c(z ) = 0, where al + bm + cn is equal to

(A) 1

(B) 1

(C) 2

(D) 0

hb :
Since straight line lies in the plane so it will be perpendicular to the normal at the given
plane. Since direction cosines of straight line are l, m, n and direction ratios of normal to the plane are a,
b, c. So, al + bm + cn = 0.

Hence (D) is the correct answer.

14.
The shortest distance between the two straight lines x4/3 / 2 = y+6/5 / 3 = z3/2 /
4 and 5y+6/8 = 2z3/9 = 3x4/5 is

hb :

(A) 29

(B) 3

(C) 0

(D) 610

Since these two lines are intersecting so shortest distance between the lines will be 0.

Hence (C) is the correct answer.

15.
A straight line passes through the point (2, 1, 1). It is parallel to the plane 4x + y +
z + 2 = 0 and is perpendicular to the line x/1 = y/2 = z5/1. The equations of the straight line are

hb :

(A) x2/4 = y+1/1 = z+1/1

(B) x+2/4 = y1/1 = z1/1

(C) x2/1 = y+1/1 = z+1/3

(D) x+2/1 = y1/1 = z1/3

Let direction cosines of straight line be l, m, n

4l + m + n = 0

l 2m + n = 0

=> l/3 = m/3 = n/9 => l/1 = m/+1 = n/3

Equation of straight line is x2/1 = y+1/1 = z+1/3.

Hence (C) is the correct choice.

16.
sphere is

If centre of a sphere is (1, 4, 3) and radius is 3 units, then the equation of the

(A) x2 + y2 + z2 2x 8y + 6z + 17 = 0

(B) 2(x2 + y2 + z2) 2x 8y + 6z + 17 = 0

(C) x2 + y2 + z2 4x + 16y + 12z + 17 = 0

(D) x2 + y2 + z2 + 2x + 8y 6z 17 = 0

hb :

Equation of sphere will be (x 1)2 + (y 4)2 + (z + 3)2 = 9

Hence (A) is the correct answer.

17.
If equation of a sphere is 2(x2 + y2 + z2) 4x 8y + 12z 7 = 0 and one extremity of
its diameter is (2, 1, 1), then the other extremity of diameter of the sphere will be

hb :

(A) (2, 9, 13)

(B) (0, 9, 7)

(C) (0, 5, 7)

(D) (2, 5, 13)

The centre of the sphere is (1, 2, 3) so if other extremity of diameter is (x 1, y1, z1), then

x1+2/2 = 1, y11/2 = 2, z1+1/2 = 3

Required point is (0, 5, 7).

Hence (C) is the correct answer.

18.
The direction cosines of the line which is perpendicular to the lines with direction cosines
proportional to (1, 2, 2), (0, 2, 1) is

(A) (2/3, 1/3, 2/3)

(B) (2/3, 1/3, 2/3)

(C) (2/3, 1/3, 2/3)

(D) (2/3, 1/3, 2/3)

hb :

Let direction ratios of the required line be

Therefore a - 2 b - 2 c = 0

And 2 b + c = 0

=> c = - 2 b

a - 2 b + 4b = 0 => a = - 2 b

Therefore direction ratios of the required line are <- 2b, b, - 2b> = <2, - 1, 2>

direction cosines of the required line

19.

hb :

The points (4, 7, 8), (2, 3, 4), (1, 2, 1) and (1, 2, 5) are :

(A) the vertices of a parallelogram

(B) collinear

(C) the vertices of a trapezium

(D) concyclic

Let A (4, 7, 8), B (2, 3, 4), C (- 1, - 2, 1), D (1, 2, 5)

Direction cosines of AB (2/6, 4/6, 4/6) = (1/3, 2/3, 2/3)

Direction cosines of CD (2/6, 4/6, 4/6)

= (1/3, 2/3, 2/3)

So, AB parallel to CD

Direction cosines of AD (3/43, 5/43, 3/43)

Direction cosines of BC (3/43, 5/43, 3/43)

= (3/43, 5/43, 3/43)

so, AD is parallel to BC.

Therefore ABCD is a parallelogram.

20.
The equation of the plane parallel to the plane 4x 3y + 2z + 1 = 0 and passing through the
point (5, 1, 6) is :

(A) 4x - 3y + 2z - 5 = 0

(B) 3x - 4y + 2z - 5 = 0

(C) 4x - 3y + 2z + 5 = 0

(D) 3x - 4y + 2z + 5 = 0

hb :

Equation of the plane parallel to the plane 4x - 3y + 2z + 1 = 0 is of the form of

4x 3y + 2z + k = 0 again it passes through (5, 1, - 6)

so, 20 - 3 - 12 + k = 0 k = - 5

Therefore required equation is 4x - 3y + 2z - 5 = 0.

21.
A plane is passed through the middle point of the segment A (2, 5, 1), B (6, 1, 5) and is
perpendicular to this line. Its equation is :

hb :

(A) 2x - y + z = 4

(B) 2x - y + z = 4

(C) x - 3y + z = 5

(D) x - 4y + 2z = 5

Mid-point of A (- 2, 5, 1) and B (6, 1, 5) is (2, 3, 3)

direction ratios of the line joining A and B is <2, - 1, 1>

Therefore equation of the line perpendicular to AB and passing through (2, 3, 3) is

2(x 2) 1(y 3) + 1(z 3) = 0 => 2x - y + z = 4

22.
A plane meets the co-ordinates axes in A, B, C such that the centroid of triangle ABC is (a, b, c).
The equation of the plane is :

hb :

(A) x/a + y/b + z/c = 3

(B) x/a + y/b + z/c = 1

(C) x/a + y/b + z/c = 2

(D) None of these

The plane meets the co-ordinate axes at A, B, C such that centroid of the triangle ABC is (a, b, c)

so, the plane cuts X-axis at (3 a, 0, 0)

So, X-intercept = 3 a

The plane cuts Y-axis at (0, 3 b, 0)

=> Y-intercept = 3 b

the plane cuts Z-axis at (0, 0, 3 c)

=> Z-intercept = 3 c

Therefore required equation is x/3a + y/3b + z/3c = 1

=> x/a + y/b + z/c = 3.

23.

The radius of the sphere (x + 1)(x + 3) + (y 2)(y 4) + (z + 1)(z + 3) = 0 is:

hb :

(A) 2

(B) 2

(C) 3

(D) 3

(x + 1) (x + 3) + (y - 2) (y - 4) + (z + 1)(z + 3) = 0 is the given equation of sphere.

So, end points of the diameter are

(- 1, 2, - 1) and (- 3, 4, - 3)

radius = (2+1)2 + (32)2 + (2+1)2 = 3

24.

The sum of the direction cosines of a straight line is

(A) zero

(C) constant

hb :

(B) one

(D) none of these

cos = l, cos = m, cos = n

sum of direction cosines cos + cos + cos

=l+m+n

which is constant.

25.
The angle between straight lines whose direction cosines are (1/2, 1/2, 1/2) and(1/3, 1/3,
1/3) is

(A) cos1 (2/3)

(B) cos1 (1/6)

(C) cos1 (1/6)

(D) none of these

hb :

26.
Which one of the following is best condition for the plane ax + by + cz + d = 0 to intersect the x
and y axes at equal angle

hb :

(A) |a| = |b|

(B) a = b

(C) a = b

(D) a2 + b2 = 1

The plane a x + b y + c z + d = 0 intersects x and y axes at equal angles

Therefore |cos | = |cos |

=> |l| = |m|

=> |a| = |b|.

27.

hb :

The equation of a straight line parallel to the x-axis is given by

(A) xa/1 = yb/1 = zc/1

(B) xa/0 = yb/1 = zc/1

(C) xa/0 = yb/0 = zc/1

(D) xa/1 = yb/0 = zc/0

Equation of straight line parallel to x-axis is

xa/1 = yb/0 = zc/0

because l = cos = 1, m = cos = cos /2 = 0, n = cos = 0.

28.

hb :

If P (2, 3, 6) and Q (3, 4, 5) are two points, the direction cosines of line PQ are

(A) 1/171, 7/171, 11/171

(B) 1/171, 7/171, 11/171

(C) 1/171, 7/171, 11/171

(D) 7/171, 1/171, 11/171

P (2, 3, - 6), Q (3, - 4, 5)

direction ratios = <1, - 7, 11>

direction cosines = (1/1+49+121, 7/1+49+121, 11/1+49+121)

= (1/171, 7/171, 11/171)

29.

hb :

The ratio in which yzplane divide the line joining the points A(3, 1, 5) and B(1, 4, 6) is

(A) 3 : 1

(B) 3 : 1

(C) 1 : 3

(D) 1 : 3

A (3, 1, - 5), B (1, 4, - 6)

Therefore 3+/+1 = 0 => = 3

Therefore required ratio is - 3 : 1

30.
A straight line is inclined to the axes of x and z at angels 450 and 600 respectively, then the
inclination of the line to the y-axis is

hb :

(A) 300

(B) 450

(C) 600

(D) 900

A straight line is inclined to the axes of x and z at an angle 45 0 and 600

l2 + m2 + n2 = 1

=> m2 = 1/4

=> m = 1/2

=> angle made by 600

31.

hb :

The angle between two diagonals of a cube is

(A) cos = 3/2

(B) cos = 1/2

(C) cos = 1/3

(D) none of these

cos = a2 + a2 + a2 / a2 + a2 + a2 a2 + a2 + a2 of side is a

= 1/3.

32.

Given that A (3, 2, 4), B (5, 4, 6) and C (9, 8, 10) are collinear. The ratio in which B divides AC

(A) 1 : 2

(B) 2 : 1

(C) 1 : 2

(D) 2 : 1

hb :

9 + 3/+1 = 5 => 9 - 5 = 2

=> = 1/2.

33.
If P1P2 is perpendicular to P2P3, then the value of k is, where P1(k, 1, 1), P2(2k, 0, 2) and P3(2 +
2k, k , 1)

hb :

(A) 3

(B) 3

(C) 2

(D) 2

Direction ratios of P1 P2 = - 1, 3>

direction ratios of P2 P3 = <2, k, - 1>

Therefore 2 k - k - 3 = 0

=> k = 3.

34.
A is the point (3, 7, 5) and B is the point (3, 2, 6). The projection of AB on the line which joins the
points (7, 9, 4) and (4, 5, 8) is

(A) 26

(B) 2

(C) 13

(D) 4

hb :

Distances of the line joining (7, 9, 4) and (4, 5, - 8) is < 3/13, 4/13, 12/13 >

Therefore required projection is 26/13 = 2 (B)

Exercise # 1

35.
The shortest distance of the point from the x-axis is equal to(Ref.P.K.Sharma_Three
Dimen._P.C6.1Q.1)

Ans.

(A) x12 + y12

(B) x12 + z12

(C) y12 + z12

(D) None of these

(C)

hb . Foot of perpendicular drawn from P to x-axis will have its coordinates as (x, 0, 0).

Required distance = y12 + z12

36.
The point of intersection of the xy plane and the line passing through the points and is:
(Ref.P.K.Sharma_Three Dimen._P.C6.1Q.4)

(A)

(13/5, 23/5, 0)

(B)

(13/5, 23/5, 0)

(C)

(13/5, 23/5, 0)

(D)

(13/5, 23/5, 0)

Ans.

(B)

hb . Direction ratios of AB are 2, -3, 5.

Thus equation of AB is, x3/2 = y4/3 = z1/5

For the point of intersection of this line with xyplane, we have

Z=0
=> x3/2 = y4/3 = 1/5
=> x=3 2/5 = 13/5, y = 4 + 3/5 = 23/5

Hence, the required point is (15/5, 23/5, 0)

37.
The projections of the line segment AB on the coordinate axes are 9, 12, -8 respectively. The
direction cosines of the line segment AB are:(Ref.P.K.Sharma_Three Dimen._P.C6.1Q.5)

Ans.

(A) 9/17, 12/17, 8/17

(B) 9/289, 12/289, 8/289

(C) 9/17, 12/17, 8/17

(D) None of these

(A)

Length of segment AB = 81 + 144 + 64 = 17

Thus direction cosines of AB are

.9/17, 12/17, 8/17

38.
The direction cosines of two mutually perpendicular lines are l 1,m1,n1 andl2,m2,n2. The direction
cosines of the line perpendicular to both the given lines will be:(Ref.P.K.Sharma_Three
Dimen._P.C6.1Q.6)

Ans.

(A)

l1+ l1, m1+m2, n1+n2

(B)

l2 l2, m1 m2, n1+n2

(C)

l1 l2, m1m2, n1n2

(D)

m1n2 m2n1, n1 l2 n2, l1m2 l2m1

(D)

Let the direction cosine of the required line be l, m and n.

We must have,

ll1 + mm1 + nn1 = 0, ll2 + mm2 + nn2 = 0

We have l1l2 + m1m2 + n1n2 = 0

Thus

(m1n2n1m2)2 = (l2)2 (l2)2 l1l2 = 1

=> l = m1n2 n1m2, m = n1 l2 n2 l1,

n = l1m2 l2m1

39.
A directed line segment angles ,, with the coordinate axes. The value ofcos2 is always
equal to:(Ref.P.K.Sharma_Three Dimen._P.C6.1Q.7)

Ans.

(A)

-1

(B)

(C)

-2

(D)

(A)

cos2 = (2cos21)

= 2l23

= 1

40.

Ans.

The locus represented by xy + yz = 0 is:(Ref.P.K.Sharma_Three Dimen._P.C6.2Q.9)

(A)

A pair of perpendicular lines

(C)

A pair of parallel planes

(B)

(D)

A pair of parallel lines

A pair of perpendicular planes

(D)

xy + yz = 0

=> x(y+z) = 0

=> x=0, y+z = 0

Thus it represents a pair of planes

X = 0, y + z = 0

that are clearly mutually perpendicular.

41.
The plane 2x 3y + 6z 11 = 0 makes an angle (a) with x-axis. The value of a is equal to:
(Ref.P.K.Sharma_Three Dimen._P.C6.2Q.10)

Ans.

(A)

3/2

(B)

2/3

(C)

2/7

(D)

3/7

(C)

If be the angle between the plane and xaxis, then

sin 2/4+9+36 = 2/7

=> = sin1 (2/7)


=> a = (2/7)

42.

Ans.

The planes x + y = 0, y + z = 0 and x + z = 0:(Ref.P.K.Sharma_Three Dimen._P.C6.2Q.12)

(A)

meet in a unique point

(B)

meet in a unique line

(C)

are mutually perpendicular

(D)

none of these

(B)

Clearly, given planes have a common line of intersection namely the z-axis.

43.
The equation of a plane passing through (1, 2, -3), (0, 0, 0) and perpendicular to the plane 3x 5y
+ 2z = 11, is:(Ref.P.K.Sharma_Three Dimen._P.C6.2Q.13)

Ans.

(A)

3x + y + 5/3 z = 0

(B)

4x + y + 2z = 0

(C)

3x y + z/3

(D)

x+y+z=0

(D)

Let the required plane be

Ax + by + cz = 0.

We have 3a 5b + 2c = 0, a + 2b 3c = 0

=> a/154 = b/2+9 = c/6+5

=> a : b : c = 11 : 11 : 11

Thus plan is

x+y+z=0

44.
The direction ration of a normal to the plane passing through (1, 0, 0), (0, 1, 0) and making an
angle /4 with the plane x + y = 3 are:(Ref.P.K.Sharma_Three Dimen._P.C6.2Q.15)

(A)

(1, 2, 1)

(B)

(1, 1,2)

(C)

Ans.

(1,1,2)

(D)

(2, 1, 1)

(B)

Let the plane be x/a + y/b + z/c = 1

=> 1/a = 1, 1/b = 1

.=> a = b = 1

Also,

=> c = 1/2

Thus direction rations are

(1,1,2) or

(1,1,2)

45.
The equation of a plane passing through the line of intersection of the planes x + y + z = 5, 2x y
+ 3z = 1 and parallel to the line y = z = 0 is:(Ref.P.K.Sharma_Three Dimen._P.C6.2Q.16)

(A)

3x z = 9

(B)

3y z = 9

(C)

x 3z = 9

(D)

y 3z = 9

Ans.

(B)

Plane will be in the form

(x + y + z 5) + a(2x y + 3z) 1) = 0 i.e., x(1 + 2a) + y(1 a) + z(1 + 3a) = 5 + a

It is parallel to the line y = z = 0.

Since, (1 + 2a) = 0

a = 1/2

Thus required plane is

3/2y 1/2 z = 9/2

i.e.,

3y z = 9

46.
The angle between lines whose direction cosines are given by l + m + n = 0, l2 + m2 n2 = 0, is:
(Ref.P.K.Sharma_Three Dimen._P.C6.2Q.17)

(A)

/2

(B)

/3

(C)

/6

(D)

None of these

Ans.

(D)

.l + m + n = 0, l2 + m2 n2 = 0

We also have

l2 + m2 + n2 = 1

=> 2n2 = 1

=> n = 1/2, 1/2

Also,

l2 + m2 = n2 = ((l+m))2

=> lm = 0 and l + m = 1/2

Hence, direction cosines are lines are

(1/2, 0, 1/2), (1/2, 0, 1/2)

(0, 1/2, 1/2), (0, 1/2, 1/2)

Angle between these lines in both cases is zero.

47.
Centroid of the tetrahedron OABC, where , , and O is the origin is (1, 2, 3). The value of is equal
to:(Ref.P.K.Sharma_Three Dimen._P.C6.3Q.19)

Ans.

(A)

75

(B)

80

(C)

121

(D)

None of these

(A)

We have

4 = a + 1 + 2 + 0,

=> a = 1,

8=2+b+1+0

=> b = 5,

12 = 3 + 2 + c + 0.

=> c = 7,

a2 + b2 + c2 = 1 + 25 + 49 = 75

48.
The equation of the plane passing through the points (2, -1, 0), (3, -4, 5) and parallel to the line 2x
= 3y = 4z is:(Ref.P.K.Sharma_Three Dimen._P.C6.3Q.20)

Ans.

(A)

125x 90y 79z = 340

(B)

32x 21y 36z = 85

(C)

73x + 61y 22z = 85

(D)

29x 27y 22z = 85

(D)

Give line is 2x = 3y = 4z

i.e.,

x/6 = y/4 = z/3

Let the plane be

Ax + by + cz = 1.

We have

6a + 4b + 3c = 0

2a b = 1

3a 4b + 5c = 1.

=> a = 29/85, b = 27/85, c = 22/85

Thus equation of plane is

29x 27y 22z = 85.

49.
A plane passes through the point . If the distance of this plane from the origin is maximum, then
its equation is:(Ref.P.K.Sharma_Three Dimen._P.C6.4Q.1)

Ans.

(A)

x + 2y 3z + 4 = 0

(C)

2y - x + 3z = 0

(B)

x + 2y + 3z = 0

(D)

(B)

Clearly in this case OA will be a normal to the plane.

Direction cosine of segment OA are

1/14, 2/14, 3/14

and .OA = 1 + 4 = 0 = 14

Thus the equation of plane is

x - 2y + 3z = 0

x + 2y + 3z = 14.

50.
The shortest distance of the plane 12 + 4y + 3z = 327, from the sphere , is equal to:
(Ref.P.K.Sharma_Three Dimen._P.C6.4Q.3)

Ans.

(A)

39 units

(B)

26 sq. units

(C)

13 units

(D)

None of these

(C)

Center and radius of given sphere are (2, 1, 3) and 13 unit respectively.

Now, distance of center of the sphere from the given plane

= |24 + 4 + 9 327| / 122 + 42 + 32 = 26 units

Shortest distance = (26 13) = 13 units.

51.
The lines x = ay + b, z = cy + d and x = ay + b, z = cy + d will be mutually perpendicular
provided:(Ref.P.K.Sharma_Three Dimen._P.C6.4Q.5)

(A)

(a + a)(b + b)(c + c)

(B)

aa + cc + 1 = 0

(C)

aa + bb + cc + 1 = 0

(D)

(a + a) (b + b) (c + c) + 1 = 0

Ans.

(B)

Give lines are

xb/a = y = z d/c and xb' / a' = y = zd'/c'

These lines will be mutually perpendicular, provided

a.a' + 1.1' + c.c' = 0

.=> a.a' + c.c' + 1= 0

52.
The straight lines x2/1 = y3/1 = z4/k and x1/k = y4/2 = z5/1, will intersect provided:
(Ref.P.K.Sharma_Three Dimen._P.C6.4Q.6)

Ans.

(A)

k = {3, -3}

(B)

k = {0, -1}

(C)

k = {-1, 1}

(D)

k = {0, -3}

(D)

Any point on the first line can be takes as

P1 = (r1 + 2, r1 + 3, kr1+4)

These lines will intersect if for some r1 and r2 we have

r1 + 2 = kr2 + 1,

r1 + 3 = 2r2 + 1,

kr1 + 4 = r2 + 5,

r1 + kr2 + 1 = 0, r1 = 2r2 + 1,

=> r2 = 2/k2, r1 = k+2 / k2

putting these values in the last condition, we get

k2 + 3k = 0

.=> k = {3,0}

53.
The plane ax + by + cz = d, meets the coordinate axes at the points, A, B and C respectively. Area
of triangle ABC is equal to:(Ref.P.K.Sharma_Three Dimen._P.C6.4Q.7)

(A)

d2a2 + b2 + c2 / |abc|

(B)

d2a2 + b2 + c2 / 2|abc|

(C)

d2a2 + b2 + c2 / 4 |abc|

(D)

None of these

Ans.

(B)

A = (d/a, 0, 0), B = (0, d/a, 0), c = (0,0, d/a)

Area of triangle OAB = 1 = 1/2 d2/|ab|

Area of triangle OBC = 2 = 1/2 d2/|bc|

Area of triangle OAC = 3 = 1/2 d2/|ac|

If area of triangle ABC be , then

54.
Equation of the plane passing through (-1, 1, 4) and containing the line x1/3 = y2/1 = z/5, is:
(Ref.P.K.Sharma_Three Dimen._P.C6.4Q.9)

(A)

9x 22y + 2z + 23 = 0

(C)

9x + 22y - 3z = 1

(B)

(D)

x + 22y + z = 25

22y 9x + z = 35

Ans.

(D)

Equation of any plane containing the line x1/3 = y2/1 = z/5 will be

a(x1) + b (yz) + cz = 0

where,

3a + b + 4c = 0

. (i)

It is given that plane passes through (1, 1, 4).

2a b + 4c = 0

. (ii)

From (i) and (ii), we get

a/9 = b/22 = c/1.

Thus the equation of required plane is,

9(x 1) + 22(y 2) + z = 0

i.e.,

22y 9x + z = 35

55.
Equation of the plane containing the lines x/1 = y2/3 = z+4/1 and x4/2 = y/3 = z/1 is,:
(Ref.P.K.Sharma_Three Dimen._P.C6.5Q.10)

Ans.

(A)

x + y 4z = 6

(B)

x - y + 4z = 6

(C)

x + y + 4z = 6

(D)

None of these

(D)

Equation of any plane containing the line x/1 = y2/3 = z+4/1 is ax + b (y2) + c (z+4) = 0

where a + 3b c = 0

This plane will also contain the second line if

2a 3b + c = 0

and

4a b(0 2) + c(0 + 4) = 0

Solving these equation, we get

a = 0, b = 0, c = 0.

That means the given lines are noncoplanar.

56.
Equation of the plane such that foot of altitude drawn from (-1, 1, 1) to the plane has the
coordinate (3, -2, -1), is:(Ref.P.K.Sharma_Three Dimen._P.C6.5Q.11)

Ans.

(A)

x+y+z=0

(B)

4x - 3y 2z = 20

(C)

3x + y z = 8

(D)

4x + 3y z = 7

(B)

Clearly, the direction ratio of the plane are

4, 3, 2

Thus equation of plane will be

4x 3y 2z = d

It will necessarily pass through (3, 2, 1)

i.e,,

d = 12 + 6 + 2 = 20

Thus the equation of plane is

4x 3y 2z = 20.

57.
The distance of the point (-1, 2, 6) from the line x2/6 = y3/3 = z+ 4/4, is equal to:
(Ref.P.K.Sharma_Three Dimen._P.C6.5Q.13)

Ans.

(A)

7 units

(B)

9 units

(C)

10 units

(D)

12 units

(A)

Any point on the line is

P = (6r1 + 2, 3r1 + 3, 4r1 4).

Direction ration of the line segment PQ, where Q = (1, 2, 6), are

6r1 + 3, 3r1 + 1, 4r, 10.

If P be the foot of altitude drawn from Q to the given line, then

6(6r1 + 3) + 3(3r1 + 1) + 4(4r1 + 10) = 0.

=>

r1 = 1.

Thus, P = (4, 0, 0)

Required distance = 9 + 4 + 36

= 7 units.

58.
The point of intersection of the lines x+1 / 3 = y + 3/5 = z + 5 / 7 and x2/1 = y4/3 = z6/5 is:
(Ref.P.K.Sharma_Three Dimen._P.C6.5Q.14)

Ans.

(A)

(1/3, 1/3, 2/3)

(B)

(1/2, 1/2, 3/2)

(C)

(1/2, 1/2, 1/2)

(D)

(1/3, 1/3, 1/3)

(B)

Any point on the first line is

(3r1 1, 5r1 3, 7r1 5)

and any point on the second line is

(r2 + 2, 3r2 + 4, 5r2 + 6).

At the point of intersection, we must have

3r1 1 = r2 + 2,

5r1 3 = 3r2 + 4.

7r1 5 = 5r1 + 6.

Thus, r1 = 1/2, r2 = 3/2

Hence required point is i.e. (1/2, 1/2, 3/2)


59.
The shortest distance between the line x + y + 2z 3 = 2x + 3y + 4z 4 = 0 and the z-axis is:
(Ref.P.K.Sharma_Three Dimen._P.C6.5Q.15)

Ans.

(A)

1 unit

(B)

2 units

(C)

3 units

(D)

4 units

(B)

We have,

x + y + 2z 3 = 0, x + 2z 2 + 3/2 y = 0

Solving these equations, we get

Y = 2.

Thus required shortest distance is 2 units.

60.
The length of projection, of the line segment joining the points (1, -1, 0) and (-1, 0, 1), to the plane
2x + y + 6z = 1, is equal to:(Ref.P.K.Sharma_Three Dimen._P.C6.5Q.17)

Ans.

(A)

255/61

(B)

237/61

(C)

137/61

(D)

155/61

(B)

Let A = (1, 1, 0), B = (1, 0, 1).

Direction rations of segment AB are

2, 1, 1.

If be the acute angle between segment AB and normal to plane,

Length of projection

= (AB) sin

= 6.19/246 = 237/61 units

61.
Reflection of the line x1/1 = y2/3 = z3/1 in the plane x + y + z = 7 is:(Ref.P.K.Sharma_Three
Dimen._P.C6.5Q.18)

Ans.

(A)

x1/3 = y2/1 = z4/1

(B)

x1/3 = y2/1 = z4/1

(C)

x1/3 = y2/1 = z4/1

(D)

x1/3 = y2/1 = z4/1

(C)

Given line passes through (1, 2, 4) and this point also lies on the given plane.

Thus required line will be in the form of x1/l = y2/m = z4/n.

Any point on the given line is

(r1 + 1, 3r1 + 2, r1 + 4).

If r1 = 1, this point becomes P = (0, 5, 5).

Let Q = (a, b, c) be the reflection of P in the given plane, then

a/2.1 + b+5/2.1 + 5+c/2.1 = 7

i.e,

a + b + c = 4,

and

a/1 = b5/1 = c5/1 = (say)

=> a = , b = 5 + , c = 5 +

=> 10 + 3 = 4

=> = 2

Thus, Q = (2, 3, 3)

Hence direction rations of reflected line are

3, 1, 1

Thus its equation is

x1/3 = y2/1 = z4/1

Anda mungkin juga menyukai